CP2 - ONG

Ace your homework & exams now with Quizwiz!

Detailed understanding of the use of anti-D in prophylaxis of Rhesus isoimmunization

~15% Caucasian women Rh -ve so at risk of developing anti-D antibodies during or immediately after pregnancy - antibodies can cause harm to subsequent fetuses as antibodies can cross placenta & attack RBC of Rh +ve fetus = anaemia, hydrops, neonatal jaundice, kernicterus or fetal death in utero sensitising events include normal delivery, miscarriage, TOP, ectopic pregnancy, invasive prenatal diagnosis, abdo trauma, antepartum haemorrhage, external cephalic version anti-Rh D prophylaxis given at 28 & 34 weeks gestation to prevent maternal immunization to Rh +ve fetus

Demographics and epidemiology of pregnancy

1) <18 conception rate in 2015 in UK was 21/1000 women aged 15 to 17 - lowest rate recorded since comparable statistics were first produced in 1969 2) estimated number of conceptions to women <16 fell to 3,466 in 2015, compared with 4,160 in 2014 3) estimated 880,000 conceptions to women of all ages in 2015 4) conception rates in 2015 inc for women 25+, and dec for women <25 years 5) largest % inc = among women 40+

Screening for fetal abnormality (including chromosomal abnormality)

1) Down's: a) bloods - 10-14 weeks - tests for BhCG + PAPPA b) nuchal transleuceny scan - 11-14 weeks c) if present late - quadruple bloods: hCG (high), AFP (low), oestriol 3 (low), inhibin A (high) d) if US & bloods suggest Down's need amniocentesis or CVS to confirm 2) 20 week scan - most organ systems developing, can look for: a) defects in the heart (4 chamber view) b) NT defects - frontal bone scalloping (lemon sign) + abnormally shaped cerebellum (banana sign) c) urinary tract eg/ Potter's syndrome, polycystic kidneys etc d) GI eg/ oesophageal atresia, obstruction, anterior abdo wall defects e) thoracic abnormalities eg/ congenital diaphragmatic hernia f) severe skeletal dysplasia g) facial/neck abnormalities eg/ cleft lip/palate, Pierre-Robin syndrome - karyotyping may be indicated as associated with chromosomal abnormalities 3) amniocentesis - tests amniotic fluid (shed fetal cells). Used for: a) chromosomal analysis - mainly for Down's b) DNA analysis for certain genetic disorders c) enzyme assay for inborn errors of metabolism d) AFP & acetylcholinesterase e) diagnosis of fetal infection eg/ toxoplasmosis, CMV f) investigation of fetal lung maturity g) bilirubin (for fetal Rh status) 4) chronic vilius sampling/placental biopsy - sampling of acutely dividing trophoblast cells for rapid karyotyping eg/ US suggests aneuploidy, DNA analysis when Fx of inherited diseases 5) antenatal fetal blood sampling - higher risk so only when need to know: fetal Hb, WCC, specific IgM eg/ fetal hydrops, suspected infection, Rh etc 6) fetal tissue sampling - a) skin biopsy: harlequin icthyosis, Sjorgren-Larsson syndrome etc b) liver biopsy: G6PD deficiency etc

Different normal values for pregnancy (non pregnant vs pregnant)

1) FBC - a) dec: Hb, platelets b) inc: WBC 2) renal function - a) dec: urea, creatinine, Na, K, uric acid b) inc: 24hr urinary protein, 24hr creatinine clearance 3) LFTs - a) no change: bilirubin, GGT b) dec: total protein, albumin c) inc: Alk, ALT, bile acids 4) clotting screen - a) inc: fibrinogen 5) TFTs - a) dec: TSH, fT3, fT4

Identification of risk factors for & monitoring of complicated pregnancies

1) HTN 2) PCOS 3) diabetes - gestational if fasting plasma glucose level 5.6+ mmol/l, or 2-hour plasma glucose level of 7.8+ mmol/l 4) kidney diseas - inc risk of miscarriage 5) autoimmune diseases eg/ lupus and MS 6) thyroid disease 7) infertility 8) obesity 9) HIV/AIDS 10) teen pregnancy - inc risk of HTN, pre-term labour & anaemia 11) older first-time mothers - inc risk of C-section, delivery complications eg/ excessive bleeding in labor, prolonged labor (>20 hours), labor that does not advance, infant with Down syndrome 12) alcohol use, cigarette smoking 13) multiple gestation - inc risk of prematurely 14) preeclampsia and eclampsia - sudden inc in BP after 20th week (>140/90) + protein in urine 15) managment - a) at least 400 micrograms of folic acid daily before & through pregnancy b) up to date immunizations c) healthy diet and maintaining proper weight d) regular physical activity, unless advised otherwise by doctor e) avoiding cigarettes, alcohol, and drugs (except for medications approved by doctor) f) regualr obstetrician appointments & scans - can be up to every week (or sometimes every day) - generally every 4 weeks

Effects on pregnancy and the risks to the woman and her fetus of preexisting illness

1) HTN - if uncontrolled can lead to damage to mother's kidneys & inc risk for low birth weight or preeclampsia 2) PCOS - inc risk of miscarriage, gestational diabetes, preeclampsia, and premature delivery 3) diabetes - high blood sugar levels can cause birth defects during first few weeks of pregnancy 4) kidney disease - inc risk of miscarriage 5) autoimmune disease eg/ lupus & MS - inc risk for problems during pregnancy eg/ lupus - inc risk of preterm birth and stillbirth 6) thyroid disease - can cause problems for the fetus eg/ heart failure, poor weight gain, birth defects 7) infertility - women who take drugs that inc chances of pregnancy are significantly more likely to have pregnancy complications than those who get pregnant without assistance - complications often involve placenta & vaginal bleeding 8) HIV/AIDS - can be passed to fetus during pregnancy, labor and giving birth, breastfeeding

Clinical, investigative and psychological principles of recurrent miscarriage

1) Ix - a) day 2-5 FSH & LH b) thrombophilia screen c) anticardiolipin antibodies d) thyroid peroxidase antibodies e) karyotyping (male & female) f) USS (unless already done in previous pregnancy) - only do hysterectomy if USS shows abnormality 2) psychological - traumatic event for couples, can have s/s of of depression, anxiety & dec self-esteem 3) mx - at first +ve pregnancy test: a) aspirin 75mg a day b) cyclogest pessary vaginally daily c) clindamycin cream (2%) applied to vagina for 4 days, repeated every 4 weeks d) if no evidence of thrombophilia continue aspirin to 24 weeks pregnancy, if thrombophilia add enoxaparin & continue aspirin to 36 weeks NB/ miscarriage support groups available - offer an opportunity to meet others who have experienced pregnancy loss, to share thoughts, feelings & experiences in a safe + supportive atmosphere

Issues relating to psychiatric disorders of the puerperium and its management

1) PN psychological symptoms/psychosis - severe episode of mental illness which begins suddenly in the days or weeks after having a baby eg/ mania, depression, confusion, hallucinations and delusions, psychiatric emergency, need to be treated in hospital on mother & baby unit, most women need an antipsychotic, a mood stabiliser or both. In the long term being some may need counselling or couple therapy. Can take 6 -12 months or more to recover 2) PN depression - a) self-help strategies - talk to your partner, friends and family, accept help from others when it's offered and ask your loved ones if they can help look after the baby and do tasks such as housework, cooking and shopping, make time for yourself, rest when you can, exercise regularly, eat regular, healthy meals and don't go for long periods without eating, don't drink alcohol or take drugs, as this can make you feel worse b) therapy - 1st line, Guided self-help: working through a book or an online course on your own or with some help from a therapist (focuses on issues you might be facing, with practical advice on how to deal with them), CBT (aims to break negative cycles and find new ways of thinking that can help you behave in a more positive way), interpersonal therapy (talking to a therapist about the problems you're experiencing) c) medication - antidepressants if moderate or severe depression & don't want psychological treatment or psychological treatment doesn't help, usually need to be taken for at least a week before benefits starts to be felt, usually need to take them for around six months after starting to feel better d) if very severe will be referred to mental health team for - more intensive CBT, other psychological treatments eg/ psychotherapy, therapies such as baby massage to help you bond better with your baby, different medications, electroconvulsive therapy (ECT) e) charities and support groups eg/ National Childbirth Trust, Mind, the mental health charity NB/ different from PP blues - only lasts ~2 days, starts a couple days after delivery 3) PN psychosis - peaks at 2 weeks PP, mostly primiparous, can be depressive, manic or schizophrenic, 5% suicide & 4% infanticide risk, pt needs to be hospitalised with baby in mother & baby units, antidepressants + antipsychotics + ECT 4) bipolar - lithium is excreted at high levels in mother's milk, and infant serum levels are about one-third to one-half of the mother's serum levels. Signs of toxicity - cyanosis, hypotonia, and hypothermia in babies. Although breastfeeding typically is avoided in women taking lithium, some choose to use lithium while nursing - must use lowest possible effective dose & maternal + infant serum lithium levels measured, alongside infant TSH, blood urea nitrogen, and creatinine 5) PP anaemia - iron-rich diet, avoid excess calcium-rich food as it inhibits the absorption of iron, iron supplements, may need IV iron, in an extreme scenario may need blood transfusion

Implications of the small-for-date and large-for-date babies

1) SGA - prognosis depends on if they are constitutionally small, small because of IUGR, or due to non-placenta mediated growth restriction (eg/ structural or chromosomal anomaly, inborn errors of metabolism or fetal infection) a) constitutionally SGA babies usually have an excellent prognosis assuming there is no other health problem b) prognosis for babies with non-placenta mediated growth restriction depends on underlying condition c) IUGR babies prone to complications after birth eg/ perinatal asphyxia, meconium aspiration, persistent pulmonary hypertension, hypothermia, hypoglycaemia, hyperglycaemia, hypocalcaemia, polycythaemia, jaundice, feeding difficulties, feed intolerance, necrotising enterocolitis, late-onset sepsis and pulmonary haemorrhage d) IUGR babies have inc risk of neuro-behavioural abnormalities, poor growth and inc susceptibility to adult-onset diseases in infancy and adolescence, including obesity, metabolic syndrome, type 2 diabetes and cardiovascular disease. e) long term complications of SGA babies - inc risk of attaining an adult height below their target height, as well as of developing metabolic disorders (obesity, metabolic syndrome, type 2 diabetes) and CV diseases. Also prone to precocious pubarche, exaggerated precocious adrenarche, earlier onset of menarche, faster progression of puberty than children born appropriate for gestational age f) inc risk of other long term complications eg/ HTN, hypercholesterolaemia, kidney disease, liver disease, lung abnormalities: reactive airways disease, cancer: breast, ovarian, colon, lung, blood, schizophrenia, Parkinsonism, Alzheimer's, PCOS, shortened lifespan, depression, anxiety, bipolar disorder, immune dysfunction, osteoporosis, social problems 2) LGA - most common cause is diabetic mother. Common risks - shoulder dystocia, hypoglycemia, metatarsus adductus, hip subluxation and talipes calcaneovalgus due to intrauterine deformation a) delivery complications: shoulder dystocia (anterior shoulder becoming impacted on maternal symphysis pubis), fracture of clavicles or limbs, perinatal asphyxia b) proportional inc in morbidity and mortality due to the following: respiratory distress (and need for ventilatory assistance), meconium aspiration, hypoglycaemia (independent of whether mother has diabetes), polycythaemia c) infants of diabetic mothers (IDMs) at inc risk of: hypoglycaemia, hypomagnesemia, polycythaemia, hyperbilirubinemia, RDS, certain congenital anomalies eg/ congential heart disease (hypertrophic cardiomyopathy, ventricular septal defect, transposition of the great arteries, and aortic stenosis), caudal regression syndrome, spina bifida, small left colon syndrome

Impact of social problems on pregnancy outcome

1) Saving Mothers' Lives (2007) - socially excluded women have inc risk of death during or after pregnancy than other women. Vulnerable women with socially complex lives who died were far less likely to seek antenatal care early in pregnancy or to stay in regular contact with maternity services - more likely to be: black African or Caribbean; experiencing domestic abuse; substance misusers; known to social services or child protection services; or unemployed 2) Maternity Matters (2007) - women in all vulnerable groups are likely to recognise their pregnancy later, to first see a health professional later and to book later for antenatal care 3) Perinatal Mortality (2009) - women from non-white ethnic groups and in the most deprived population quintile had stillbirth and neonatal death rates 2x those of white women and those resident in the least deprived areas. Maternal stress in pregnancy has a detrimental effect on subsequent childhood development

Issues involved in the diagnosis and management of pregnancy complicated by of cardiac disease

1) diagnosis - can be difficult as s/s of cardiac disease eg/ SOB, palpitations & syncope, bounding pulse, systolic murmur etc are mimicked by pregnancy, diagnose with echo, ECG etc 2) management - multi-disciplinary team, stressors eg/ anaemia & infection should be minimized, meds and anti-coagulation may be needed, fetal growth scans + Doppler for cardiac defects, in labour minimize pain & get fluid balance correct, careful surveillance just after birth NB/ pregnancy puts strain on CV system - rise in cardiac output NB2/ inc risk of - preeclampsia, IUGR, preterm, fetal loss - ACEI & warfarin teratogenic

Use of ultrasound and endocrine assessment in early pregnancy problems

1) USS can show: babies heartbeat, no of babies, detect an ectopic, find out the cause of any bleeding, accurately date pregnancy, assess risk of Down's, help with diagnostic tests eg/ chorionic villus sampling (CVS) or amniocentesis, examine babies organs, diagnose most abnormalities, assess amount of amniotic fluid 2) endocrine - endocrinology of pregnancy involves endocrine & metabolic changes as a consequence of physiological alterations at the foetoplacental boundary between mother and foetus. Neuroendocrine events and their timing in the placental, foetal and maternal compartments are critical for initiation and maintenance of pregnancy, for foetal growth and development, and for parturition. As pregnancy advances trophoblast no inc & foeto-maternal exchange begins to be dominated by secretory function of the placenta. As gestation progresses toward term, the number of cytotrophoblasts again declines and the remaining syncytial layer becomes thin and barely visible. This arrangement facilitates transport of compounds including hormones and their precursors across the foeto-maternal interface. The endocrine system is the earliest system developing in foetal life, and it is functional from early intrauterine existence through old age. Regulation of the foetal endocrine system relies, to some extent, on precursors secreted by placenta and/or mother

Advantages and disadvantages of surgical interventions for subfertility, including adhesiolysis, surgical management of endometriosis, myomectomy

1) adhesiolysis - adhesions are bands of scar tissue that form between organs eg/ after abdominal surgery or bout of intra-abdominal infection (PID, diverticulitis). Can cause problems with infertility, bowel obstruction, pain etc a) laparotomy with open adhesiolysis has been treatment of choice for acute complete bowel obstructions (some pts with partial obstructions may also require surgery if nonoperative measures fail) I) but often leads to formation of new intra-abdominal adhesions in 10-30% pts II) indications of laparoscopic adhesiolysis - pts with complete small-bowel obstruction or partial small-bowel obstruction not resolving with nonoperative therapy, but without signs of peritonitis or bowel perforation or ischemia, pts with resolved bowel obstruction but hx of recurrent, chronic small-bowel obstruction demonstrated by a contrast study III) contraindications to laparoscopic adhesiolysis - acute perforation and peritonitis, necessitating bowel resection and handling of severely inflamed organs, massive abdo distention that precludes insufflation and a sufficient working space during laparoscopy, haemodynamic instability, pts unable to tolerate pneumoperitoneum because of severe comorbid conditions of the heart and lung 2) endometriosis - found in 25% laparoscopes for ix infertility. More sveer endometriosis = greater chance of infertility a) medical mx - analgesia, COCs, progestogens, GnRH analogues +/- HRT, IUS, if fallopian tubes unaffected medical tx will not inc fertility, but laparoscopic excision or ablation may b) surgical mx - I) scissors, laser or bipolar diathermy (laparoscopic ablation +/- adhesiolysis) - can be used laparoscopically to destroy endometriotic lesions. Surgery may improve conception rates so preferable to medical tx for women with endometriosis pain & infertility II) more radical surgery - dissection of adhesions & removal of ovarian endometriomas, or even hysterectomy with bilateral salpingo-oopherectomy (BSO). When treating ovarian endometriotic cysts - open & drain fluid, stripping cyst wall associated with lower cyst recurrence rate & higher spontaneous conception rate, bit chance of causing ovarian damage III) surgery can be very difficult due to severity of adhesions & anatomical distortion. Risks of damaging bowel, bladder, blood vessels & ureters. Symptomatic improvement in 70% pts & longer term than medical therapy IV) with severe disease affecting fallopian tubes, surgery has limited benefit - IVF is best option V) hysterectomy = last resort & only if women's family is complete. HRT needed is ovaries removed (however if endometriosis remains give combined (oestrogen + progesterone) preparation 3) myomectomy - removal of fibroids from myometrium once medical mx has failed. Removed via powered morecellation in which morcellator instrument, introduced through laparoscopic ports apple cores the fibroid a) open or laparoscopic (<4 fibroids or <8cm diameter) b) used if fibroids causing symptoms but fertility still required and uterine artery embolization not desired c) GnRH agonists or ulipsrital acetate used preoperatively (for 2-3 months) to dec size of fibroids. Perioperative injection of vasopressin directly into myometrium dec blood loss d) complications - I) blood loss may be heavy (risk of blood transfusion or hysterectomy to save life) II) small fibroids can be missed, new fibroids can develop III) adhesions can form at site of myomectomy - dec fertility IV) if endometrial cavity opened during myomectomy (or fibroids multiple or large) than C-section indicated in future pregnancies (risk of uterine rupture during labour) V) if an unrecognised leiomyosarcoma is morcellated, risk of malignancy spread

Hyperemesis in pregnancy

1) aetiology - 1% pregnancies, but general N+V common (>50%) 2) pathology - level of hCG directly related to severity - higher incidence in molar + multiple pregnancies 3) s/s - N+V, dehydration, electrolyte imbalance, ketonuria, occasionally muscle wasting, rarely may precipitate thyrotoxic crisis or Wernicke's encephalopathy (vit B1 deficiency) 4) ix - FBC, U&E, LFT, TFT, pelvic USS, urine microscopy, MC&S 5) mx - inpatient or outpatient, fluids, antiemetics, K+ replacement, thromboprophylaxis, vit B1 replacement NB/ usually improves by 12 weeks gestation as HCG dec, severe cases = prolonged hospital stay & in rare cases TPN

Detailed knowledge and understanding of aetiology, recognition and management of primary and secondary postpartum haemorrhage

1) aetiology - 4 Ts: a) tone - uterine atony b) trauma - cervical, vaginal or perineal tears c) tissue - retained placenta or membranes d) thrombin - coagulation disorder 2) types a) primary PPH = >500mL blood loss within 24hrs delivery b) secondary PPH = >500mL blood loss after 24hrs delivery c) risks - prolonged delivery, multiparity, previous PPH, assisted delivery 3) recognition - PV bleeding evident, dependent on amount of blood loss & preceding Hb levels pt may have - tachycardia, tachypnoea, hypotension 4) mx - obstetric emergency - resuscitation of pt & treat cause a) fluid resuscitation b) uterine massage c) tocolytics - oxytocin, then ergometrine, then misoprostol d) exam to exclude retained tissue & perineal trauma e) if simple measures fail need surgery - insertion of intrauterine balloon, uterine artery embolisation, uterine artery ligation, hysterectomy

Aetiology and clinical characteristics (including symptoms) of utero-vaginal prolapse

1) aetiology - pelvic organs mainly supported by levator ani muscles & endopelvic fascia (connective tissue network connecting organs to pelvic muscles & bones) - GU prolapse occurs when this support structure is weakened through direct muscle trauma, neuropathic injury, disruption or stretching. A multifactorial cause is likely a) confirmed risk factors - inc age, vaginal delivery, inc parity, obesity, previous hysterectomy b) possible risk factors - obstetric factors eg/ prolonged 2nd stage of labour, inc birth weight, pregnancy itself, forceps, <25 years at first delivery, shape of pelvis, Fx of prolapse, constipation, CT disorders eg/ Marfan's syndrome, Ehlers-Danlos syndrome, occupations involving heavy lifting 2) clincal - may be asymptomatic and an incidental finding, or can severely affect QOL a) vaginal/general symptoms common to all types of prolapse eg/ sensation of pressure, fullness or heaviness, bulge/protrusion or 'something coming down' - may be visible, difficulty retaining tampons, spotting (if ulceration of prolapse) b) urinary symptoms eg/ incontinence, frequency, urgency, feeling of incomplete bladder emptying, weak or prolonged urinary stream, the need to dec prolapse manually before voiding, need to change position to start or complete voiding c) coital difficulty eg/ dyspareunia, loss of vaginal sensation, vaginal flatus, loss of arousal, change in body image d) bowel symptoms eg/ constipation/straining, urgency, incontinence of flatus or stool, incomplete evacuation, need to apply digital pressure to perineum or posterior vaginal wall to enable defecation (splinting), digital evacuation necessary in order to pass a stool e) exam - I) examine in standing & L lateral position, ask woman to strain & observe both whilst standing and supine II) Sims' speculum inserted along posterior vaginal wall to assess anterior wall & vaginal vault. Ask pt to strain III) bivalve speculum to identify the cervix or vaginal vault. Ask pt to strain, and slowly remove speculum. Look for degree of descent of vaginal apex - determine parts of vagina (anterior, posterior or apical) that prolapse affects IV) DRE if bowel symptoms

Detailed knowledge and understanding of the aetiology, diagnosis, management and implications of thromboembolism - detection and prevention

1) aetiology - pregnancy causes prothrombotic state - inc coagulation factors, endogenous anticoagulants dec, fibrinolysis dec - starts in 1st trimester & lasts until a few weeks after birth, venous stasis in lower limbs from compression on pelvic vessels exacerbates problem 2) diagnosis - DVT with swelling & tenderness of leg, PE with SOB and pleuritic chest pain or collapse a) tests - DVT: D-dimer test as -ve predictive value is high, Doppler US of lower limb veins or MRI of pelvic veins; PE: D-dimer, spinal artery CT or venous perfusion scanning b) management - risk assessment for VTE, high risk = antenatal prophylaxis + graduated elastic compression stockings + LMWH, acute episode = prompt heparin therapy, avoid dehydration c) implications - main problems are with medications (LMWH safe as don't cross placenta, warfarin = teratogenic 1st & 3rd trimester), risk remains high throughout pregnancy & afterwards for some weeks (need to continue therapy 6 weeks post-partum), problems if not stopped at birth (PPH)

Anatomy and physiology of genitourinary tract & maintenance of urinary continence in women

1) anatomy a) uterus, bladder + rectum supported by pelvic floor - encompasses levator ani + coccygeal + internal obturator + piriformis + 2 transverse perineal muscles + pelvic fascia b) ligaments that support uterus - transverse cervical/cardinal, uterosacral, pubocervical, pubourethral (round + broad don't provide much support) 2) maintenance of continence - a) urethra = 4 cm long in females & imbedded in connective tissue supporting anterior vagina. Urethra composed of inner epithelial lining, spongy submucosa, middle SM layer, and outer fibroelastic connective-tissue layer. Spongy submucosa contains rich vascular plexus responsible, in part, for providing adequate urethral occlusive pressure. Urethral SM & fibroelastic connective tissues circumferentially augment the occlusive pressure generated by the submucosa. Thus, all structural components of the urethra, including the striated sphincter muscle, contribute to its ability to coapt and prevent urine leakage b) inner mucosal lining keeps urothelium moist & urethra supple. Vascular spongy coat produces mucus important in mucosal seal mechanism. Compression from middle muscular coat helps maintain resting urethral closure mechanism. Outer seromuscular layer augments the closure pressure provided by the muscular layer c) urethral epithelium composed of stratified squamous cells, which becomes transitional as the bladder is approached. The epithelium is arranged in longitudinal folds - at the base of the folds are scattered gland openings along the entire urethral length. The epithelium is supported by a loose lamina propria consisting of collagen fibrils and elastic fibers, arranged both circumferentially and longitudinally. A rich network of blood vessels is in the subepithelial layer d) SM of urethra is arranged longitudinally & obliquely (few circular fibers). Nerve supply is cholinergic & a-adrenergic. Longitudinal muscles may contribute to shortening and opening of urethra during voiding. Oblique & circular fibers contribute to urethral closure at rest e) voluntary urethral sphincter = group of circular muscle fibers and muscular loops within pelvic floor. Innermost layer (proximal 2/3 of urethra) = sphincter urethrae. More distally = compressor urethrae & urethrovaginal sphincter - striated muscles composed primarily of slow-twitch muscle fibers to maintain resting urethral closure & contribute to voluntary closure and reflex closure of urethra during acute instances (eg/ coughing, sneezing, laughing) of inc abdo pressure. Medial pubovisceral portion of levator ani also contributes to active bladder neck & urethral closure in similar situations f) urethral meatus empties into vaginal vestibule after distal urethra pierces the perineal membrane. The mucosa of the meatus is continuous with that of the vulva g) posterior wall of urethra is embedded in & supported by endopelvic connective tissue (collagen, elastin, & SM). Connective tissue envelops the anterior vagina. This supportive tissue has been likened to a sling or a hammock around the urethra and bladder neck. Endopelvic CT attached to perineal membrane ventrally & laterally to levator ani muscles by way of the arcus tendinous fascia pelvis (CT which extends bilaterally from inferior part of pubic bone along junction of the fascia of the obturator internus & levator ani muscle group to near the ischial spine). Stress incontinence may be associated with a deficiency in the hammocklike support of the endopelvic CT h) similar to urethral sphincter muscle groups, the fast-twitch fibers of the levator ani complex aid in suddenly stopping the urinary stream during voluntary guarding reflex i) anterior distal wall of urethra is attached to pubic bone by pubourethral ligaments (extensions of perineal membrane + caudal & ventral portions of arcus tendinous fascia pelvis.) Ligaments may limit movement of the anterior wall of the urethra during increases in intra-abdominal pressure but probably exert a lesser degree of support to the posterior wall

Ethical and legal issues relating to fertility control

1) at present, the law is playing "catch-up" with science & reproductive law has emerged 2) nowadays embryo, sperm and egg donations, and gestational carrier arrangements have become viable 3) until quite recently, it was standard to name the woman who physically delivered the child as the mother on the birth certificate - these laws are becoming more and more archaic and obsolete 4) ethical & legal issues vary between countries and with religious beliefs a) countries that have laws and statutes applying to fertility treatment legal issues include Canada, France, Germany, UK and Finland b) using donor sperm or eggs - no anonymity of donors in UK after age of 18 c) commercial surrogacy is not permitted in UK 5) laws in the UK - Human Fertilisation and Embryology Authorisation (HFEA) Act 1990: a) limits the number of embryos allowed to be transferred for IVF treatment to 2 embryos, 3 if >40 b) embryo freezing is permissible but limited to 10 years, providing yearly costs are made c) mixing eggs, sperm or embryos for more than one person is currently forbidden in the UK d) sex selection is prohinited unless done to avoid the risk of genetic transmission of sex-linked diseases e) donated sperm stored for 6 months before it can be used, to screen donor for transmitted infections f) any fertility centre in the UK must take into account the welfare of any child who may be born as a result of a treatment, and of any other children who may be affected by the birth

Detailed knowledge and understanding of emergency investigation and management of ovarian cysts

1) before menopasue most ovarian masses are benign (99%), incidence inc as age inc (post-menopasual risk = 30%) 2) symptoms - abdo distension or bloating or pain, urinary frequency, postmenopausal bleeding, loss of appetite, rectal bleeding 3) US - TVUS, sometimes TA US a) functional ovarian cysts - unilateral, unilocular and thin walled, <5cm diameter. Solid component is most significant feature of malignancy b) sonographic characteristics associated with ovarian malignancy: I) solid component, often nodular or papillary II) septations, if present, that are thick (>2 to 3 mm) III) presence of ascites IV) peritoneal masses V) enlarged lymph nodes 4) MRI/CT/PETCT - if findings on US cannot characterize nature & origin of adnexal mass 5) tumour marker - not diagnostic but can help to characterise ovarian mass a) CA 125: inc in 80% women with ovarian cancer. CA 125 also inc in pts with other conditions eg/ endometriosis, pelvic infection, fibroids, diverticulitis 6) mx - 'risk of malignancy index' (RMI) shows pts at greater risk of malignancy a) if RMI > 200 need gynaecological oncology MDT b) RMI is a clinical prediction rule based on ultrasound, CA-125, and menopausal status defined as: RMI = U x M x CA-125 I) US score - 1 for each of: multilocular cyst, solid areas, metastases, ascites, bilateral lesions - U = 0 if none, U = 1 for US score of 1, U = 3 for ultrasound score ≥ 2 II) CA-125 = serum CA-125 in kU/L III) menopausal status (M = 1 if premenopausal and M = 3 if postmenopausal) c) asymptomatic ovarian cysts = no tx - observation eg/ premenopausal simple ovarian cysts < 5cm, postmenopausal simple ovarian cysts < 1cm d) conservative mx - US features suggestive of functional ovarian cyst, premenopausal simple cysts ≤7 cm with no other features of malignancy I) follow up in 6 months (US & Ca125) e) surgery - if cyst does not meet criteria for conservative mx - if postmenopausal should involve salpingoophorectomy (usually bilateral)

Booking visit and routine investigations

1) booking visit - should be in 10-14 weeks of pregnancy a) measure maternal height & weight - low BMI predicts inc risk of fetal growth restriction & perinatal mortality, high BMI predicts ante and intrapartum risks b) BP measured c) full antenatal history & routine clinical exam 2) routine ix - a) Hb conc + FBC, repeated at 28 and 34 weeks gestation (anaemia) b) blood group & antibodies screening for Rh status - if -ve then screen for Rh antibodies at first visit & at 28 weeks c) infection screening: rubella, syphilis, hep B+C, HIV d) GBS: low vaginal swab at 34-36 weeks (not universal) e) UTIs: associated with pregnancy loss + preterm labour f) gestational diabetes: I) 2hr 75g OGTT at booking visit (& 28 weeks if doubt) if high risk eg/ 1st degree relative with diabetes, previous pregnancy with GD, BMI >35 etc II) universal screening at 26-28 weeks - modified GTT with 50g dose in 1hr (+ve if blood glucose >7.7mmol/L, then followed by formal GTT)

Advise regarding postpartum contraception

1) breastfeeding - suckling an infant dec release of gonadotrophins, which suppress ovulation but, as suckling reduces, ovulation returns. Over 98% effective in preventing pregnancy if a woman is: <6 months postpartum, amenorrhoeic (no vaginal bleeding after the first 56 days postpartum), fully breast-feeding day (at least 4-hourly feeds) and night (at least six-hourly feeds) 2) non breast-feeding women <21 days postpartum: a) unrestricted - POP, progestogen-only injectables and implants, barrier methods b) not recommended or restricted - COC, combined contraceptive patch and combined contraceptive vaginal ring, c) IUCD & IUS - unless fitted within 48hrs of birth, delay until after four weeks postpartum d) sterilisation - usually delayed until at least six weeks post delivery 3) non breast-feeding women ≥21 days postpartum: a) unrestricted - COC, combined contraceptive patch, combined contraceptive vaginal ring, POP, progestogen-only injectables and implants, barrier methods 4) fully or almost fully breast-feeding women between 6 weeks and 6 months postpartum - LAM, POP, progestogen-only injectables and implants, copper IUCD & IUS, barrier methods, sterilisation 5) breast-feeding women >6 months postpartum - can use any method

Detailed understanding of the general principles of postoperative care including wound healing

1) caesarian section - a) type of incision - normally Pfannenstiel incision: cut on horizontal (slightly curved) line just above pubic symphysis ("bikini line incision") b) TWOC when ready c) uterine involution, lochia, BP, temperature, pulse & any perineal wounds checked daily d) pelvic floor exercises e) any pain managed 2) signs of DVT or infection - inc pain, swelling, redness, or warmth around affected area, drainage of pus from area, fever, inc HR, dec BP 3) signs of bleeding post-op - inc HR, new nausea, dizziness, agitation, visible bleeding, or dec urine output, dec BP. Most sensitive sign = inc RR 4) general - a) preventing DVT - mobilise early, TED stockings, higher risk mothers will have anticoagulation injection b) need to be able to pass urine & bowels normally before leaving hospital

In normal pregnancy, to demonstrate understanding of preconceptual care, the use of lifestyle changes, folic acid, nutritional requirements

1) eating sensibly & exercising moderately - if extremes of body weight fertility may be affected & have problems in pregnancy 2) pregnant women need an extra 350 kcal per day 3) need to avoid undercooked meat & egg, pates, soft cheese, shellfish, raw fish, unpasteurized cheese (listeria) 4) periconceptual multivitamins including folic acid (400micrograms) 5) too much vit A (>150,000IU) associated with fetal growth restriction & urinary tract abnormalities 6) low zinc = inc risk of pre and post-term labour & IUGR, eat leafy & root vegetables, whoe grains & nuts, milk & dairy 7) avoid alcohol & smoking 8) iron may be prescribed when needed 9) avoid use of drugs 10) should be advised to relax when possible

Detailed knowledge of the role of cervical and endometrial biopsy

1) cervical a) CIN has no symptoms & not visible to cervix b) cervical smears - performed on all women from 25, then repeated every 3 years until 49. Between 50-64 every 5 years, then from 65 if haven't been screened since 50 or recent abnormal tests. Tested using LBC which also allows testing of HPV NB/ women <25 often have abnormal cervical changes, but risk of cervical cancer is very low c) dyskariosis suggests presence of CIN: if borderline or low grade - HPV -ve = back to routine recall, HPV +ve = colposcopy, if high grade then colposcopy d) colposcopy - cervix inspected with speculum using an operating microscope with 10-20x magnification. Areas of suspected change stained with 5% acetic acid - turn white if changes. however, need histological diagnosis for confirmation, so biopsy needed e) CIN II or III - transformation zone excised with cutting diathermy under LA = large loop excision of transformation zone (LLETZ). Enables diagnosis & tx to be achieved at same time - has replaced laser or diathermy ablation treatment. Otherwise a small biopsy of abnormal area can be taken colopscopically & confirmatory results awaited before performing LLETZ f) LLETZ complications - post-op haemorrhage, subsequent preterm labour proportional to depth of excision 2) endometrial - medical procedure that involves taking a tissue sample of the lining of the uterus. The tissue subsequently undergoes a histologic evaluation which aids the physician in forming a diagnosis a) indications for obtaining an endometrial biopsy in a non-pregnant woman: women with chronic anovulation eg/ PCOS, women with abnormal vaginal bleeding, pts with suspected uterine cancer, in female infertility b) TV US generally done before endometrial biopsy as may help in gynecologic diagnosis, or even make the taking of a biospy superfluous if the lining is thin. If the endometrial lining is less than 5 mm thick on sonography, it is highly unusual to encounter endometrial cancer c) test usually done in women >35. A more thorough histologic evaluation can be obtained by a dilatation and curettage, which requires anesthesia d) contraindications - pregnancy, PID, coagulopathies e) risks - generally considered safe, cramps or pelvic pain is a common if short-lived SE. Afterwards pt may experience some bleeding. A uterine perforation or infection are rare complications f) endometrial biopsy pain is quite common

Risk factors, symptoms, investigation and management of both the preclinical and clinical phases of invasive squamous cell carcinoma

1) cervical cancer = 3rd most common gynaecological cancer (after uterine & ovarian). 3000 cases/yr diagnosed in UK & 1000 deaths/year. 90% malignancies are squamous cell carcinomas 2) risk factors - same as for CIN (pre-invasive stage) eg/ HPV infection, immunosuppression (eg/ HIV or steroids) - not familial 3) s/s - a) occult carcinoma - no symptoms, diagnosis made by biopsy or LLETZ b) clinical carcinoma - I) bleeding - postcoital +/- IMB +/- postmenopausal II) offensive vaginal discharge III) pain not an early feature IV) later involvement of ureters, bladder, rectum & nerves causes uraemia, haematuria, rectal bleeding & pain respectively V) exam - ulcer or mass may be visible or palpable on cervix (early on cervix may appear normal) 4) staging - a) stage 1 - lesions confined to cervix b) stage 2 - invasion into vagina, but not pelvic side wall c) stage 3 - invasion of lower vagina or pelvic wall, or causing ureteric obstruction d) stage 4 - invasion of bladder or lower rectal mucosa or beyond true pelvis 5) ix - a) to confirm diagnosis - tumour is biopsied b) to stage disease - vaginal & rectal exams to assess size of lesion, & parametrial or rectal invasion. Unless clearly small, exam under anaesthetic performed. Cystoscopy detect bladder involvement, MRI detects tumour size, spread, and LN involvement c) to assess fitness for surgery - CXR, FBC, U&Es checked. May be abnormal with advanced disease. Blood cross-matched before surgery 6) tx - a) stage 1a - cone biopsy (low risk of LN spread). Complications - post-op haemorrhage, preterm labour in future pregnancies. Simple hysterectomy preferred in older women b) all other stage 1 + 2a - choice between surgery & chemo-radiotherapy I) chemo-radiotherapy if LN involved II) radical hysterectomy (Wertheim's) - pelvic node clearance, hysterectomy, removal of parametrium & upper 1/3rd of vagina. Ovaries left in young women. Complications - haemorrhage, ureteric & bladder damage & fistula, voiding problems & accumulation of lymph III) radical trachelectomy - less invasive, conserves fertility. Removal of 80% cervix & upper vagina. Laparoscopic pelvic lymphadenectomy done first, if nodes +ve, chemo-radiotherapy performed instead. Cervical suture inserted to help prevent preterm delivery. If excision margins incomplete chemo-radiotherapy needed IV) stage 2b+ - radiotherapy & chemotherapy eg/ platinum agents - dec recurrence & inc survival. Palliative radiotherapy for bone pain or haemorrhage V) recurrent tumours - chemo-radiotherapy if not used before, otherwise pelvic exenteration considered if disease central - removal of vagina (uterus + cervix), bladder +/- rectum, 30% success rate. Preop MRI & PET to look for metastases

Screening and management of premalignant conditions of the cervix

1) cervical screening/smears - from 25-49 every 3 years, from 50-64 every 5 years, over 65 - only if not screened since 50 or recently had abnormal tests 2) speculum inserted & swabs taken 3) changes in cells of the cervix often caused by HPV. High risk = HPV-16 & 18 4) if a sample taken during cervical screening shows low-grade or borderline cell abnormalities, sample is tested for HPV. If HPV is found pt is referred for colposcopy for further investigation and, if necessary, treatment 5) if no HPV is found pts has normal routine screening 6) if sample has significant cell changes pt referred for colposcopy without HPV testing 7) at colposcopy cervix is viewed, acetic acid put onto cervix - glow white with abnormal cells, biopsy taken of abnormal cells (eg/ upper & lower lips) 8) may remove cells if abnormal or shown to be by biopsy - a) large loop excision of transformation zone (LLETZ) - heated wire loop used to remove abnormal cells, normally awake with local on cervix, home same day b) cone biopsy - cone-shaped piece of tissue containing abnormal cells is cut out from cervix, under GA, may need to stay in hospital overnight

Short- and long-term complications of therapies for gynaecological cancers

1) chemotherapy - a) anaemia & leucopenia & thrombocytopenia b) N+V - give antiemetics eg/ ondansetron c) hair loss - need wig within 3 weeks of treatment d) renal toxicity eg/ with cisplatin e) neurotoxicity eg/ cisplatin & taxanes, vincristine f) cardiomyopathy eg/ doxorubicin g) palmar-plantar erythrodysaesthesia eg/ caelyx h) cystitis eg/ cyclophosphamide & ifosfamide i) inc methotrexte toxicity eg/ sulfonamides, aspirin j) lung fibrosis eg/ bleomycin 2) radiotherapy - a) lethargy b) diarrhoea & frequency of micturition in last 7 days c) vagina stenosis d) fistula in bladder, rectum or small bowel, skin changes (all rare)

Detailed knowledge and understanding of the emergency investigation and management of acute unscheduled vaginal bleeding

1) chronic AUB - bleeding abnormal in volume, regularity and/or timing, present at least 3 months a) if cyclical = adenomyosis, fibroids, coagulopathy or endometrial pathology b) totally unpredictable bleeding = ovulatory disturbances - endometrial hyperplasia or malignancy 2) acute AUB - episode of sufficient volume to require urgent or emergency intervention 3) causes of AUB - uterine polyp, adenomyosis, fibroids, malignancy/hyperplasia, coagulopathy, ovulatory dysfunction, endometrial, iatrogenic NB/ intermenstrual bleeding normally due to polyp 4) ix - a) hx - gynae hx, sudden weight loss b) exam - look for obesity, bruising, endocrine problems, anaemia, inspection of perineum for local causes, speculum exam (cervical polpy, ectropion, cervical tumour, cervicitis), bimanual exam (pelvic mass, fibroid, adenomyosis) c) exclude pregnancy d) measure Hb, serum ferritin, thyroid function & luteal phase progesterone e) coagulation studies f) evaluate uterus - endometrial biopsy, US (endometrial thickness), hysteroscopy 5) mx - a) mirena IUS b) tranexamic acid, danazol c) cyclooxygenase inhibitors d) progesterone - noretheristone, depo-provera e) combined OCP f) trial mx in primary care for 3-6 months, if no success, refer to secondary care - endometrial ablation, hysterectomy NB/ modestly heavy to excessive bleeding associated with hypovolumeic shock, if pt unstable - refer into hospital

Knowledge and experience of pelvic endoscopy (colposcopy, hysteroscopy)

1) colposcopy - medical diagnostic procedure to examine an illuminated, magnified view of the cervix & tissues of the vagina and vulva. Many premalignant lesions and malignant lesions in these areas have discernible characteristics which can be detected through the exam done using a colposcope, which provides an enlarged view of the areas, allowing the colposcopist to visually distinguish normal from abnormal appearing tissue and take directed biopsies for further pathological examination main goal is to prevent cervical cancer by detecting precancerous lesions early and treating them 2) hysteroscopy - inspection of uterine cavity by endoscopy with access through the cervix. Allows for the diagnosis of intrauterine pathology and a method for surgical intervention (operative hysteroscopy) a) hysteroscope - endoscope that carries optical and light channels or fibers. Introduced in a sheath that provides an inflow and outflow channel for insufflation of the uterine cavity. In addition, an operative channel may be present to introduce scissors, graspers or biopsy instruments. Hysteroscopic resectoscope allows entry of an electric loop to shave off tissue eg/ fibroid b) simple operative hysteroscopy don't need analgesics, but [aracervical block may be used using a Lidocaine injection in upper part of cervix. Pt in lithotomy position during procedure. c) diameter of hysteroscope generally too large to conveniently pass cervix directly - needs cervical dilation prior to insertion eg/ misoprostol prior to hysteroscopy d) hysteroscope inserted TV into uterine cavity, cavity insufflated (fluids or CO2) & inspection is performed. e) typical procedures - endometrial ablation, submucosal fibroid resection, and endometrial polypectomy

Diagnosis, implications and management of molar pregnancy

1) complete mole: contains no fetal tissue, 90% are 46,XX, divided into 2 types, androgenetic complete mole and biparental complete mole a) androgenetic complete mole - homozygous = 2 identical paternal chromosome complements, from duplication of paternal haploid chromosomes. Affected individuals are always female, heterozygous - male or female, all chromosomes are paternal origin, most likely due to dispermy b) biparental complete mole - rare, maternal & paternal genes present but failure of maternal imprinting so only paternal genome expressed 2) partial mole - fetal tissue often present; fetal erythrocytes & vessels in villi common. 69,XXX or 69,XXY, from fertilization of a haploid ovum & duplication of paternal haploid chromosomes or dispermy. As in a complete mole, hyperplastic trophoblastic tissue and swelling of the chorionic villi occur 3) diagnosis - a) hCG >100,000 mIU/mL indicate exuberant trophoblastic growth & raise suspicion for a molar pregnancy b) biochemical hyperthyroidism can happen c) serum inhibin A and activin A levels - 7-10x higher in molar pregnancies d) imaging - USS to identify complete & partial molar pregnancies: snowstorm pattern (hydropic chorionic villi). Complex intrauterine mass containing many small cysts e) histology - complete mole = oedematous placental villi, hyperplasia of trophoblasts, and lack of fetal blood vessels; partial mole = fetal tissue often present 4) implications - a) ~20% women with a complete mole develop a trophoblastic malignancy but these are almost 100% curable b) perforation of uterus during suction curettage c) haemorrhage is a frequent complication during evacuation d) factors released by molar tissue could trigger the coagulation cascade e) trophoblastic embolism could cause acute respiratory insufficiency 5) management - a) medical - stabilize pt, blood (anaemia), correct any coagulopathy, treat HTN, watch for thyroid storm (rare), give Rh Ig to nonsensitized RhD-ve women (partial moles) b) gynecologic oncologist consulted if pt at risk for malignant disease c) pelvic rest for 2-4 weeks after evacuation, and pt instructed not to become pregnant for 6 months - needs effective contraception d) monitor serial B-hCG levels to identify pts who develop malignant disease e) surgical - evacuation by dilation & curettage, IV oxytocin started after dilation of cervix & continued postop to dec likelihood of haemorrhage f) long-term monitoring - serial quantitative serum B-hCG levels - weekly until levels within reference range for 3-4 weeks, then 6 monthly, levels should consistently drop & never inc. Normal levels reached within 8-12 weeks after evacuation. If serum hCG levels plateau or rise, pt considered to have malignant disease (gestational trophoblastic neoplasia). Evaluate all future pregnancies early with US

Detailed knowledge and understanding of SARC

1) contraceptive vaginal ring or contraceptive patch - <1/100 women get pregnant in 1 yr a) small flexible plastic ring in vagina, or patch on skin: release oestrogen & progesterone - stops ovulation, thickens cervical mucous, thins uterus b) advantages - not affected by V+D, can make periods lighter, less painful & regular, R: easy to insert & remove, P: can help acne c) disadvantages - not suitable if very overweight or smoker >35, low risk of serious SE (blood clots, breast & cervical cancer), temporary SE (inc vaginal discharge, headaches, nausea, breast tenderness), breakthrough bleeding & spotting may occur in first few months, some meds interfere, R: 3 weeks in & 1 week out, P: possible skin irritation, new patch each week for ¾ weeks 2) combined pill - <1/100 women get pregnant in 1 yr a) oestrogen & progesterone - stops ovulation, thickens cervical mucous, thins uterus b) advantages - usually periods more regular, lighter & less painful, dec risk of cancer of ovary + uterus + colon, ok if <50 & non-smokers, fertility returns quick c) disadvantages - not suitable for very overweight or smokers >35, low risk of serious SE (blood clots, breast & cervical cancer), temporary SE (headaches, nausea, mood changes, breast tenderness), missing pills or V+D dec effectiveness, some meds interact, breakthrough bleeding & spotting common in first few months 4) progesterone only pill - <1/100 women get pregnant in 1yr a) contains progesterone - thickens cervical mucous, stops ovulation b) advantages - used if can't use oestrogen, smoke & over 35, breastfeeding c) disadvantages - periods may stop, be irregular, light, or more frequent, temporary SE (acne, breast tenderness, weight changes, headaches), ovarian cysts, V+D dec effectiveness, some meds may interact 5) male condom - 2/100 women get pregnant in 1yr a) thin latex (rubber) or polyurethane (plastic) - put over erect penis to stop sperm reaching vagina b) advantages - free, stop STIs, no serious SE, additional spermicide not needed c) disadvantages - may split or slip off, man needs to withdraw as soon as ejaculation & not spill any semen, put on before penis touches woman's genitals 6) female condoms - 5/100 women get pregnant in 1yr a) soft thin polyurethane which loosely lines vagina & covers area just outside b) advantages - put on any time before sex, stops STIs, no serious SE c) disadvantages - needs to make sure penis enters vagina not between vagina & condom, may get pushed into vagina, not as widely available as male condoms 7) diaphragm/cap with spermicide - 6/100 women get pregnant in 1yr a) flexible latex or silicone device, used with spermicide, put into vagina to cover cervix - stops sperm entering uterus b) advantages - put it in any time before sex, no serious health risks c) disadvantages - extra spermicide needed if have sex again, some people sensitive to spermicide, left in for 6hrs after sex 8) natural family planning - 1/100 women get pregnant in 1yr a) fertile & infertile times of menstrual cycle identified - shows when you can have sex without risking pregnancy. Identified using persona = small handheld device which tests urine to measure hormonal changes b) advantages - no physical SE, woman has inc awareness of her body, can be used to plan a pregnancy c) disadvantages - avoid sex or use a condom at fertile times, takes 3-6 cycles to learn effectively, have to keep daily records

Significance & management of menopause

1) diagnosed after 12 months of amenorrhea 2) s/s - hot flashes or flushes (most common), insomnia, weight gain and bloating, mood changes, irregular menses, mastodynia, depression, headache, vaginal epithelium becomes redder, then as atrophies the surface becomes pale, rugation dec & vaginal wall becomes smooth, ovaries dec in size & no longer palpable during gynecologic examination, uterus dec, fibroids, if present, become less symptomatic, dec pelvic muscle tone - can manifest as prolapse of reproductive or urinary tract organs 3) menopausal markers - inc serum FSH, dec estradiol & inhibin 4) osteoporosis - bone loss accelerates in late menopausal transition and continues for the first few years after menopause. Prevention of fractures with: bisphosphonates (alendronate, etc), selective estrogen receptor modulators (raloxifene), calcium, vit D, calcitonin, monoclonal antibodies 5) HRT - to provide relief of vasomotor symptoms, dec risk of unwanted pregnancy, avoid the irregularity of menstrual cycles, preserve bone, lower the risk of disease, improve quality of life

Detailed knowledge of diagnosis, management and implications of hypertensive disorders of pregnancy

1) diagnosis - a) pregnancy induced HTN: >140/90mmHg after 20 weeks & was normotensive b) associated with proteinuria >300mg/L c) previously hypertensive: inc BP by >30/15mmHg from baseline d) resolves in 6 weeks post-partum (usually in 10 days) e) asymptomatic or: N+V, malaise, headaches, visual disturbances, epigastric pain, irritability & altered conscious state f) also: clonus (4+ beats), hyperreflexia, focal neurological deficits, papilledema g) haematological - hyperuricemia, thrombocytopenia, abnormal LFTs 2) mx - a) mild-moderate: BP 140/90 - 160/110mmHg + proteinuria >300mg/L I) anti-hypertensives - methyldopa II) regullarly check BP - if unstable give anti-hypertensives, consider delivery III) pregnancy may need to be terminated if risk of IUD > risk of prematurity IV) may need delivery preterm - betamethasone to mature fetal lungs b) severe: >160/110mmHg + proteinuria >1g/L + absence of UTI I) H2 blocker as may need LCSC - delivery is only cure II) close monitoring III) strict fluid balance (85ml/hr input) & catheter (risk of fluid overload) IV) stabilise BP - aim for MAP <125mmHg eg/ hydralazine every 15 mins until BP controlled, or IV labetolol or nifedipine sublingually V) magnesium sulphate as eclampsia prophylaxis 3) implications - a) to mother - eclampsia, cerebral vascular damage, renal & liver failure, HELLP, DIC b) fetus - IUGR, IUD, iatrogenic preterm delivery NB/ eclampsia = convulsions (tonic-clonic) not attributable to other cerebral causes + s/s of eclampsia. Both due to defect in blood vessels of placenta NB2/ if pre-eclampsia - need to keep in (not discharge) as high risk

Diagnosis and management of sexual dysfunction

1) diagnosis - a) sexual desire disorders/dec libido - lack or absence for some period of time of sexual desire or libido for sexual activity or of sexual fantasies. Can be general or specific for current partner causes - dec in production of normal oestrogen in women or testosterone in both men and women, ageing, fatigue, pregnancy, medications (eg/ SSRIs) or psychiatric conditions eg/ depression and anxiety 2) sexual arousal disorders - lack of desire, lack of arousal, pain during intercourse, and lack of orgasm. Aversion to, and avoidance of, sexual contact medical causes eg/ dec blood flow or lack of vaginal lubrication, or chronic disease 3) anorgasmia - persistent delays or absence of orgasm following a normal sexual excitement phase in at least 75% sexual encounters can have physical, psychological, or pharmacological origins eg/ SSRIs, menopause 4) sexual pain disorders - affect women almost exclusively - dyspareunia (painful intercourse) or vaginismus (involuntary spasm of muscles of vaginal wall that interferes with intercourse) causes - dyspareunia, insufficient lubrication (vaginal dryness) in women (insufficient excitement & stimulation, or hormonal changes caused by menopause, pregnancy, or breast-feeding), vaginismus - past sexual trauma 5) post-orgasmic diseases - post-coital tristesse (PCT) = feeling of melancholy & anxiety after sexual intercourse lasting up to 2 hours 6) pelvic floor dysfunction 2) mx - a) although no approved pharmaceuticals for addressing female sexual disorders, several are under investigation for their effectiveness b) vacuum device = only approved medical device for arousal and orgasm disorders - designed to inc blood flow to clitoris & external genitalia. c) dyspareunia often prescribed pain relievers or desensitizing agents, lubricants and/or hormone therapy, may have counselor or therapist for psychosocial counseling d) menopause - oestrogens responsible for maintenance of collagen, elastic fibers, and vasoculature of urogenital tract - all needed for maintaining vaginal structure & functional integrity; maintaining vaginal pH & moisture levels (keeps tissues lubricated & protected) prolonged oestrogen deficiency = atrophy, fibrosis, and dec blood flow to urogenital tract women with dec sexual functioning have dec oestradiol levels androgen therapy for hypoactive sexual desire disorder (HSDD) has a small benefit but safety is not known

Diagnosis, management and implications of anaemia

1) diagnosis - FBC at booking visit & around 28 weeks to identify anaemia (<11g/dL in 1st trimester, <10.5g/dL in 2nd and 3rd), some may have s/s eg/ SOB, lethargy 2) management - treat the cause: most are iron deficient so need oral iron supplements, dietary advice 3) implications - a) foetus - associated with low birth weight & preterm delivery, may have anaemia at birth & 1st yr of life b) mother - inc risk through pregnancy due to iron deficiency (fatigue, inc risk of infection), inc risk of PPH & need for blood transfusion postnatally

Diagnosis, impact and management of preterm labour

1) diagnosis - delivery from 24 weeks to 36 + 6 (~10% births), associated with low social conditions & maternal nutrition, APH, multiple pregnancies, uterine abnormalities, cervical incompetence, PROM, previous preterm baby, smoking + substance abuse, infection eg/ GBS 2) impact - if caused by infection fetus can be highly affected, each day after 24 weeks inc chance of survival by 5%, infants <500g unlikely to survive but 1500+g normally fine a) complications - RDS, jaundice, hypoglycemia, hypothermia, pulmonary dysplasia, neurodevelopmental delay 3) mx - a) prevention - avoid heavy lifting & excess exercise b) delay delivery to give corticosteroids (fetal lung surfactant) c) drugs to inhibit uterine activity (if <34 weeks & membranes intact & effaced <5cm dilated) - It's Not My Time: I) B-adrenergic agonists eg/ salbutamol, Terbutaline - inhibit actin-myosin interactions, administered with 5% dextrose II) prostaglandin synthetase inhibitors eg/ Indomethacin - inhibit prostaglandin production & uterine activity, also closes ductus arteriosus so only if necessary III) Magnesium sulphate - tocolytic, neuroprotective effect IV) slow Ca2+ channel blockers eg/ Nifedipine - may cause problems if given in organogenesis V) corticosteroids eg/ betamethasone or dexamethasone VI) oxytocin antagonists

Diagnosis and management of puerperal pyrexia

1) diagnosis - fever, rigors, mastitis (painful, engorged, erythematous breast, purulent nipple discharge), endometritis (abdo pain, offensive vaginal lochia) 2) management - empirical Abx base on likely cause, involve microbiology if failure to respond or suspected severe infection NB/ maternal pyrexia (>38°C) within first 14 days of delivery

Diagnosis and management of pregnancy complicated by epilepsy

1) diagnosis - for most women pregnancy doesn't change seizure frequency, but may inc due to poor compliance or hyperemesis in early pregnancy or changes in drug bioavailability 2) management - a) folic acid 5mg o.d (dec NTD level) b) medication continued in majority of pts due to mortality & morbidity of seizures c) vit K from 36 weeks (risk of hemorrhagic disease of newborn) NB/ anti-epileptics are teratogenic & cause inc risk of NTD, orofacial clefts & heart defects

Diagnosis, management and implications of preterm labour/rupture of membranes

1) diagnosis - labour & delivery before 37 weeks gestation (6-15% deliveries) 2) management - a) delivery at unit with facilities to care for neonate, intra-utero transfer may be needed b) corticosteroids - dec rates of neonatal death, RDS, intraventricular hemorrhage in newborn c) tocolytics - astosiban, nifeipine d) surgical - cervical cerclage (suture) for those at risk of identified short cervix 3) implications - higher risk of mortality: overall survival rates - 20% at 23 weeks, 50% at 25 weeks, >95% at 30 weeks

Diagnosis, impact and management of episiotomy and perineal repair

1) diagnosis - surgical incision of perineum & posterior vaginal wall during 2nd stage of labor to quickly enlarge opening for baby to pass through a) under local anesthetic (pudendal anesthesia) & sutured after delivery b) for a forceps delivery, cut is made after applying forceps, but before traction c) mediolateral episiotomy recommended - start at posterior part of fourchette, move backwards, then turn medially before border of the anal sphincter d) cutting in midline produces an incision in a comparatively avascular area & cuts the aponeurosis of muscles rather than bodies, but can extend backwards into rectum 2) impact - 85% women with vaginal delivery have some perineal trauma & 65% will require suturing 3) mx - check extent of lacterations, repair vaginal mucosa - continuous stitch to close mucosa, then interrupted sutures close to muscle layer, then closure of skin, when finished put a finger in the rectum to check no sutures have passed into the rectal mucosa & sphincter is intact 4) 3rd and 4th degree tears - rare, occult damage to anal sphincter is not uncommon, especially after instrumental delivery

Detailed knowledge of the diagnosis, management and implications of gestational diabetes

1) diagnosis - usually done early if high risk eg/ high BMI (30+), Fx of diabetes, otherwise may be picked up at 24-28 week scan, fasting plasma glucose 5.6+mmol/l, or 2hr plasma glucose of 7.8+mmol/l 2) management - inc insulin needed, need strict glycaemic control (prevent hypos & hypers) - give metformin, induction of labour at 38 weeks 3) implications - a) fetus - inc risk of: congenital abnormalities, perinatal mortality, macrosomia, shoulder dystocia, polyhydramnios b) mother - diabetic nephropathy & retinopathy, inc risk of miscarriage, pre-eclampsia & operative delivery

Diagnosis and management of pregnancy complicated by obesity

1) diagnosis -BMI of 30+ kg/m2 at 1st antenatal consultation. 3 classes - BMI 30.0-34.9 (Class I), BMI 35.0-39.9 (Class 2), BMI 40+ (Class 3 or morbid obesity) 2) management - preconceptual counselling to defer pregnancy until dec weight, hospital based care, dietician input, folic acid until 12 weeks, may have aspirin (dec risk of pre-eclampsia) +/- thromboprophylaxis (dec risk of VTE), OGTT offered in late 2nd trimester, should try for NVD, but C-section may be needed NB/ risks to mum - 1st trimester = miscarriage & congenital abnormalities (esp NTDs); risk of VTE throughout; 2nd/3rd trimester = preeclampsia, gestational diabetes; poor labour progress, instrumental birth/traumatic C-section & anaesthetic complications; PPH NB2/ risks to fetus - macrosomia, stillbirth, shoulder dystocia, NICU & neonatal death, childhood obesity & juvenile diabetes

Detailed understanding of the common surgical procedures in obstetrics and gynaecology including indications, contraindications, principles and complications

1) diagnostic hysteroscopy - uterine cavity inspected with rigid or flexile hysteroscope passed through cervical canal. Cavity distended with CO2 or saline. No anaesthetic or local or general. Adjunct to endometrial biopsy if menstrual problems don't respond to medical tx 2) hysteroscopic surgery - operating hysteroscope used in which small instruments passed down parallel channel. Endometrium or intra-cavity fibroids or polyps removed using cutting diathermy or glycine irrigation fluid. If uterus septum present can be resected up to fundus of cavity a) complications - uterine perforation, fluid overload. Sterility not ensured so sometimes laparoscopic tubal sterilisation performed at same time 3) diagnostic laparoscopy - peritoneal cavity inflated with CO2 after passing small hollow Veress needle through abdo wall - enables sharp trocar to be inserted through umbilicus. Laparoscope then inserted down trocar for visualisation of pelvis. Used to assess macroscopic pelvic disease in mx of pelvic pain & dysmenorrhoea, infertility, suspected ectopic pregnancy, and pelvic masses 4) laparoscopic surgery - instruments to grasp or cut tissue inserted at separate ports in abdo wall. Commonly used for sterilisation, to remove adhesions or areas of endometriosis, or remove ectopic pregnancy. Many operations now laparoscopic. Gives better visualisation of tissues, less tissue handling, less infection, dec hospital stay, faster recovery with less pain a) complications - serious visceral damage can occur 5) hysterectomy - most common major gynae surgery. Fallopian tubes normally removed (bilateral salpingectomy) to dec risk of ovarian cancer, sometimes ovaries if hx of cysts or older (bilateral oophorectomy). Most commonly done for menstrual disorders, fibroids, endometriosis, chronic PID, prolapse, malignancies a) types - I) total abdominal hysterectomy - removal of uterus + cervix though abdo incision - ligation of anastomosis between uterine & ovarian arteries + round ligament, bladder dissected off cervix + upper vagina, then ligation of main uterine artery + cardinal ligament, then cervicovaginal branches of uterine artery + uterosacral ligaments ligated. Indications - malignancy (ovarian + endometrial) large or immobile uterus, abdo inspection needed. In subtotal hysterectomy cervix retained (uterosacral ligament left) - dec risk of damaging uterus + bladder II) vaginal hysterectomy - removal or cervix + uterus after incising vagina from below (opposite order to TAH), indications - uterine prolapse. Lower morbidity & quicker recovery than TAH III) laparoscopic hysterectomy - alternative to THA, subtotal can be performed this way IV) Wertheim's (radical) hysterectomy - removal; of parametrium, upper 1/3d of vagina & pelvis + LN. indication - stage 1a-2a cervical carcinoma/ occasionally done vaginally 6) dilation & curettage (D&C) - cervix dilated with steel rods (Hagar dilator) of increasing size, endometrium then curetted to biopsy it - diagnostic procedure, inferior to hysterectomy as cavity not inspected 7) evacuation of retained products of conception (ERPC) = cervix dilated & retained non-viable foetus or placental tissue removed using suction curette 8) surgery for CIN - a) large loop incision of transformation zone (LLETZ) - cutting diathermy under LA to remove transformation zone of cervix. Slight inc risk of preterm delivery b) cone biopsy - removal of transformation zone & much of cervix by making circular cute with scalpel or loop diathermy n cervix. Used to stage early carcinoma, sufficient for treatment of stage 1a, under general or epidural or spinal anaesthetic. Significant inc risk of preterm delivery 9) operation for prolapse - a) 'repair' - anterior repair (cystocele) involves excision of prolapsed vaginal wall & application of bladder base & fascia, then vagina closed. Posterior repair (rectocele) is similar - levator ani muscle wither side being plicated between rectum & vagina. Operations often performed together, or with vaginal hysterectomy for uterine prolapse. Complications - retention of urine, overtightening of vagina (important to know if pt sexually active) b) hysteropexy - resuspension of prolapsed uterus using strip of non-absorbable bifurcated mesh to lift uterus & hold it in place. One end of mesh attached to cervix & other to anterior longitudinal ligament over sacrum - can be done open or laparoscopic. Advantages over hysterectomy - preserves fertility, stronger repair, less risk of recurrent prolapse, cuts to vagina avoided (less risks of subsequent sexual problems) c) sacrocolpopexy - used for prolapse of vaginal vault after hysterectomy, mesh attached from vaginal vault to sacrum, can be performed open or laparoscopic d) sacrospinous fixation - using blind vaginal approach, used for vault prolapse. Less effective than sacrocolpopexy 10) operation for urinary stress incontinence - need to elevate bladder neck so can be compressed when abdo pressure rises a) tension free vaginal tape (TVT) - tape made of polypropylene mesh, approx. 1cm wide, fixed to trocar at each end. 2cm vertical incision made on anterior vaginal wall over mid-urethral section. After lateral dissection around urethra, tape is introduced vaginally with trocars entering retropubic space. Trocars brought out through small transverse suprapubic incisions with tape in position without tension & vaginal skin closed over. Cystoscopy performed to ensure bladder hasn't been perforated. If tape overtight post-op urinary retention can occur. Tension on tape can be adjusted within first 2 weeks after insertion b) transobturator tape (TOT) - variation of TVT, tape passed into obturator canal c) burch colposuspension - dissection through abdo incision in extraperitoneal space over bladder & anterior vaginal wall. Vaginal wall either side bladder neck lifted up to iliopectineal ligament either side of symphysis pubis with non-absorbable sutures. Operation is usually performed now for failed tap[e procedures 11) operations for fibroids - a) myomectomy performed through cervix (TCRF) or abdominally (laparoscopically or open). Complications - risk of adhesions, uterine rupture during labour (greater with laparoscope), peri-operative haemorrhage, occasionally requiring hysterectomy. Remote risk of spread of unsuspected leiomyosarcoma during laparoscopic morcelllation b) uterine artery embolisation - alternative to hysterectomy for women with fibroids, effect on fertility is variable & pregnancy complications appear more common

Detailed understanding of the role of microbiological investigations in the investigation and management of urogynaecological problems

1) dipstick analysis of urine - may treat as bacterial if +ve for nitrite and/or leukocytes. It is advised that dipstick testing is not used to diagnose UTIs in adults with indwelling urinary catheters nitrite +ve = UTI, leucocyte +ve = infection but not specific 2) urine culture - to exclude diagnosis, or if high-risk (eg/ pregnant, immunosuppressed, renal tract anomaly), or if failed to respond to earlier empirical treatment. Urine culture should always be performed in men with a history suggestive of UTI regardless of results of dipstick test. Urine culture not required for symptomatic lower UTI in non-pregnant women 4) mx - a) pharmacological - trimethoprim or nitrofurantoin = 1st choice for empirical treatment of uncomplicated UTI. Treatment should be no longer than 3 days in women with uncomplicated UTI, although should remain at 7 days for men b) 1st line in mild cases of uncomplicated pyelonephritis is oral ciprofloxacin for 7-10 days. Co-amoxiclav and cefalexin are alternatives c) paracetamol and/or NSAIDS for symptomatic relief d) asymptomatic bacteriuria should not be treated in adults with catheters or in non‑pregnant women

Risk factors, symptoms, investigation and management of endometrial hyperplasia and adenocarcinoma of the endometrium

1) endometrial carcinoma = most common gynaecological cancer - women have cumulative risk of developing it of 1% by age 75 2) caused by 2 subtypes - a) type 1 - majority, low-grade endometrial cancers which are oestrogen sensitive, associated with obesity & usually less aggressive. Often have atypia as a precursor b) type 2 - high-grade, clear cell, serous or carcinosarcoma cancer, more aggressive & not oestrogen sensitive, not related to obesity 3) risk factors - exposure to endogenous & exogenous oestrogens associated with obesity, PCOS, diabetes, early age of menarche, nulliparity, last-onset menopause, being >55, unopposed oestrogen HRT, use of tamoxifen (oestrogen antagonist in breast but agonist in uterus), Lynch type II syndrome a) COCP & pregnancy are protective 4) premalignant disease - endometrial hyperplasia with atypia - oestrogen unopposed or erratic can cause endometrial hyperplasia. Further stimulation predisposes to abnormalities of cellular & glandular architecture or 'atypical hyperplasia' - can cause menstrual abnormalities or postmenopausal bleeding, and is premalignant. Hyperplasia with atypia often coexists with carcinoma elsewhere in uterine cavity. Endometrial hyperplasia with atypia common in women of reproductive age a) if this diagnosis made, hysterectomy should be considered, if fertility concerns progestogens (IUS or continuous oral) + 3-6 monthly hysteroscopy & endometrial biopsy used, along with referral to fertility specialist 5) s/s - postmenopausal bleeding most common (as age inc, risk of cause being malignant inc). if premenopsaul - irregular or IMB a) exam - pelvis appears normal, atrophic vaginitis may coexist 6) staging - a) stage 1 - 1a = only endometrium, 1b = <1/2 myometrium, 1c = >1/2 myometrium b) stage 2 - 2a = cervical glands, 2b = cervical stroma c) stage 3 - outside uterus 7) ix - US + endometrial biopsy with Pipelle or hysteroscopy - biopsy needed for diagnosis a) staging after hysterectomy b) MRI for estimate of myometrial invasion c) CXR to exclude pulmonary spread d) to assess fitness for surgery - FBC, U&Es, glucose, ECG 8) tx - a) surgery - 75% present with stage 1 = total laparoscopic hysterectomy & BSO. Pelvic & para-aortic lymphadenectomy may be performed if high risk b) adjuvant therapy - I) external beam radiotherapy - used follow hysterectomy if high risk for LN involvement. Also used for pelvic recurrence II) vaginal vault radiotherapy - used when high risk, usage dec local recurrence but doesn't prolong survival III) chemotherapy - may be helpful, +/- radiotherapy IV) progestogens - seldom used

Epidemiology, aetiology, diagnosis, management and prognosis of vulval cancer

1) epidemiology - 1800 cases/yr - disease of >60s 2) aetiology - vulval intraepithelial neoplasia (premalignant vulval disease) or lichen sclerosus (autoimmune inflammatory skin condition) 3) diagnosis - a) symptoms - vulval pain, bleeding, feeling a lump b) signs - vulval mass or ulceration, inguinal LN enlargement if advanced c) ix - punch biopsy of lesion, USS or CT of groin d) staging & prognosis: I) stage 1 - confined to vulva, -ve groin nodes - 90% 5yr SR II) stage 3 - nodes +ve - 50% 5yr SR 4) mx - depends on stage a) surgery - radical excision of tumour (1cm clear margin around tumour) I) tumour <4cm - sentinel LN identification & removal II) >4cm - groin node dissection b) radiotherapy - adjuvant if +ve nodes, or primary tx if tumour is inoperable or near anal sphincter (close surgical margin) c) chemotherapy - may be given with radiotherapy 5) prognosis - good if early stage, esp if -ve nodes. Poor if locally advanced disease eg/ fistula

Epidemiology, aetiology, diagnosis, management and prognosis of ovarian cancer

1) epidemiology - 2nd commonest UK gynae cancer (6000 cases/yr) 2) aetiology - disease of menopause, type 1 (low grade), type 2 (high grade) - genetic mutations eg/ BRAC 1 3) diagnosis - a) symptoms (BEAT in women >55) - persistent bloating, abdo pain, eating trouble (feeling full after a meal), trouble with bladder (urinary frequency) b) signs - abdominopelvic mass, ascites, poor nutrition if present late c) ix - TV USS to assess ovaries, Ca-125 tumour marker, CT CAP (metastases) d) staging & prognosis - pts usually present with stage 3 disease (spread from ovaries to omentum & LN), 40% 5yr SR 4) mx - majority have surgery and chemotherapy a) surgery - hysterectomy & bilateral salpingoophoectomy +/- pelvic LN removal b) radiotherapy - not usually given due to non-localised disease c) chemotherapy - pre & post op adjuvant chemotherapy. 6 cycles 5) prognosis - good if early stage, type. Most are late stage so often recur and die from recurrence within 5 years. If in remission good QOL

Detailed understanding of the epidemiology, aetiology, presentation and management of miscarriage

1) epidemiology - accounts for 42,000 hospital admissions in UK annually a) occurs in 20% recognised pregnancies - true rate probably higher b) 85% spontaneous miscarriages occur in 1st trimester c) risk falls rapidly with advancing gestation - 9% at 6 complete weeks of gestation, 4% at 7 weeks, 2% at 8 weeks, 0.5% at 9 weeks 2) aetiology - often no cause is found but common recognised causes include: abnormal fetal development, genetically balanced parental translocation, uterine abnormality, incompetent cervix (2nd trimester), placental failure, multiple pregnancy, PCOS, antiphospholipid syndrome, inherited thrombophilias, infections, poorly controlled diabetes or thyroid disease 3) presentation - see criteria for threatened, inevitable, incomplete/complete (essentially heavy bleeding, may have pain, os may be closed or open) 4) management - a) threatened - reassure pregnancy should continue as normal once bleeding stops, avoid strenuous activity but continue ADL b) inevitable - expectant so await spontaneous miscarriage (2 weeks), surgical evacuation under GA if products of conception protruding though cervical os, medical management eg/ vaginal misoprostol - check up in 2 weeks, counselling & support c) incomplete - may need to remove all products of conception eg/ oxytocin infusion (maintains uterine contractions if vaginal bleeding is heavy), US (shows retained products), surgical evacuation NB/ medical mx - pain, may not always work (may need something else), heavy bleeding - clots

Detailed understanding of the epidemiology, aetiology and characteristics of urinary infections

1) epidemiology - asymptomatic bacteriuria occurs in 2-5% pregnant women 2) aetiology - E coli is most common cause (85% cases) - from fecal flora colonizing periurethral area, causing ascending infection. Other pathogens: Klebsiella pneumoniae, Proteus mirabilis, Enterobacter, Staph saprophyticus, GBS 3) characteristics - a) hx - asymptomatic or: dysuria, frequency, urgency, suprapubic pain, and hematuria in the absence of systemic symptoms b) pyelonephritis symptoms - fever (>38°C), shaking chills, costovertebral angle tenderness, anorexia, N+V, flank pain c) physical exam - suprapubic or costovertebral tenderness, may have no physical findings, assessment of fetal heart rate on the basis of gestational age should be included as part of the evaluation. Often, owing to maternal fever, the fetal heart rate is elevated to >160 bpm

Risks and management of multiple pregnancy

1) risks - a) maternal - miscarriage, hyperemesis, anaemia, pre-eclampsia (& eclampsia), gestational diabetes, operative delivery, post-natal depression, APH, PPH b) foetus - preterm labour & C-section, IUGR, IUD, twin-twin transfusion syndrome, congenital abnormalities, malpresentation 2) management - hospital led care, serial USS for growth & presentation, mode of delivery depends on presentation of twin 1, maternal wishes & chorionicity (many 1st are NVD & 2nd are forceps or CS)

Risks and management of pregnancy complicated by thrombophilia

1) risks - pregnancy = prothrombotic state so inc risk of VTE, also risk of fetal loss, preeclampsia, placental abruption, IUGR, recurrent miscarriages 2) management - seen in hematology-obstetric clinic, antenatal/postnatal LMWH, avoid dehydration, graduated compression stockings, aspirin in antiphospholipid syndrome

Epidemiology, aetiology, diagnosis, management and prognosis of endometrial cancer

1) epidemiology - commonest UK gynae cancer (8000 cases/yr), disease >60s 2) aetiology - disease of menopause, type 1 (low grade) - related to obesity and PCOS, type 2 (high grade) - genetic mutations? 3) diagnosis - a) symptoms - postmenopausal bleeding +/- discharge (from tumour) NB/ 10% pts with PMB have a cancer. Others - lack of vaginal oestrogen, polyps b) signs - often nothing to find clinically as disease is often early stage c) ix - TV USS & endometrial biopsy (Pipelle) depending on endometrial thickness seen on USS (> 4mm), MRI to assess myometrial invasion & nodal status d) staging & prognosis - pts usually present with stage one disease (confined to uterus), overall prognosis is 70% 5yr SR (most present with early stage) 4) mx - majority have surgery a) surgery - hysterectomy & bilateral salpingoophorectomy +/- pelvic LN removal b) radiotherapy - post op adjuvant radiotherapy if nodes +ve, high-grade tumour or disease spread to cervix and parametrium c) chemotherapy - not given unless advanced 5) prognosis - good if early stage, type one cancers. If in remission good QOL

Epidemiology, aetiology, diagnosis, management and prognosis of cervical cancer

1) epidemiology - commonest cancer in young women 2) aetiology - oncogenic HPV + smoking causes premalignant disease, which can lead to cancer over years 3) diagnosis - a) symptoms - IMB, post coital bleeding, smelly vaginal discharge from tumour, late stage - malignant pain and fistula b) signs - mass felt or seen on cervix, screen detected cervical cancer c) ix - colposcopy & biopsy of lesion, MRI to assess myometrial invasion & nodal status, screen detected cervical cancers usually have a loop diathermy d) staging & prognosis - screening prevents 70% cases (detects CIN = pre-invasive disease) I) overall prognosis is 60% 5yr SR, stage 1 - 90% 5yr SR 4) mx - depends on stage (microscopic to advanced) a) surgery - microscopic: loop diathermy, macroscopic (>4cm): radical hysterectomy +/- pelvic LN removal b) radiotherapy - macroscopic (>4cm) c) chemotherapy - given with radiotherapy to inc survival rates 5) prognosis - good if early stage esp if -ve nodes. Poor if locally advanced disease eg/ fistula

Detailed understanding of the epidemiology, aetiology, presentation and management of ectopic pregnancy

1) epidemiology - tubal pregnancies in 1/100 pregnancies in UK, causes 1/300,00 deaths each year 2) aetiology - inc risk with: previous ectopic, IUCD in situ, failed sterilization, previous tubal surgery, previous history of PID 3) presentation - a) acute - amenorrhea (6-8 weeks), lower abdo pain, uterine bleeding, referred shoulder tip pain & syncopal episodes may occur, shock with hypotension, tachycardia, signs of peritonism (abdo distension, guarding, rebound tenderness) b) subacute - short amenorrhea, recurrent vaginal bleeding & abdo pain c) subacute may be mistaken for miscarriage, acute salpingitis, appendicitis etc, Hb may be low & WCC inc, hCG confirms pregnancy - combine with US to confirm if intra-uterine (ectopic = free fluid in cavity) 4) management - blood urgent cross match & transfusion a) medical - administration of methotrexate systemically or by injection into ectopic pregnancy by laproscopic visualization or ultrasound guidance, most effective when ectopic is <2cm in size & hCG <1500iu/L b) surgical - salpingectomy if tube badly damaged or contralateral tube healthy, salpingotomy - when ectopic contained within tube can conserve tube by removing pregnancy, especially when contralateral tube lost

Risk factors, symptoms, investigation and management of epithelial tumours and germ cell tumours of the ovary

1) epithelial tumours - 95% ovarian cancers, most common in postemenopausal women, derived from epithelium over ovary (or fallopian tube if high grade) 2) germ cell tumours - originate from undifferentiated primordial germ cells of gonad, 3% ovarian malignancies a) teratoma or dermoid cyst - common, benign, young women. Malignant form = solid teratoma b) yolk sac tumor - highly malignant, young women or children c) dysgerminoma - rare, malignant, younger women, sensitive to radiotherapy 3) ovarian cancers normally asymptomatic so present late, 10 yr survival is 40-50%. 7000 new cases/yr in UK, 4200 death/yr. Lifetime risk of developing is 1 in 60 4) risk factors - relate to number of ovulations: early menarche, late menopause, nulliparity. Fmx via BRCA1, 2 or HNPCC gene mutations (if 2 relatives affected, lifetime risk = 13%, unless BRCA1 then 50%) a) pregnancy, lactation & COCP protective 5) s/s - symptoms initially vague and/or absent - 70% pts present with stage 3-4 disease a) may have - abdo distension or mass palpated by pt, early satiety, pelvic +/- abdo pain, urinary frequency +/- urgency, vaginal bleeding -many symptoms similar to IBS b) exam - cachexia, abdo or pelvic mass, ascites, breasts should be palpated for metastases 6) staging - a) stage 1 - confined to ovaries (1a = 1 ovary, 1b = 2 ovaries, 1c = tumour on surface of capsule) b) stage 2 - disease extending into pelvic eg/ uterus, tubes c) stage 3 - abdo disease +/- affected LN (eg/ omentum, SI, peritoneum) d) stage 4 - disease beyond abdomen eg/ lungs, liver parenchyma 7) ix - a) initial detection - CA 125 measured in women >50 with abdo symptoms. If CA 125 >351IU/mL US abdo & pelvis. If US or exam identifies ascites or pelvic/abdo mass, urgent referral to secondary care b) secondary care - I) if <40: aFP + hCG measured to check for germ cell tumours II) RMI calculated: U x M x CA125 III) U: 1 each for - multilocular cysts, solid areas, metastases, ascites, bilateral lesions (1 if 1, 3 if >2 present) IV) M: 1 = premenopausal, 3 = post V) if RMI >250, referred to MDT, CT of pelvis & abdomen to establish extent of disease 8) tx - a) surgery - mid-line laparotomy for total hysterectomy + BSO + partial omentectomy, with biopsies of peritoneal deposits, random biopsies of peritoneum + retroperitoneal LN (LN biopsied if stage 1, removed if stage 2+) I) may have ultaradical surgery eg/ removal of bowel, spleen etc I) if wish to preserve fertility & early or borderline disease - uterus & unaffected ovary preserved, with follow up b) chemo - after confirmed tissue diagnosis (from surgery or image guided biopsy) I) very early stage - no chemo II) 1c - 6 cycles of platinum agent carboplatin III) 2-4 - carboplatin (or cisplatin) alone or in combination with paclitaxel IV) many women have chemo, then half-way through have surgery V) early stage malignant germ cell tumours tx - removal of adnexa, otherwise chemo

Describe in detail the regional anatomy and histology relevant to obstetrics and gynaecology

1) female reproductive organs subdivided into internal & external genitalia - internal = organs within true pelvis eg/ vagina, uterus, cervix, uterine tubes (fallopian tubes), and ovaries. External genitalia lie outside true pelvis eg/ perineum, mons pubis, clitoris, urethral meatus, labia majora and minora, vestibule, greater vestibular (Bartholin) glands, Skene glands, and periurethral area 2) external genitalia - vulva (pudendum) describes external organs visible in perineal area: mons pubis, labia minora & majora, hymen, clitoris, vestibule, urethra, Skene glands, greater vestibular (Bartholin) glands, and vestibular bulbs. Boundaries - mons pubis anteriorly, rectum posteriorly, genitocrural folds (thigh folds) laterally a) mons pubis - rounded portion of the vulva where sexual hair development occurs at puberty. May be described as directly anterosuperior to the pubic symphysis b) labia - labia majora are 2 large, longitudinal folds of adipose & fibrous tissue, vary in size & distribution from female to female (size dependent upon adipose content). Extend from mons anteriorly to perineal body posteriorly. The labia majora have hair follicles. Labia minora (nymphaea) - 2 small cutaneous folds between labia majora & introitus or vaginal vestibule. Anteriorly, the labia minora join to form the frenulum of the clitoris c) hymen - thin membrane at entrance to vaginal orifice. Often is perforated before onset of menstruation, allowing flow of menses. The hymen varies greatly in shape d) clitoris - erectile structure beneath the anterior joining of the labia minora. Width in an adult female ~1 cm & length of 1.5-2.0 cm. Made up of 2 crura, which attach to the periosteum of the ischiopubic rami. Very sensitive structure, analogous to the male penis. Innervated by the dorsal nerve of the clitoris, a terminal branch of the pudendal nerve e) vestibule and urethra - between the clitoris & vaginal introitus (opening) is a triangular area = vestibule, which extends to the posterior fourchette. Vestibule is where urethral meatus is found (~1 cm anterior to vaginal orifice) & gives rise to opening of Skene glands bilaterally. Urethra composed of membranous CT & links urinary bladder to vestibule externally f) Skene and Bartholin glands - Skene glands secrete lubrication at the opening of the urethra. The greater vestibular (Bartholin) glands secrete lubrication to the vagina just outside the hymen, bilaterally, at the posterior aspect of the vagina. Each gland is small, similar in shape to a kidney bean g) vestibular bulbs - 2 masses of erectile tissue deep to bulbocavernosus muscles bilaterally 3) internal genitalia - a) vagina - extends from vulva externally to uterine cervix internally. Located within pelvis, anterior to rectum & posterior to bladder. Vagina lies at a 90º angle in relation to the uterus & held in place by endopelvic fascia and ligaments. Vagina lined by rugae, situated in folds throughout - allow easy distention, especially during child bearing I) pelvic diaphragm, sphincter urethrae & transverse peroneus muscles, and perineal membrane support the vagina. Sphincter urethrae & transverse peroneus innervated by perineal branches of pudendal nerve. Pelvic diaphragm primarily refers to levator ani & coccygeus, innervated by branches of sacral nerves S2-S4 II) vascular supply to the vagina primarily from vaginal artery (branch of anterior division of internal iliac artery) III) nerve supply to vagina primarily from ANS - sensory fibers to lower vagina from pudendal nerve, and pain fibers from sacral nerve roots IV) lymph drainage of vagina generally to external iliac nodes (upper third of vagina), common & internal iliac nodes (middle third), & superficial inguinal nodes (lower third) b) uterus - inverted pear-shaped female reproductive organ in the midline of the body, within the pelvis between the bladder & rectum. Thick-walled and muscular, with a lining that, during reproductive years, changes in response to hormone stimulation throughout a woman's monthly cycle. Uterus divided into 2 parts: most inferior aspect = cervix, bulk = body of uterus (corpus uteri). Between these 2 is the isthmus, a short area of constriction I) body - globe-shaped, typically in anteverted position, at 90º to vagina. Upper aspect is dome-shaped (fundus) - most muscular part of uterus. Body of uterus is responsible for holding a pregnancy, and strong uterine wall contractions help to expel the foetus during labour and delivery. Nonpregnant, nulliparous uterus ~40-50 g, multiparous uterus may weigh slightly more than this, menopausal uterus is small and atrophied and typically weighs much less II) cavity of uterus is flattened & triangular - uterine tubes enter uterine cavity bilaterally in superolateral portion of the cavity III) uterus connected to surrounding structures by ligaments & CT. Pelvic peritoneum attached to body & cervix as the broad ligament, reflecting onto the bladder. The broad ligament attaches the uterus to the lateral pelvic side walls. Within the broad base of the broad ligament, CT strands associated with uterine & vaginal vessels help to support uterus & vagina - these strands = cardinal ligament IV) rectouterine ligaments - within peritoneal folds, stretch posteriorly from cervix to sacrum. Round ligaments of uterus much denser & connect uterus to anterolateral abdo wall at deep inguinal ring. Lie within anterior lamina of broad ligament. Within the round ligament = artery of Sampson (small artery ligated during hysterectomy) V) vasculature of uterus derived from uterine arteries & veins. Uterine vessels arise from anterior division of internal iliac, & branches of uterine artery anastomose with ovarian artery along the uterine tube VI) lymph drainage primarily to lateral aortic, pelvic, & iliac nodes that surround iliac vessels VII) nerve supply - SNS (by way of hypogastric & ovarian plexuses) + PNS (by way of pelvic splanchnic nerves from second through fourth sacral nerves) c) cervix - inferior portion of uterus, separating body of the uterus from the vagina. Cylindrical in shape, with endocervical canal in midline, allowing passage of semen into uterus. External opening into vagina = external os, internal opening into endometrial cavity = internal os. Internal os is the portion of a female cervix that dilates to allow delivery of the foetus, average length of cervix is 3-5 cm I) vasculature - descending branches of uterine artery (run bilaterally at 3 o'clock and 9 o'clock position of cervix) II) nerve supply to cervix is via PNS by way of 2nd-4th sacral segments. Many pain nerve fibers run alongside these parasympathetics III) lymph drainage - obturator, common iliac, internal iliac, external iliac, and visceral parametrial nodes are the main drainage points d) uterine tubes - (fallopian tubes) are uterine appendages located bilaterally at the superior portion of the cavity. They transport sperm toward the egg & allow passage of fertilized egg back to uterus for implantation I) tubes exit the uterus through cornua & form a connection between the endometrial + peritoneal cavities. Each tube ~ 10 cm in length and 1 cm in diameter & situated within part of broad ligament called mesosalpinx. Distal portion of tube ends in an orientation encircling the ovary II) tube has 3 parts - 1st segment, closest to uterus = isthmus. 2nd segment = ampulla - becomes more dilated in diameter & typical place of fertilization. 3rd segment, furthest from the uterus = infundibulum, which gives rise to fimbriae, fingerlike projections responsible for catching the egg released by the ovary III) arterial supply - branches of uterine & ovarian arteries, small vessels in mesosalpinx IV) nerve supply - PNS + SNS. Sensory fibers from thoracic segments 11-12 & lumbar segment 1 V) lymph drainage - through the iliac and aortic nodes e) ovaries - paired organs either side of uterus within mesovarium portion of broad ligament below uterine tubes. Responsible for housing & releasing the ova for reproduction. At birth, a female has ~1-2 million eggs, but only 300 of these ever mature & released I) small & oval-shaped, with grayish color & uneven surface, ~3-5 cm in length during childbearing years (become smaller & atrophic once menopause occurs). Contain many cystic structures that vary in size - represent ovarian follicles at different stages of development and degeneration II) several ligaments support the ovary - ovarian ligament connects uterus & ovary. Posterior portion of broad ligament forms mesovarium, which supports the ovary & houses vascular supply. Suspensory ligament of ovary (infundibular pelvic ligament), a peritoneal fold overlying ovarian vessels, attaches ovary to pelvic side wall III) vasculature - via ovarian artery (R+L ovarian arteries originate directly from descending aorta at L2 vertebra). Ovarian a&v enter & exit ovary at hilum. L ovarian vein drains into L renal vein, R ovarian vein empties directly into IVC IV) nerve supply - run with vasculature within suspensory ligament of ovary, entering ovary at hilum. Supply is through the ovarian, hypogastric, and aortic plexuses V) lymph drainage - primarily to lateral aortic nodes; iliac nodes may also be involved 4) histology - a) vulva: predominantly keratinized, stratified squamous epithelium b) labia majora: both sebaceous and sweat glands; the labia minora are made up of dense connective tissue with erectile tissue and elastic fibers c) hymen: fibrous tissue with few small blood vessels, covered by stratified squamous epithelium d) body of clitoris: composed of 2 channels of vessels & nerve endings that function as erectile tissue, the corpora cavernosa e) mucosa of proximal 2/3rds of urethra composed of stratified transitional epithelium similar to that of bladder. Distal 1/3rd composed of stratified squamous epithelium f) greater vestibular glands: cuboidal epithelium, with ducts lined by transitional epithelium g) vagina has 3 layers: 1st layer = mucosa, epithelium of which is composed of stratified squamous cells that contain a small amount of keratin. Lamina propria composed of loose connective tissue with vast elastic fibers, giving vagina capability to distend. 2nd layer is muscular, mainly SM. 3rd layer is adventitia, also rich in elastic fibers. Large plexus of blood vessels also present h) uterine corpus has 3 layers: inner-outer - endometrium, myometrium, and serosa. Endometrium composed of cells resembling embryonic CT, with some cytoplasm and large nuclei - subdivided into inner stratum basale & outer stratum functionale. Stratum functionale responds to hormonal stimulation. Myometrium composed of 3 layers of SM. Serosa is a continuation of the visceral peritoneum i) most of cervix = collagenous CT, SM, and mucopolysaccharide ground substance. Endocervical canal is rich in mucous glands & primarily columnar epithelium. External portion of cervix (within vagina) composed of stratified squamous epithelium j) area surrounding external os = transformation zone, which is transition point between squamous cells externally and columnar cells of endocervical canal. Zone is the area where cervical cell changes (dysplasia) can occur k) internally uterine tubes have many folds (plicae) - most evident in ampulla. Within mucosa of uterine tubes, 3 different cell types exist: columnar ciliated epithelial cells (25%), secretory cells (60%), and narrow peg cells (< 10%). Layer of SM surrounds mucosa l) ovary divided into 2 main sections: outer cortex & inner medulla. Germinal layer coats entire ovary, made of cuboidal epithelial cells. Cortex is where follicles are found at various stages of development and degeneration - cortex made of tightly packed CT. Medulla is where ovarian vasculature found & composed primarily of loose stromal tissue

Detailed knowledge and understanding of irreversible contraception

1) female sterilization (tubal occlusion) - failure rate is 1/200, permanent a) fallopian tubes cut, sealed or blocked, stops egg & sperm meeting b) advantages - can't be easily reversed, don't have to think about contraception, periods unaffected c) disadvantages - all operation carry some risk (low), small inc risk of ectopic pregnancy if fails, need GA or LA, may be uncomfortable, can't be reversed 2) male sterilization (vasectomy) - 1/2000 fail, permanent a) vas deferens that carry sperm from testicles to penis cut, sealed or tied b) advantages - can't be reversed, don't have to think about contraception, local anaesthetic c) disadvantages - contraception used until semen test shows no sperm left (can take at least 8 weeks), some may have testicular pain, counselling

Assessment of fetal wellbeing (e.g. meconium, fetal heart rate monitoring, fetal scalp blood sampling)

1) fetal HR - a) accelerations - transient inc in HR >15 bpm for over 15 seconds (fetal mvmt) - reassuring if present b) decelerations - transient dec in HR >15 bmp for over 15 seconds, if happens with uterine contraction = due to head compression (not worrying), if happens after contraction & doesn't return to baseline over 20 secs after contraction completed = placental insufficiency c) bpm - should be 110-160 (<100 or >180 abnormal) d) variable decelerations - vary in timing & amplitude, slight rise in baseline rate followed by quick fall then quick recovery slightly beyond baseline, due to cord compression, 5-25bpm normal, <5 for 90+ mins or >25 = abnormal 2) meconium - first stool passed, if amniotic fluid is stained with meconium it will be homogeneously distributed throughout the fluid making it brown - means fetus passed meconium some time ago a) terminal meconium occurs when fetus passes meconium a short enough time before birth/CS that amniotic fluid remains clear, but clumps of meconium in fluid b) meconium ileus - meconium becomes thickened and congested in intestines, early sign of CF. No meconium is passed, abdo distension & N+V soon after birth c) risk of meconium aspiration with prolonged delivery 3) fetal scalp blood sampling - abnormalities in FHR may indicate acidosis. Fetal blood through amnioscope, normal pH = 7.25-7.35, if <7.2 delivery recommended unless spontaneous delivery imminent, NB/ mother's wellbeing moitored by HR, O2 sats, BP and temperature NB2/ normal - no intervention + intermittent auscultation, suspecious = 1 non-reassuring feature - conservative measures, pathological = 2+ - conservative measures + inform obs reg. In all review every 4hrs

Principles of palliative care with gynaecological oncology

1) general palliative care - focus on QOL 2) specialist care - team who specialise in gynae oncology 3) symptom management - using drugs to combat, analgesia (morphine is opiod of choice), non-phamaceutical approaches

Hormonal changes in early pregnancy

1) hCG - produced from placenta, tells body woman is pregnant, causes ovaries to stop maturing an egg every month, levels inc 8 days after ovulation, peak at 60-90 days then dec slightly, leveling off for the remainder of the pregnancy. During first 10 weeks of pregnancy levels double every 2 days 2) progesterone - made early in pregnancy by corpus luteum. CL continues to produce progesterone until about 10 weeks, when placenta takes over. In 1st trimester, levels of progesterone rise exponentially, and then plateau. Progesterone keeps uterus muscle relaxed and plays a role in the immune system helping the body tolerate foreign DNA (fetus) 3) oestrogen - secreted by CL until placenta takes over. Key role in development of fetus, with several organs and other bodily systems in the fetus triggered into development by oestrogen. Levels highest at end of 1st trimester then plateau. Helps stimulate hormone production in fetus's adrenal gland, stimulates growth of adrenal gland, enhances mother's uterus, enabling it to respond to oxytocin

Risks of substance abuse in pregnancy

1) heroin - inc incidence of IUGR, perinatal death & preterm labour, 50% neonates have heroin withdrawal a) need replacement with methadone or buprenorphine 2) amphetamines - inc risk of miscarriage, preterm birth, growth restriction, placental abruption, fetal death in utero, developmental abnormalities 3) cocaine - cardiac arrhythmias & CNS damage in mothers, placental abruption, fetal growth restriction, preterm labour 4) marijuana - may have teratogenic effects

Caesarean section; indications, procedures and complications

1) indications - a) elective - known cephalopelvic disproportion, placenta praevia, malpresentation eg/ brow presentation, history of suburethral repair, vesicovaginal fistula repair, maternal infections eg/ herpes & HIV I) if repeated section need to localize placenta on US to exclude placenta praevia (more common when scar & more likely to be complicated by placenta accreta) II) usually after 39 weeks, but may be earlier in special cases b) emergency - antenatal complications eg/ severe pre-eclampsia, abruptio placentae (baby still alive), or during labour eg/ fetal distress in 1st stage of labour, failure to progress in 1st stage, prolapsed cord (fetus alive), obstructed labour, disproportion becoming evident during labour & after failed induction I) fetal indications - severe IUGR, preterm fetus that needs delivery presents by breech. Postmortem caesarian if fetus alive II) maternal indications - previous surgery eg/ hysterectomy or myomectomy 2) procedures - a) lower uterine segment incision/Pfannenstiel - most common, bladder is reflected from lower segment & transverse incision made. Presenting part delivered through lower segment. Lower segment muscle wound closed in 2 layers. This surgery is easier, associated with less blood loss, heals better, and has lower rate of intrauterine & abdominal infection. Uterine rupture in future spontaneous labour is 5/1000, using oxytocin is 8/1000, with prostaglandins is 25/1000 b) classical - vertical incision in upper segment of uterus & child delivered through incision. Higher risk of morbidity postoperatively & risk of scar rupture. Indications for this type: I) transverse lie, ruptured membranes & liquor draining II) lower incision difficult due to structural abnormality III) constriction ring present due to neglected labour IV) fibroids in lower segment V) anterior placenta praevia with abnormally vascular lower segment VI) mother is dead & delivery needs to be rapid VII) very preterm foetus (especially breech) where lower segment poorly formed c) caesarian hysterectomy - can be performed at same time eg/ with uterine rupture, placenta accreta, uncontrollable postpartum haemorrhage, cervical malignant disease 3) complications - a) immediate - primary or secondary haemorrhage leading to shock, complications of anesthesia, damage to bladder/ureters/colon, vesico-uterine fistula, retained placental tissue & bleeding b) Rh activation in Rh -ve mother (give anti-D and Kleihauer test performed) c) risk of thromboembolism (may need heparin prophylaxis)

Methods of induction and augmentation of labour; indications, contraindications and complications

1) indications: a) prolonged pregnancy (>42 weeks) b) pre-eclampsia c) placental insufficiency & IUGR d) APH: placental abruption & APH of unknown origin e) Rh isoimmunization f) diabetes mellitus g) chronic renal disease 2) need cervical assessment - Bishop score (6+ is strongly predictive of labour following induction, <5 = needs cervical ripening) 3) methods: a) forewater rupture: finger through cervix & fetal membranes separated from lower segment, ruptured with Kocher's forceps b) hindwater rupture: rupture of membranes behind presenting part, Drewe-Smythe catheter into cervix, advantage = dec risk of prolapsed cord c) medical induction following amniotomy - combine surgical induction + Syntocinon infusion (inc strength until 3-4 contractions/10 mins, lasting >40 secs each) I) complications - hyperstimulation (fetal asphyxia), prolapse of cord, infection d) medical induction and cervical ripening - gold standard when membranes intact or cervix unsuitable for surgical induction - give: Syntocin infusion or prostaglandins or mechanical dilation of cervix I) syntocinin - induces uterine contractions, more effective with surgical induction II) prostaglandins - prostaglandin E2 to ripen cervix (SE = V + diarrhea, not commonly used), vaginal = pessaries or xylose gel into posterior fornix III) mechanical cervical ripening - insertion of balloon catheter through cervix - distends over 12 hours then removed for amniotomy

Significance and investigation of, intermenstrual, postcoital and postmenopausal bleeding, menstrual irregularity, heavy menstrual bleeding (menorrhagia) and dysmenorrhoea

1) intermenstrual bleeding (IMB): vaginal bleeding at any time during menstrual cycle other than during normal menstruation. Differentiate from metrorrhagia (irregular periods), affects 14% premenopausal women at some point a) causes - I) pregnancy eg/ ectopic pregnancy & gestational trophoblastic disease II) physiological eg/ spotting around ovulation, hormonal fluctuation during perimenopause (diagnosis of exclusion) III) vaginal causes eg/ adenosis, vaginitis, tumours IV) cervical causes eg/ infection - chlamydia, gonorrhoea, cancer, cervical polyps, cervical ectropion, condylomata acuminata of cervix V) uterine causes eg/ fibroids, endometrial polyps, cancer (endometrial), adenomyosis, endometritis, oestrogen-secreting ovarian cancers VI) iatrogenic eg/ tamoxifen, after smear or tx to cervix, missed OCP, drugs altering clotting eg/ anticoagulants, SSRIs, corticosteroids 2) postcoital bleeding (PCB): non-menstrual bleeding immediately after sex a) causes - infection, cervical ectropion (esp if taking COCP), cervical or endometrial polyps, vaginal or cervical cancer, trauma NB/ breakthrough bleeding: irregular bleeding associated with hormonal contraception 3) ix recommended for: a) women with an abnormal-looking cervix (urgent referral) b) women with a cervical polyp not easily removed in primary care c) women with a pelvic mass found on examination d) women at high risk of endometrial cancer - fx of hormone-dependent cancer, prolonged & irregular cycles, taking tamoxifen e) women > 45 with IMB and women < 45 with persistent symptoms or risk factors for endometrial cancer f) bleeding > three months' duration 4) ix - a) exclude pregnancy b) infection screen eg/ chlamydia, with IMB and PCB, gonorrhoeae c) bloods - FBC, clotting, TFT, FSH/LH levels (if onset of menopause suspected) d) TV US for structural abnormality - immediately postmenstrually, as endometrium at its thinnest. Evidence of endometrial thickening = biopsy e) endometrial biopsy - incidence of endometrial cancer rises sharply at 40 f) for women with persistent PCB, colposcopy recommended NB/ abnormal bleeding = PALM (structural) COEIN (non-structural) - Polyp, Adenomyosis, Leiomyomas (fibroids), Malignancy (& hyperplasia), Coagulopathy, Ovulatory dysfunction, Endometrial, Iatrogenic, Not yet specified

Investigation & management of urinary incontinence in women

1) involuntary leakage of urine, different types: a) functional incontinence - pt unable to reach toilet in time eg/ poor mobility b) stress incontinence - involuntary leakage of urine on effort, exertion, sneezing or coughing as incompetent sphincter (may have GU prolapse) c) urge incontinence - involuntary urine leakage + urgency of micturition, detrusor instability or hyperreflexia causing involuntary detrusor contraction, idiopathic or secondary to neurological problems eg/ stroke, Parkinson's, MS d) mixed incontinence - involuntary leakage of urine associated with urgency + exertion, effort, sneezing or coughing e) overactive bladder syndrome - urgency +/- urge incontinence, usually with frequency and nocturia, due to detrusor overactivity f) overflow incontinence - chronic bladder outflow obstruction eg/ from prostatic disease in men g) true incontinence - due to fistula between vagina & ureter, bladder, urethra 2) ix - a) hx - type, full obstetric history, bladder chart for 3+ days b) exam - digital assessment of pelvic floor muscle contraction, VE for prolapse, signs of vaginal atrophy, abdo, pelvic and neurological examination c) primary care - urinary dipstick testing, if s/s of UTI & +ve dipstick, MC&S d) post-void residual volume if s/s of voiding dysfunction or recurrent UTI e) urinary flow rates f) urodynamic studies: urodynamic testing including multi-channel cystometry, ambulatory urodynamics or video urodynamics g) cystoscopy 3) mx - temporary containment products (eg/ pads or collecting devices) to achieve social continence until diagnosis & mx plan a) stress incontinence - 3-months of pelvic floor muscle exercises (8 contractions, 3x/day) 1st line, duloxetine 2nd-line if don't want surgery or unsuitable, surgery (retropubic mid-urethral tape procedures) b) mixed incontinence - pelvic floor exercises and bladder training 1st-line, oxybutynin (anti-muscarinic) 2nd line NB/ newer antimuscarinics eg/ darifenacin, solifenacin if oxybutynin working c) overflow incontinence - relieving/treating obstruction, intermittent self-catheterisation may be carried out d) catheterization - intermittent catheterisation or indwelling urethral or suprapubic catheterisation for some eg/ persistent urinary retention, renal impairment e) botulinum toxin type A if neurological disease eg/ SC disease f) desmopressin for in women with troublesome nocturia

Management of collapsed obstetric pt

1) is patient unresponsive? a) if no - put onto left lateral position + call for help if appropriate + check maternal obs + assess fetal wellbeing + call for obstetric review b) if yes - open airway, look for signs of life (move to step 2) - ABCDE 2) wedge/tilt patient + call obstetric resuscitation team + call consultant obstetrician & anaesthetist + call neonatal team if >22 weeks of gestation 3) CPR 30:2 until defibrillator/monitor attached (NB/ give adrenaline every 3-5 mins) + 100% supplemental O2 + intubate early + insert 2 IV cannulae (wide-bore) a) if no response to CPR after 4 minutes, proceed to delivery/perimortem caesarean section 4) assess rhythm when defibrillator attached a) if shockable (VF/pulseless VT) - 1 shock 150-360 J biphasic or 360 J monophasic then return to CPR for 2 mins, then assess rhythm b) if non-shockable (PEA/asystole) - return to CPR for 2 mins, then assess rhythm c) return of spontaneous circulation - immediate postcardiac arrest treatment: ABCDE approach + controlled oxygenation & ventilation + 12-lead ECG + treat precipitating cause + temperature control/therapeutic hypothermia 5) if cardiac arrest - perimortem CS to deliver within 5 mins, otherwise as per non pregnant 6) remembermother is primary concern NB/ reversible causes - hypoxia, hypovolaemia, hypo-/hyperkalaemia/metabolic, hypothermia, thrombosis - coronary or pulmonary, tamponade - cardiac, toxins, tension pneumothorax

Detailed knowledge of the mechanisms, diagnosis and management of normal labour

1) mechanism - a) descent throughout pregnancy - engagement of head normally happens before onset of labour, descent of head gives measure of progress of labour b) flexion of head as descends & meets the medially & forward sloping pelvic floor, bringing chin into contact with fetal thorax. Flexion produces smaller diameter of presentation changing from occipito-frontal diameter when head is deflexed to suboccipitobregmatic diameter when head fully flexed c) internal rotation - head rotates as reaches pelvic floor & occiput normally rotates anteriorly from lateral position towards pubic symphysis - due to force of contractions transmitted via fetal spine to head at point where spine meets skull (which is more posterior), occasionally rotates posteriorly towards hollow of sacrum & head may deliver as a face-to-pubis delivery d) extension - acutely flexed head descends to distend pelvic floor & vulva, base of occiput comes in contact with inferior rami of pubis - head extends until delivered. Maximal distension of perineum & introitus accompanies final expulsion of head = crowning (head seen at introitus & doesn't recede between contractions) e) restitution - after head delivered it rotates back to be in line with normal relationship of fetal shoulders, direction of occiput following restitution points to position of vertex before delivery f) external rotation - when shoulders reach pelvic floor, they rotate to anteroposterior diameter of pelvis, accompanied by rotation of fetal head so face looks laterally (mum's thigh) g) delivery of shoulders - anterior shoulder delivered first by traction posteriorly on fetal head so shoulder emerges under pubic arch, posterior shoulder delivered by lifting head anteriorly over perineum, followed by rapid delivery of remainder h) 3rd stage of labour - placenta & membranes should be delivered within 5-10 minutes of fetus, if over 30 mins is abnormal 2) diagnosis - clinical signs: a) regular, painful contractions that inc in frequency & duration, and produce progressive cervical dilatation b) passage of blood stained mucus form cervix (the 'show') associated with labour c) rupture of fetal membranes (but may occur without uterine contractions) - if latent period between rupture of membranes to onset of painful contractions is >4hrs = prelabour rupture of membranes - can occur at term or in preterm period 3) management - a) antenatal classes - strategies to deal with pain, expulsion in 2nd stage etc, should contact hospital/midwife when contractions at 10-15 minute intervals or show or membranes rupture b) exams on admission: I) full general exam - temperature, pulse, RR, BP, state of hydration, urine tested for glucose, ketone bodies, protein II) obstetric exam of abdomen - inspection, palpation to determine fetal lie, presentation & position, station of presenting part by estimating 5ths of head palpable, auscultation of fetal heartbeat (stethoscope or Doptone device) III) vaginal exam - after cleansing vulva + introitus, aspectic technique - position, consistency & dilatation of cervix, if membranes intact or ruptured (if rupture the colour & quantity of amniotic fluid), fetal presentation (eg/ vertex, breech), position of presenting part & relationship to level of ischial spines, if vertex then degree of caput, moulding & synclitism noted, assessment of bony pelvis at upper, middle & lower pelvic outlet c) observation of progress of labour & intervention if slow d) monitoring fetal & maternal condition e) pain relief & emotional support for mother f) adequate hydration & nutrition throughout labour (IV fluids - saline or Hartmann's after 6hrs if delivery not imminent), check urine for ketones

Demonstrate understanding of the physiology of the menstrual cycle, including ovulation

1) menstrual cycle - day count for menstrual cycle begins on 1st day of menstruation. If cycle lasts 28 days each phase lasts: menstrual phase (from day 1 to 5), follicular phase (day 1 to 13), ovulation phase (day 14), luteal phase (day 15 to 28) 2) menstrual phase (day 1-5) a) uterus sheds inner lining of soft tissue & blood vessels - exits body from vagina in the form of menstrual fluid - blood loss of 10 ml to 80 ml is considered normal (contraction of uterine & abdominal muscles to expel blood) 3) follicular phase (day 1-13) a) pituitary gland secretes FSH & oestrogen that stimulates egg cells in the ovaries to grow b) 1 egg cell begins to mature in a sac-like-structure called follicle - takes 13 days to reach maturity c) while it matures, its follicle secretes a hormone that stimulates the uterus to develop a lining of blood vessels and soft tissue called endometrium. 4) ovulation - anti-mullerian hormone released by small ovarian follicles & decreases release of oestrogen a) at start of cycle low oestrogen causes +ve feedback on hypothalamus so GnRH stimulates anterior pituitary gland to produce gonadotrophins FSH + LH b) as follicle grows AMH dec & oestrogen inc = -ve effect on hypothalamus so dec FSH & LH produced c) usually only 1 follicle is large enough with enough gonadotrophin receptors to survive & continue to grow - development also regulated by inhibin B which suppresses FSH d) as follicle matures it inc production of oestradiol - causes +ve feedback of FSH & LH (peak) leading to rupture of ripe follicle (2cm diameter, due to LH) - swept into fallopian tube by cilia of fimbriae e) following ovulation follicle becomes corpus leuteum & releases oestrogen & progesterone to maintain secretory endothelium for embryo implantation - if doesn't happen corpus leuteum involutes & hormone levels fall = menstruation (14 days after ovulation). If implantation occurs hCG produced by trophoblast tissue acts as corpus leuteum to maintain oestrogen & progesterone levels until fetoplacental unit takes over at 8-10 weeks 5) luteal phase (day 15-28) a) egg cell released during ovulation stays in the fallopian tube for 24 hours - if a sperm cell does not impregnate the egg cell within that time, the egg cell disintegrates b) progesterone (causes uterus to retain its endometrium) gets used up by the end of the menstrual cycle - causes menstrual phase of the next cycle to begin NB/ importance of multidisciplinary team in treating subfertility

Detailed knowledge of the indications for pain relief/anesthesia

1) narcotics - pethidine or morphine or remifentanil (SE = N+V in mum, respiratory depression in baby), used in mothers who can't have regional analgesia eg/ on anti-convulsants 2) inhalation - used in early labour until mother switches to stronger analgesic - gives short-term relief. Entonox (50/50 nitrous oxide & O2), NO can cause dec fertility, bone marrow changes & neurological changes if used too long 3) non-pharmacological - transcutaneous electrical nerve stimulation - early labour (also acupuncture, subcut sterile water injections, massage & relaxation) 4) regional analgesia/epidural - most effective & widely used form (complete pain relief). May be started at any time, doesn't interfere with uterine contractions but may dec desire to bear down & dec uterine activity. Local anesthetic eg/ bupivacaine injected - addition of opioid dec dose requirement of bupivacaine, sparring motor fibres to lower libs a) insertion of IV cannula & preloading with <500ml of saline or Hartmann's b) insertion of epidural cannula at L3-4 space & injection of local anaesthetic c) monitor BP, HR, FHR, adjust maternal position for desired effect d) complications - hypotension (avoided by preloading), accidental dural puncture, postdural headache e) contraindications - maternal refusal, coagulopathy, local or systemic infection, uncorrected hypovolemia, inadequate or inexperienced staff or facilities 5) spinal analgesia - used for operative delivery 6) less frequently used - paracervical blockade, pudendal nerve blockade 7) repair of perineal wounds involves injection into perineal tissues over episiotomy site

Significance & management of vaginal discharge

1) normal vaginal discharge - caused by mucus from cervix, helps keeps vagina moist & protects from infection. Clear or white, not smelly, thick and sticky for most of menstrual cycle. Clearer, wetter and more slippery around ovulation - might not happen if using hormonal contraception. Most women produce more discharge during pregnancy and less after the menopause 2) unusual vaginal discharge - change in colour eg/ grey, green, yellow, pink or blood-tinged, unpleasant smell eg/ fishy or rotten meat smell, discharge thick or lumpy, like cottage cheese, excessive discharge, other symptoms eg/ itching and soreness around entrance of the vagina, dysuria, pelvic pain, and bleeding or spotting between periods or after sex a) tx depends on cause, but often involves a short course of medication b) abnormal vaginal discharge usually signifies infection eg/ bacterial vaginosis (bacterial infection of vagina), thrush (overgrowth of yeast in vagina), trichomoniasis (STI from parasite), gonorrhoea or chlamydia (STIs from bacteria), genital herpes (STI from virus) c) white or grey fishy-smelling discharge = bacterial vaginitis: abx d) thick white discharge ('cottage cheese') with itchiness = thrush: antifungals e) green, yellow or frothy discharge, foul smelling = trichomoniasis: abx f) abnormal discharge with pain or bleeding = chlamydia or gonorrhea: abx g) abnormal discharge with blisters = genital herpes: antivirals 3) young girls and postmenopausal women - unusual to have abnormal vaginal discharge - see GP

Management of postpartum sepsis

1) obtain blood cultures prior to antibiotic administration 2) administer IV broad-spectrum antibiotic within 1 hour of recognition of severe sepsis 3) measure serum lactate 4) in the event of hypotension and/or a serum lactate > 4 mmol/l: I) deliver an initial minimum 20 ml/kg of crystalloid or an equivalent II) apply vasopressors for hypotension not responding to initial fluid resuscitation to maintain MAP >65 mmHg 5) in the event of persistent hypotension despite fluid resuscitation (septic shock) and/or serum lactate greater than 4 mmol/l: I) achieve a central venous pressure of ≥ 8 mmHg II) achieve a central venous oxygen saturation ≥ 70% or mixed venous oxygen saturation ≥ 65% 6) the focus of infection should be sought and dealt with. This may be by uterine evacuation or by drainage of a breast, wound or pelvic abscess

Palliative and terminal care including symptom relief and the role of the hospice environment in women with gynaecological cancer

1) only 30% women cured after gynaecological carcinoma - ovarian cancers cause most deaths 2) palliative care - active total care of pt whose disease is incurable 3) aim is to inc QOL for pt & family - addressing symptoms eg/ pain, nausea, bleedings, symptoms of intestinal obstruction, as well as meeting pts social, psychological & spiritual needs - therefore care needs to be individualised 4) problems - prolonged poor QOL, euthanasia, symptom control vs drug SE, making transition from curative to palliative, and resource allocation 5) 3 levels of care working together - GP, specialist practitioners eg/ MacMillan nurses, and specialist hospices or gynaecology units 6) controlling pain - a) analgesic ladder: start with non-opioids eg/ NSAIDs, then to mild opioids eg/ low-dose codeine, then to moderate opioids eg/ high-dose codeine, then to strong opioids eg/ morphine b) co-analgesics eg/ antidepressants, steroids, and cytotoxics may be used c) opioid analgesic can be patient controlled & normally accompanied by antiemetics d) alternative therapies - acupuncture, behavioural techniques etc 7) controlling N+V - affects 60% pts with advanced carcinoma a) can be due to opiates, metabolic causes (eg/ uraemia), vaginal stimulation (eg/ bowel distension) or psychological factors, all of which should be addressed b) antiemetics include anticholinergics, anti-histamines, dopamine antagonists or 5HT-3 antagonists )eg/ ondansetron) 8) controlling heavy vaginal bleeding - may occur with advanced cervical & endometrial carcinomas a) high dose progesterone's may be helpful, radiotherapy used if hasn't been used before 9) controlling ascites & bowel obstruction - features of advanced ovaria carcinoma a) ascites best drained slowly by repeated paracentesis b) obstruction ideally managed at home - spontaneous resolution occurs in up to 1/3rd pts I) if obstruction is partial metoclopramide used (pro-motility & anti-emetic) and stool softeners with enemas for constipation, + trial of dexamethasone to dec tissue oedema II) if obstruction complete cyclizine + ondansetron used for N+V, with hyoscine for spasm. Pt encouraged to E+D small amounts if able III) surgical palliation indicated with acute, single-site obstruction, stents may be inserted low in sigmoid colon or rectum 10) controlling terminal distress - last 24hrs are those pts family will remember most - needs to be managed sensitively with time for pt & relatives in quiet environment, good symptom control with anxiolytics + analgesics without overly sedating

Complications of ovulation induction

1) ovarian hyperstimulation syndrome (5-10%), in mild forms ovaries are enlarged (5-12 cm) & may have ascites with mild abdominal distension, abdominal pain, nausea, and diarrhea. In severe forms may be haemoconcentration, thrombosis, distension, oliguria, pleural effusion, and respiratory distress - normally due to hCG 2) multiple pregnancy 3) medications used to induce ovulation can sometimes damage the lining of the uterus

Diagnosis and management of common causes of subfertility including ovulatory dysfunction, male factor subfertility, tubal disease, endometriosis, coital dysfunction and unexplained infertility

1) ovulatory dysfunction - 30% (most common cause) a) causes - hypothalamic pituitary failure, hypothalamic-pituitary-ovarian dysfunction eg/ PCOS, ovarian failure b) diagnosis - menstrual hx, bloods eg/ FSH & LH & progesterone (high LH relative to FSH = PCOS), endometrial biopsy I) home testing kits - detect inc urinary LH excretion 24 to 36 h before ovulation II) pelvic ultrasonography - monitor ovarians follicle diameter and rupture III) measure serum progesterone and urinary pregnanediol glucuronide IV) serum progesterone levels of ≥ 3 ng/mL (≥ 9.75 nmol/L) or inc levels of pregnanediol glucuronide in urine indicate ovulation has occurred c) mx - dec weight, PCOS - fertility drugs eg/ Clomiphene (trigger ovulations) + insulin sensitizing drugs eg/ Metformin for >6 months, Gonadotropins if clomiphene ineffective 2) male factor subfertility - no pregnancy after 1 year of regular sex a) diagnosis - signs of low hormone levels eg/ dec body hair & inc breast tissue, bloods, genetics, semen & sperm analysis, spermatic venography, scrotal US, urinalysis b) mx - abx for infection, testosterone if low, steroids if immune system, percutaneous embolization (varicocele), sperm extraction, surgery (repair a blockage in sperm duct or remove a pituitary tumor) 3) tubal disease - fallopian tube obstruction or epithelial dysfunction that impairs oocyte, zygote, and/or sperm motility a) diagnosis - hysterosalpingography 2 to 5 days after cessation of menstrual flow. Intrauterine and tubal lesions detected by sonohysterography, laparoscopy, and hysteroscopy b) mx - during surgery remove adhesions etc, IVF 4) endometriosis - functioning endometrial tissue is implanted in the pelvis outside uterine cavity, s/s: dysmenorrhea, dyspareunia, infertility, dysuria a) diagnosis - pelvic laparoscopy & sometimes biopsy. Areas of endometriosis on pelvic peritoneum are red, blue, or purplish brown spots > 5 mm (powder burn lesions) b) mx - I) NSAIDs for discomfort II) drugs to suppress ovarian function eg/ COP (recurrance) III) conservative surgical resection or ablation of endometriotic tissue IV) total abdominal hysterectomy +/- bilateral salpingo-oophorectomy if disease is severe and the patient has completed childbearing V) progestins, gonadotropin-releasing hormone (GnRH) agonists, danazol 5) coital dysfunction - difficulty experienced by an individual or a couple during any stage of sex, including physical pleasure, desire, arousal or orgasm a) diagnosis - sexual hx, assessing performance anxiety, guilt, stress and worry b) mx - I) males - psychotherapy, lifestyle eg/ stop smoking, drugs or alcohol, meds eg/ Viagra, intracavernous pharmacotherapy (injecting vasodilator drug directly into penis to stimulate an erection), insertion of a penile prosthesis, or penile implant, pelvic floor physical therapy if sexual problems and pelvic pain II) females - vacuum device for arousal and orgasm disorders, if pain with intercourse give pain relievers or desensitizing agents, lubricants, hormone therapy, counselling 6) unexplained fertility - a) diagnosis - semen in man is normal, ovulation and fallopian tubes are normal and ovulation is regular in woman b) mx - controlled ovarian stimulation: clomiphene + hCG to trigger ovulation, for up to 3 menstrual cycles I) if unsuccessful may use gonadotropins, then hCG, then intrauterine insemination within 2 days of hCG administration, then IVF

Non-surgical treatments for incontinence, including pads, garments, catheterisation, bladder retraining, pelvic floor exercises and drug treatment

1) pads & garments - temporary containment products (eg/ pads or collecting devices) to achieve social continence offered until specific diagnosis & management plan. Permanent use of containment products considered after exclusion of other methods of management 2) catheterisation - may use intermittent self-cathetrisation if overflow, indwelling catheters (urethral or suprapubic) may be indicated if: chronic urinary retention (& pt can't perform self-catheterisation), skin wounds, pressure sores or skin irritations contaminated by urine, distress or disruption caused by changing clothes and the bed 3) bladder retraining - aim is for pt to be able to hold on for longer, have less urinary leakage & more control of their bladder - slowly stretch the bladder so it can hold larger volumes of urine 4) pelvic floor exercises - 3-month trial = 1st-line treatment (subsequent to digital assessment of pelvic muscle contraction). 8 contractions 3x/day, continue if successful or consider electrical stimulation and/or biofeedback in women who cannot actively contract pelvic floor muscles 5) drugs - stress incontinence = duloxetine 2nd-line in women who do not want surgery or who are unsuitable for surgery, mixed = antimuscarinic oxybutynin if PFE & bladder retraining don't work NB/ botulinum toxin type A is sometimes used in some patients with neurological disease eg/ SC disease NB2/ desmopressin may be prescribed in women with troublesome nocturia

Mechanisms to promote coping with incontinence and retention of urine

1) pads & garments to achieve social continence 2) pelvic floor exercises 3) try & only go to the loo 4-5x/day 4) double voiding 5) make it easy to go to the loo - wear easy-to-pull off clothes eg/ elastic waistbands and Velcro closures 6) weight loss 7) avoid caffeine (diuretic) 8) keep hydrated 9) talk to doctor to make sure you're not taking any prescription or over-the-counter medicines that could be making your urinary incontinence worse 10) have enough fruit, vegetables and fibre in your diet to encourage regular bowel movements as constipation can make incontinence worse as it puts pressure on the bladder 11) stop smoking

Detailed knowledge and understanding of emergency investigation and management of pelvic infection

1) pelvic inflammatory disease (PID) - general term for infection of upper female genital tract, including uterus, Fallopian tubes, and ovaries. Usually results from ascending infection from cervix. Common & serious complication of some sexually transmitted diseases eg/ chlamydia and gonorrhoea. Can damage Fallopian tubes & tissues in and near uterus & ovaries. Untreated PID can lead to serious complications eg/ infertility, ectopic pregnancy, abscess formation and chronic pelvic pain 2) often polymicrobial - can be caused by genital mycoplasmas, endogenous vaginal flora (anaerobic and aerobic bacteria), aerobic streptococci, Mycobacterium tuberculosis, and Chlamydia trachomatis or Neisseria gonorrhoeae 3) risk factors - young age, new sexual partner, multiple sexual partners, lack of barrier contraception, lower socio-economic group, TOP 4) diagnosis of acute PID made only on clinical signs and positive swab results - many episodes go unrecognised, as women often have absent, mild, or atypical symptoms a) s/s - bilateral lower abdo pain or tenderness, deep dyspareunia, abnormal vaginal bleeding (postcoital, intermenstrual or menorrhagia), vaginal or cervical discharge that is purulent, mucopurulent cervical discharge & cervicitis seen on speculum exam, cervical motion tenderness and adnexal tenderness on bimanual vaginal examination, fever > 38°C 5) ix - a) pregnancy test (pregnant women with PID should be admitted; ectopic pregnancy may be confused with PID) b) cervical swabs for chlamydia & gonorrhoea (+ve result supports diagnosis of PID), endocervical swabs for C. trachomatis and N. gonorrhoeae using NAAT c) inc ESR or CRP may support diagnosis of PID d) endometrial biopsy and US may also be helpful e) laparoscopy with direct visualisation of Fallopian tubes = best diagnostic test f) urinalysis and urine culture to exclude urinary tract infection 6) mx - mild or moderate disease tx in primary care or outpatients, severe disease requires hospital admission for IV abx a) adequate pain relief b) evidence for whether IUCD should be left in situ or removed is limited c) consider referral to a GUM clinic, for a full sexually transmitted infection screen (HIV, etc), contact tracing and treatment of sexual partners d) abx - empirical until results - I) outpatient = IM ceftriaxone, then doxycycline & metronidazole for 2/52 II) severely ill pts = IV doxycycline, ceftriaxone, metronidazole, then change to oral doxycycline and metronidazole for 2/52 e) if fail to respond to tx, laparoscopy essential to confirm diagnosis f) sexual partners - avoid unprotected intercourse until they & their partner(s) have completed tx and follow-up, screen for other STIs 7) complications - infertility, ectopic pregnancy, chronic pelvic pain, perihepatitis (Fitz-Hugh Curtis syndrome - RUQ pain), tubo-ovarian abscess, Reiter's syndrome, preterm delivery, neonatal: perinatal transmission of C. trachomatis or N. gonorrhoeae = ophthalmia neonatorum. Chlamydial pneumonitis

Principles and legal issues surrounding informed consent

1) process for getting permission before conducting a healthcare intervention on a person 2) informed consent can be said to have been given based upon a clear appreciation and understanding of the facts, implications, and consequences of an action. Adequate informed consent is rooted in respecting a person's dignity 3) to give informed consent, the individual must have adequate reasoning faculties and be in possession of all relevant facts. Impairments to reasoning and judgment that may prevent informed consent include basic intellectual or emotional immaturity, high levels of stress such as posttraumatic stress disorder (PTSD) or a severe intellectual disability, severe mental disorder, intoxication, severe sleep deprivation, Alzheimer's disease, or being in a coma 4) in cases where an individual is considered unable to give informed consent, another person is generally authorized to give consent on his behalf, e.g., parents or legal guardians of a child 5) when an individual is provided insufficient information to form a reasoned decision, serious ethical issues arise. Such cases in a clinical trial in medical research are anticipated and prevented by an ethics committee or Institutional Review Board 6) for an individual to give valid informed consent, 3 components must be present: disclosure, capacity and voluntariness a) disclosure requires the researcher to supply the subject with the information necessary to make an autonomous decision & ensure that subjects have adequate comprehension of the information provided (eg/ consent form written in lay language suited for comprehension skills of subject population, as well as assessing the level of understanding during the meeting) b) capacity - ability of the subject to both understand the information provided and form a reasonable judgment based on the potential consequences of his/her decision c) voluntariness - subject's right to freely exercise his/her decision making without being subjected to external pressure such as coercion, manipulation, or undue influence

Anatomy of the pelvis

1) pelvis made of 3 bones - 2 innominate bones comprising ilium, ischium & pubis, and sacrum, 2 innominate bones joined anteriorly at symphysis pubis 2) pelvic brim formed by pubic crest, pectineal line of pubis, arcuate line of ilium & alae of sacrum 3) pelvic brim separates true pelvis (below) from false pelvis (above) 4) inferiorly it's separated from perineum by urogenital diaphragm 5) plane of pelvis is 55 degrees with horizontal 6) pelvis anatomically projects backwards from pelvic brim - upper border of symphysis pubis, ischial spines, tip of coccyx, head of femur & greater trochanter lie in same plane 7) female pelvis is different to male pelvis - broader & bones are more slender, outline of male is heart shape & brim widest at the back - in women it's transversely oval (widest at front), subpubic arch is acute (Gothic arch) in men & rounder (Roman arch) in women 8) bony pelvis is not distensible - only minor degrees of movement at symphysis pubis & sacroiliac joints - dimensions therefore critical at childbirth 9) 4 basic shapes of pelvis - a) gynaecoid - classic female pelvis, inlet transversely oval & roomier pelvic cavity b) android - inlet is heart shaped, cavity is funnel-shaped with contracted outlet c) anthropoid - results from high assimilation ie/ sacral body assimilated to 5th lumbar vertebra, long, narrow & oval in shape d) platypelloid - wide pelvis flattened at brim with promontory of sacrum pushed forward 10) pelvic walls - inner aspect of pelvic bones are covered with muscles, above brim = psoas and iliacus, sidewalls = obturator internus (+ fascia), curved posterior wall = pyriformis, pelvic floor = levator ani + coccygeus 11) vaginal delivery - true pelvis bounded anteriorly by symphysis pubis (3.5 cm long) & posteriorly by sacrum (12 cm long) a) zone of inlet made anteriorly by upper border of pubis, posteriorly by sacral promontory, laterally by iliopectineal line. Transverse diameter is 13.5cm & AP diameter is 11.5cm b) zone of cavity most roomy just below inlet zone & appears almost round with transverse diameter of 13.5cm & AP diameter of 12.5cm c) zone of mid pelvis bounded by apex of pubic arch anteriorly, tip of sacrum posteriorly, ischial spines laterally. Ovoid & narrowest part of pelvis d) zone of outlet has pubic arch (85°) as anterior border, sacrotuberous ligaments & ischial tuberosities delineate posterolateral margins, leading to coccyx posteriorly 12) ideal female pelvis - can accommodate head of fetus at term, oval brim, shallow cavity, non-prominent ischial spines, curved sacrum with large sciatic notches (>90°) & sacrospinous ligament > 3.5cm long. Angle of pelvic brim is 55° to horizontal. AP diameter is 13.5cm. subpubic arch is well rounded and >90°, ischial intertuberous diameter is >10cm 13) pelvis clinically favourable if - sacral promontory can't be felt, ischial spines not prominent, subpubic arch & base of sacrospinous ligaments both accept 2 fingers & intertuberous dimeter accepts 4 knuckles on pelvic examination

Management of perineal trauma

1) perineal tears: 1st degree - skin only 2nd degree - perineal muscles (same as episiotomy) 3rd degree - leads to partial (a) or complete (b) tear of anal sphinctor 4th degree - as 3rd but extends into rectal canal 2) mx - a) adequate exposure, lighting, analgesia b) idetify & secure apex c) deep perineal tissues sutured with interrupted stitches (absorbable synthetic suture material - polygalactin) d) vaginal wall has continuous locking stitches for better haemostasis + prevent vaginal shortening e) perineal skin has interrupted mattress sutures f) on finishing repair do VE to check haemostasis, alignment, and DRE to chec no sutures accidentally went into rectum g) instruments, swabs & needles checked at end, document analegesia given & blood loss NB/ 3rd + 4th degree - Abx + laxatives for 1 week, physio for pelvic floor exercises, seen in 6-8 weeks

Roles of other healthcare professionals in the management of the woman with incontinence, e.g. physiotherapists, continence advisors etc.

1) physios & continence advisors - a) training & strengthening pelvic floor muscles = 1st-line for women with stress, urge or a mixture of stress and urge urinary incontinence b) public health messages that improve lifestyle and wellbeing eg/ weight loss, dec caffeine / fluid intake, cessation of smoking, inc physical exercise 2) GPs - general advice on controlling symptoms of incontinence, provide information on pelvic floor exercises and bladder retraining, provide treatment for incontinence with prescribed medicines eg/ oxytocin

Diagnosis, impact and management of pre-eclampsia and eclampsia

1) pre-eclampsia - disorder of widespread vascular endothelial malfunction and vasospasm that occurs after 20 weeks' gestation, affecting placenta a) diagnosis - I) SBP >140 mmHg or DBP >90 mm Hg on 2+ occasions at least 4 hrs apart in a previously normotensive patient OR SBP >160 mm Hg or DBP >110 mm Hg II) + proteinuria >300mg in 24-hr urine specimen, protein/creatinine ratio >0.3, or urine dipstick protein of 1+ III) with or without pathologic oedema b) impact - preeclampsia without severe features may be asymptomatic, severe features display end-organ effects & s/s: headache; visual disturbances; altered mental status; blindness; SOB; oedema; epigastric or RUQ abdo pain; clonus c) mx - delivery is only cure - if no severe features induced after 37 weeks, beforehand admit & monitor carefully, foetus given corticosteroids to accelerate lung maturity in preparation for early delivery if severe features, induction of delivery after 34 weeks may need medication eg/ labetalol. nifedapine 2) eclampsia - seizures (not attributable to other causes) in a woman with preeclampsia. HELLP syndrome (hemolysis, elevated liver enzyme, low platelets) may complicate severe preeclampsia a) diagnosis - complication of severe preeclampsia - new onset grand mal seizure activity +/- unexplained coma, mostly 3rd trimester b) impact - HELLP c) mx - eclamptic convulsions are life-threatening emergencies I) delivery is the only definitive treatment II) magnesium sulfate, or 2nd line phenytoin & diazepam/lorazepam III) control of HTN - hydralazine, labetalol, nifedipine IV) dose of antenatal steroids in anticipation of emergent delivery if <32 weeks eg/ betamethasone V) regularly check neurologic status for s/s of inc ICP or bleed VI) C-section if unfavorable cervix & gestational <30 weeks, or seriously ill

Diagnosis and management of patients presenting with primary and secondary amenorrhoea

1) primary - failure of menses to occur by 16, in the presence of normal growth and secondary sexual characteristics. If by age 13 menses has not occurred and the onset of puberty eg/ breast development, is absent, a workup for primary amenorrhea should start a) tests - I) B-hCG - +ve = pregnant II) TSH - abnormal = thyroid III) prolactin - abnormal = MRI of pituitary, or meds, tx = stop meds, surgery, dopamine agonists b) if no uterus - karyotype & testosterone levels: I) 46XX = mullerian agenesis II) 46XY = androgen insensitivity, inc testosterone c) if uterus present - FSH & LH levels I) low FSH & LH = functional amenorrhoea (energy deficit), costiutional delay in puberty, primary GnRH deficiency II) normal FSH & LH = outflow tract obstruction (imperforate hymen, mullerian agenesis) - tx = surgery III) high FSH & LH = primary ovarian insufficiency - karyotype for Turner's or presence of Y chromatin b) mx - if pathology cannot be reversed give HRT to achieve peak bone density, and GnRH therapy to induce ovulation 2) secondary - cessation of menses after menarche. Oligomenorrhea = menses occurring at intervals longer than 35 days apart a) tests - as above: B-hCG, prolactin, TSH b) inc FSH & LH (repeated after 1/12) = primary ovarian insufficiency, menopause, tx = karyotyping c) normal or low FSH & LH - I) functional eg/ low intake, excess exercise II) inc ICP (headache, visual changes) - MRI head for neoplasm III) hx of obs or gynae procedures = ?hysterectomy as Asherman syndrome IV) hyperandrogenism = order testosterone. DHEA-S, 17-hydroxyprogesterone - -inc 17-hydroxy- = late onset congenital adrenal hyperplasia -PCOS (anti-müllerian hormone levels) -rapid onset = ?ovarian or adrenal tumour

Deviations from normal labour

1) prolonged latent phase - latent phase = period of time starting with the onset of regular uterine contractions and ending with the onset of the active phase (usually 3-4 cm cervical dilation) a) prolonged latent phase = >20 hours if nulliparas or 14 hours if multiparas b) most common reason is entering labor without substantial cervical effacement 2) power - uterine contractility x frequency of contractions a) Montevideo units (MVUs) - strength of contractions in mmHg x frequency per 10 minutes as measured by intrauterine pressure transducer b) uterine contraction pattern should repeat every 2-3 minutes c) uterine contractile force produced >200 MVUs/10 min for active labor to be adequate eg/ 3 contractions in 10 minutes that reach a peak of 60 mm Hg are 60 X 3 = 180 MVUs d) an arrest disorder of labor cannot be diagnosed until pt is in active phase & contraction pattern >200 MVUs for 2+ hours with no cervical change 3) pelvis or size of passageway inhibiting delivery a) shape of bony pelvis (eg/ anthropoid or platypelloid) can result in abnormal labor b) if pt is extremely short or obese, or who has had prior severe trauma to the bony pelvis 4) size and/or presentation of the infant a) abnormal labor could be secondary to the size of the infant, and/or presentation. Fetal presentation may include asynclitism or head extension b) fetal macrosomia & other anomalies (eg/ hydrocephalus, encephalocele, fetal goiter, cystic hygroma, hydrops, or any other abnormality that = inc size of infant) are likely to cause deviation from the normal labor curve 5) other factors - low-dose epidural or combined spinal-epidural anesthetics that minimize motor block & may contribute to prolonged 2nd stage - associated with inc in oxytocin use & operative vaginal delivery

Knowledge of benign conditions of lower genital tract including pruritus vulvae, vaginal discharge (both physiological and pathological)

1) pruritus vulvae - itchiness of vulva (mons pubis, labia, clitoris, perineum & external openings of urethra + vagina), patch testing may be used for cause a) causes - mostly symptom of underlying condition eg/ diabetes, lymphoma. Moisture left on skin eg/ perspiration, urine, vaginal discharge or stool. Vaginal infections, vulvitis,, Bowen's disease, or diet (caffeine, potatoes, chilli, capsicum) b) tx - symptomatic relief, dec inflammation, restore skin barrier & prevent secondary infection I) symptomatic tx - emollient + antihistamine II) consider 1/52 low potency topical corticosteroids (hydrocortisone 1% ointment) - can add antifungal or antibacterial c) chronic inflammation of vulva predisposes to premalignant changes 2) vaginal discharge - common, physiological or pathological. Most common causes are normal physiological discharge, bacterial vaginosis and candidal infections. STIs & non-infective causes should also be considered NB/ normal physiological discharge is a white or clear, non-offensive discharge that varies with the menstrual cycle a) non-infective causes - physiological: I) newborns may have a small amount of vaginal discharge for 1/52 II) when oestrogen is low, mucus is thick and sticky. As oestrogen inc mucus gets clearer, wetter and more stretchy. After ovulation, mucous is thick & sticky III) at menopause vaginal discharge dec as oestrogen dec b) cervical polyps and ectopy c) foreign bodies eg/ retained tampon (foul-smelling serosanguinous discharge) d) vulval dermatitis e) erosive lichen planus f) genital tract malignancy eg/ cancer of the cervix, uterus or ovary. g) fistulae 3) non-sexually transmitted infections - BV (most common pathological cause - fishy smelling & profuse) or candidal infections (thick, curd like) 4) STIs - Chlamydia (copious purulent discharge), gonorrhoeae (purulent discharge), Trichomonas vaginalis (offensive yellow discharge) 5) symptoms suggesting discharge is abnormal: heavier or thicker than normal, pus like, white or clumpy, greyish or yellowish or blood-tinged, foul-smelling NB/ vaginal discharge following miscarriage, abortion or delivery must be fully investigated and empirically treated whilst awaiting results of swabs 6) ix - STI screening & swabs, bloods for HIV and syphilis 7) mx - a) if suggestions of STI or recurrent infections, refer to GUM service b) infective, non-sexually transmitted causes of vaginal discharge - metronidazole & clindamycin for BV c) vaginal & oral azole antifungals for vaginal candidiasis d) retained foreign bodies - manually removed e) cervical polyps easily removed - polyp should be sent for histology f) cervical ectopy - cryocautery and silver nitrate

Psychosocial impact of sexually transmitted infections and living with HIV/AIDS

1) psychosocial impact - severe stress & depression are two of the conditions that people have as a result of becoming infected with an STD. For some, a diagnosis can even be traumatic. Social stigma when it comes to living with STDs, many people have a difficult time with their diagnoses, both physically and mentally 2) HIV/AIDs - if pt manages condition properly by taking medication correctly & avoiding illness, should be able to live a near-normal life a) improve general health & dec risk of falling ill - exercising regularly, eating a healthy, balanced diet, stopping smoking b) cannot - donate blood or organs, join the armed forces, get life insurance to cover a mortgage loan (more difficult), visit certain countries c) long-term condition so pts in regular contact with healthcare team, who review treatment on an ongoing basis d) healthcare team can provide counselling e) some people find it helpful to talk to other people who have HIV, either at a local support group or on an internet chatroom

Impact of analgesia and anaesthesia on labour

1) regional - spinal or epidural: a) spinal - gives loss of dura sensation for 2hrs b) epidural - goes into ligamentum flavum then diffuses across to dura, lasts 12-24hrs in labour but can top up as leave catheter in 2) GA - can't have partner in theatre, less safe - pregnancy compresses diaphragms so lose residual volume in 3rd trimester, but have high need of O2 so can desaturate quickly & more likely to aspirate, also 4x more difficult to intubate BUT - much quicker in an emergency

Stages of labour

1) regular painful contractions & cervical changes until reaches full dilatation & no longer palpable, this stage is divided into early latent phase when cervix becomes effaced & shorter from 3cm in length to full dilatation or 10cm (~1cm per hour) 2) from full cervical dilatation to delivery of fetus - subdivided into pelvic or passive phase when head descends down pelvis & active phase when mother gets a stronger urge to push & fetus delivered with force of uterine contractions & maternal bearing down effort 3) from delivery of fetus to delivery of placenta & membranes

Use and limitations of ultrasound scanning in pregnancy

1) reliably used to determine gestation, pregnancy site, and exclude multiple pregnancy 2) cornerstone for screening for trisomies - nuchal translucency (space between skin & soft tissue overlying cervical spine) measured between 11 and 13+6 weeks - larger it is = inc risk. large nuchal translucency also indicates inc risk of structural, particularly cardiac, abnormalities. 50% foetuses with trisomies have structural abnormalities eg/ exomphalos 3) amniocentesis & chorionic villus sampling performed using us vision 4) used to diagnose structural abnormalities ~20 weeks. congenital abnormalities of all organs & systems detectable. however, many defects eg/ heart, may remain undiagned at 20 weeks - related to operator experience. polyhydramnios in later pregnancy can be the result of fetal abnormality - warrants repeat detailed us

Risk factors, symptoms, investigation and management of vulval cancer

1) risk factors - a) VIN = pre-malignant state b) lichen sclerosus c) HPV infection d) Paget's disease of the vulva 2) s/s - a) may present with a vulval lump, vulval bleeding due to ulceration, pruritus or pain b) presentation is often delayed. c) high index of suspicion for abnormal lesions on the vulva, including 'warts' in postmenopausal woman 3) Ix - a) diagnosis made by exam & biopsy b) cystoscopy, proctoscopy, CXR and MRI scans are used for staging purposes 4) Tx - standard treatment is surgery - radical mutilating surgery is now being replaced by a conservative and individualised approach a) primary vulval cancer treated with radical/wide local resection - may need radical vulvectomy b) reconstructive surgery often undertaken c) radiotherapy +/- chemotherapy when advanced d) appropriate groin node dissection & sentinel lymph node biopsy (SLNB)

Detailed understanding of the issues relating to pregnancy complicated by diabetes mellitus

1) risks - a) 1st trimester = inc insulin requirements; miscarriage, fetal abnormality (NTDs + congenital heart disease) b) 2nd/3rd trimester = preeclampsia, recurrent infection, worsening retinopathy; macrosomia, polyhydramnios, stillbirth, growth restriction c) labour = induction, poor progress; preterm d) delivery = instrumental birth, birth trauma C-section; shoulder dystonia e) postnatal = return to pre-pregnancy control within hours; neonatal hypoglycaemia, NICU, RDS, jaundice f) longer term = diabetes in childhood 2) dec hypo symptoms 3) management - a) preconceptual counselling - aim to get HbA1C <6.1%, insulin & metformin safe in pregnancy, ACEI stopped, 5mg periconceptual folic acid b) multidisciplinary team working - maintain normal ranges without hypoglycaemia, fetal testing for chromosomal problems in 1st trimester & routine anomaly scan at 20 weeks, can have additional cardiac scans, regular serial growth scans for macrosomia & fetal growth restriction c) low dose aspirin 2nd trimester (dec risk of preeclampsia), ophthalmic assessment every trimester, delivery 38-39 weeks recommended, NVD planned but C-section often happens, strict glycemic control in labour - sliding scale infusion of insulin-dextrose

Symptoms, signs, associations and treatment of problems of the menopause (climacteric)

1) s/s - during menopause transition, cycles remain regular but interval between cycles begins to lengthen a) vagina and uterus - in transition can have shorter cycling, irregular bleeding, spotting or bleeding - related to vaginal atrophy, polyp or lesion, or functional endometrial response b) during the menopause can have - dysparenuia, vaginal dryness, atrophic vaginitis - thinning of membranes of the vulva, vagina, cervix & outer urinary tract, and shrinking and loss in elasticity of all of outer & inner genital areas c) others - lack of energy, stiffness, breast enlargement & pain, heart palpitations, headache, dizziness, dry, itchy, thinning & tingling skin, weight gain, urinary incontinence, & urgency, interrupted sleeping patterns, night sweats, hot flashes d) psychological - anxiety, poor memory, inability to concentrate, depressive, irritability, mood swings, less interest in sexual activity 2) long term - inc risk of atherosclerosis, acute MI & CV diseases, osteopenia, osteoporosis, accelerated lung function decline NB/ if have menopause <45 = inc risk of heart disease & death 3) tx - perimenopause is natural, not a disease - only give tx if physical, mental, and emotional effects are strong enough to significantly disrupt life a) HRT - oestrogen in women without a uterus & oestrogen + progestin in women with intact uterus. Tx menopausal symptoms eg/ hot flashes, prevents bone loss NB/ inc risk of strokes and blood clots 2) selective oestrogen receptor modulators - selective agonists or antagonists of oestrogen receptors throughout the body eg/ raloxifene & tamoxifen a) raloxifene - oestrogen agonist on bone and lipids, antagonist on breast & endometrium b) tamoxifen - tx for hormone sensitive breast cancer c) other meds - some SSRIs & SNRIs provide some relief eg/ low dose paroxetine, but less effective than hormone therapy d) other tx - vaginal moisturizers (dryness), lubricants (for sex), low-dose prescription vaginal oestrogen products eg/ creams (help vaginal thinning & dryness), lifestyle measures eg/ drinking cold liquids, staying in cool rooms, using fans, avoid hot flash triggers eg/ hot drinks, spicy foods, counseling or support groups, osteoporosis dec - smoking cessation, vit D, weight-bearing exercise

Human papillomavirus screening and immunisation

1) screening - a) cervical cancer: can be detected with routine cervical cancer screening (Pap test) and follow-up of abnormal results. The Pap test can find abnormal cells on the cervix so they can be removed before cancer develops. Abnormal cells often become normal over time, but can sometimes turn into cancer. A HPV DNA test, which can find certain HPV types on a woman's cervix, may also be used with a Pap test in certain cases (co-testing). anal & penile cancers: no routinely recommended screening test 2) immunization - HPV vaccine prevents against HPV types 16 & 18 (and sometimes lower risk ones such as types 6 and 11) a) protects against: cancers of the cervix, vagina, and vulva in women; cancers of the penis in men; cancers of the anus and back of the throat, including the base of the tongue and tonsils (oropharynx), in both women and men b) all girls 12-13 given injections = 2 injections within a 6-24-month period

Detailed knowledge of abnormal labour (obstetric emergencies)

1) shoulder dystocia - delivery where additional manouveres required after delivery of head, anterior shoulder of infant cannot pass below, or requires significant manipulation to pass below pubic symphysis a) risks - macrosomia, maternal diabetes, BMI >30, prolonged 1st or 2nd stage, IOL/ augmentation, instrumental delivery b) mx - HELPERR - Help, Evaluate for episiotomy, Legs (McRoberts), Pressure (suprapubic), Entry (attempt internal rotation), Removal (posterior arm), Roll the patient (to all fours) I) entry - Rubin II, Woods' screw maneuver c) complications - damage to upper brachial plexus nerves, can't open chest to breathe, brain damage (asphyxiation) 2) uterine inversion - placenta fails to detach from uterus as it exits, pulls on the inside surface, & turns organ inside out a) causes - mismanagement of 3rd stage of labor eg/ fundal pressure, excess cord traction, uterine weakness, precipitate delivery, short umbilical cord b) complications - often associated with significant PPH c) mx - treat shock & replace uterus - true obstetrical emergency, so extra doctors, nurses, anesthesiologists should be summoned to room to assist once you have achieved uterine relaxation, place your fist into the vagina, find the biggest part of the inversion & push with your fist cephalward to replace uterus, then continue more force toward the fundus to replace the uterus if external replacement fails, a laparotomy may be required - uterus is gently pulled right way round using forceps 3) retained placenta - all or part of placenta or membranes remain in uterus during 3rd stage after 30 mins a) complications - hemorrhage and infection b) mx - no effective pharmacological treatment, ensure bladder is empty, controlled cord traction, manual extraction if cord traction also fails, very rarely a curettage is necessary to ensure no remnants of placenta remain 4) cord prolapse - umbilical cord comes out of uterus with or before presenting part of fetus a) causes - breech/ abnormal lie, pre-term rupture of membranes, multiparity, low birth weight (<2.5kg at >37 weeks), multiple pregnancy (esp 2nd twin), polyhydramnios, congenital anomalies, procedure related - ARM, ECV, internal podalic version b) complications - cord compression can = inadequate blood supply, and thus oxygen c) mx - gold standard = immediate delivery by quickest & safest route (usually cesarean), techniqes that can be employed - manual elevation of presenting fetal part, repositioning of mother to be head down with feet elevated, filling of bladder with a foley catheter, or tube through urethra to elevate presenting fetal part

Problems relating to breastfeeding & breastfeeding initiatives

1) sore or cracked nipples - most common 3-7 days into breastfeeding. Usually happens if baby isn't well positioned and attached at the breast 2) not enough breast milk 3) breast engorgement - happens when breasts get too full of milk. May feel hard, tight and painful 4) baby not latching on properly 5) too much breast milk 6) breastfeeding and thrush - can sometimes happen when nipples become cracked or damaged. This means the candida fungus that causes thrush can get into nipples or breast 7) blocked milk duct - may feel a small, tender lump in your breast 8) mastitis (inflammation of the breast) - when a blocked milk duct isn't relieved. It makes the breast feel hot and painful, and can make pts feel very unwell with flu-like symptoms 9) breast abscess - if mastitis is not treated, or if it doesn't respond to treatment, it can lead to a breast abscess, which may need an operation to drain it 10) breastfeeding and tongue tie - in 4-11% of babies, the strip of skin that attaches the tongue to the floor of the mouth (frenulum) is shorter than usual 11) initiatives - start4life, national breastfeeding helpline, association of breastfeeding mothers

Principles of neonatal resuscitation

1) start with conservative measures: dry, warmth, stimulate 2) if no meconium stained liquor: a) start clock & transfer to resuscitaire b) dry baby & wrap in warm towel c) A (face upwards, head in neutral position) B (assess rate & quality) C (listen to apex beat, palpate brachial or femoral pulse, colour of trunk + limbs + tongue) 3) next actions depend on which category baby is in: a) healthy (pink, crying, HR >100bpm) - dry & give to mother b) primary apnoea (cyanosed, HR <100bpm, some respiratory effort, response to stimulation) - gentle stimulation & facial oxygen, if no response after1 minute use bag & mask ventilation c) terminal apnoea (pale, HR <60bpm, floppy & apneic) - bag & mask ventilation, may need intubating & cardiac compression d) fresh stillbirth - full CPR immediately; positive pressure ventilation via ETT + cardiac compressions 4) if 100% O2 ventilation & CPR failing (<60bpm) to work need to use drugs: a) adrenaline IV (10ug/kg) or endotracheal tube (20ug/kg) every 3-5 mins if no response b) sodium bicarbonate if HR <60bpm after adrenaline - 1mmol/kg IV c) volume expanders - 0.9% saline or 4.5% human albumin solution (10-20mL/kg over 5-10 mins) d) dextrose IV 10% at 2-3mL/kg e) naloxone if apnoea related to maternal opiate use - 100ug/kg IM

Artificial reproduction techniques

1) technology used to achieve pregnancy in procedures such as fertility medication, IVF, and surrogacy. Reproductive technology used primarily for infertility treatments, also known as fertility treatment. Mainly belongs to the field of reproductive endocrinology and infertility, and may also include intracytoplasmic sperm injection (ICSI) and cryopreservation. Some forms of ART are also used with regard to fertile couples for genetic reasons (preimplantation genetic diagnosis). ART is also used for couples who are discordant for certain communicable diseases; for example, HIV to reduce the risk of infection when a pregnancy is desired 2) process of sexual intercourse is bypassed and fertilization of the oocytes occurs in the laboratory environment (i.e., in vitro fertilization) 3) in general, involves surgically removing eggs from a woman's ovaries, combining them with sperm in the laboratory, and returning them to the woman's body or donating them to another woman, doesn't include treatments in which only sperm are handled (i.e., intrauterine—or artificial—insemination), procedures in which a woman takes medicine only to stimulate egg production without the intention of having eggs retrieve, or artificial insemination 4) IVF: transvaginal ovum retrieval (OVR - small needle inserted through the back of the vagina and guided via US into ovarian follicles to collect the fluid that contains the eggs); embryo transfer (1+ embryos are placed into the uterus of the female with the intent to establish a pregnancy); sometimes assisted zona hatching (AZH - performed shortly before the embryo is transferred to the uterus, small opening made in the outer layer surrounding egg to help the embryo hatch out and aid in the implantation process of the growing embryo) 5) intracytoplasmic sperm injection (ICSI) - beneficial with male factor infertility where sperm counts are very low or failed fertilization occurred with previous IVF attempt(s). Single sperm carefully injected into the center of an egg using a microneedle 6) autologous endometrial coculture - pts who have failed previous IVF attempts or have poor embryo quality. Pt's fertilized eggs placed on top of a layer of cells from the patient's own uterine lining, creating a more natural environment for embryo development 7) in zygote intrafallopian transfer (ZIFT) - egg cells removed from woman's ovaries & fertilized in the laboratory; the resulting zygote is then placed into the fallopian tube 8) cytoplasmic transfer - contents of a fertile egg from a donor are injected into the infertile egg of the patient along with the sperm 9) preimplantation genetic diagnosis (PGD) - genetic screening mechanisms such as fluorescent in-situ hybridization (FISH) or comparative genomic hybridization (CGH) to help identify genetically abnormal embryos and improve healthy outcomes 10) embryo splitting - used for twinning to inc number of available embryos 11) gamete intrafallopian transfer - laparoscopy procedure to place eggs along with sperm in the fallopian tube to allow for fertilization in the body 12) NHS guidelines - women 40-42 offered 1 cycle of IVF on NHS if all of the following additional criteria are also met: never had IVF before, no evidence of low ovarian reserve & have been informed of the additional implications of IVF and pregnancy at this age

Monitoring of maternal wellbeing in labour

1) temperature 2) blood pressure 3) MHR - once hourly if low risk (to differentiate from FHR), every 15mins when contracting, otherwise when needed 4) pain levels

Classification of miscarriage

1) threatened - pain & bleeding, associated with closed cervical os & viable intrauterine pregnancy on USS 2) inevitable - pain & bleeding associated with open cervical os 3) missed - s/s often minimal, diagnosis on USS if no fetal heartbeat/fetal pole identified, or embryo >7mm but no fetal heart beat or no inc in size over 7 days 4) incomplete - evidence of retained products of conception (RPOC) on USS 5) complete - products of conception passed, cervical os closes, no evidence of RPOC on USS 6) miscarriage with sepsis - s/s similar to incomplete miscarriage with uterine + adnexal tenderness, vaginal loss may be purulent & pt may be pyrexial, renal failure, DIC etc 7) spontaneous 2nd trimester loss - pregnancy loss between 12-24 weeks with spontaneous rupture of membranes or cervical dilatation despite presence of fetal heart activity

Common abnormalities of the post partum period

1) thrombosis & thromboembolism - more likely after caesarian section, need effective tromboprophylaxis (clexane + ted stockings) 2) perineal pain/discomfort - worse after instrumental delivery, episiotomy, or spontaneous tears - local cooling, topical anaesthetic, NSAIDs 3) bladder dysfunction - a) voiding difficulties & retention after regional anesthetics, loss of bladder sensation after epidural or spinal can lead to overstretching of detrusor muscle = voiding difficulties b) difficult instrumental delivery or multiple & extended vulvovaginal lacerations can cause pain & periurethral oedema making voiding difficult c) after caesarian catheter left in for 12-24 hours to avoid overdistension d) if difficulty voiding send culture to exclude infection e) incontinence early in puerperium should be investigated in case of fistula 4) bowel - constipation is common (may be made worse by fear of defecation due to pain of prolapsed haemorrhoids, anal fissure, or sutured perineum) 5) postnatal depression & tiredness 6) breast problems - nipple pain, engorgement, cracks, bleeding, mastitis 7) secondary PPH - usually due to retained placental tissue, at 7-14 days 8) traumatic neuritis - foot-drop, paresthesia, hypoesthesia, sciatic pain, muscle wasting in lower limbs 9) diastasis of pubic symphysis - pain over symphysis which is exaggerated by weight bearing, waddling gait, occasionally a palpable interpubic gap

Diagnosis and implications of molar pregnancy and be able to describe the structure and function of the national register of trophoblastic disease

1) trophoblastic tissue (part of blastocysts that normally invades endometrium) proliferates in more aggressive way than normal 2) GTD affects 1 in 500-1000 pregnancies (all forms accounted for), and more common at extremes of reproductive age 3) diagnosis - hCG excreted in excess, may be detected on routine USS - 'snowstorm' appearance of swollen villi with complete moles, needs histology to confirm 4) hydatidiform mole - a) localised & non-invasive, premalignant condition b) can be complete or partial ole based on genetic & histopathological features c) complete mole - entirely parental in origin: sperm fertilises empty oocyte & undergoes mitosis, diploid tissue, 46XX. No foetal tissue, just proliferation of swollen chorionic villi d) partial mole - usually triploid from 2 sperms entering 1 oocyte. Variable evidence of a foetus 5) otherwise proliferation may be charismatic of malignant tissue - if only local within uterus = invasive mole, if metastasis = choriocarcinoma NB/ least common form of GTD is placental site trophoblastic tumour - presents 3-4 years later 6) presentation - a) exam - uterus large, early pre-eclampsia & hyperthyroidism may occur b) hx - vaginal bleeding usual & may be heavy, hyperemesis may occur 7) mx - trophoblastic tissue removed by suction curettage (ERPC) & diagnosis confirmed histologically, bleeding may be heavy a) thereafter serial blood or urine hCG levels taken - persistent or rising levels suggestive of malignancy b) if pts levels of hCG return to normal within 56 days of operating then pt asked to provide blood and urine samples for 6 months, if hCG levels take longer to return to normal pt followed up for 6 months from the date when tests returned to normal c) in UK women with molar pregnancy should be registered with supraregional centre (eg/ Sheffield) to guide mx & follow-up d) oral contraception after ERPC should be used for molar pregnancy hCG levels monitored - pregnancy avoided until completion of surveillance period 8) complications - a) recurrence of molar pregnancy - occurs 1 in 60 subsequent pregnancies. For every future pregnancy further hCG samples required to exclude disease b) gestational trophoblastic neoplasia as an invasive mole choriocarcinoma - follows 15% complete moles & 0.5% partial, however can also follow miscarriages & normal pregnancies. Diagnosis of malignancy made from persistently elevated or rising hCG, persistent bleeding or evidence of blood born metastasis, commonly to lungs. Tumour highly malignant but normally v sensitive to chemo. Low-risk pts given methotrexate + folic acid, high risk given combo chemo. 5-yr survival almost 100% 9) since 1973 UK has had a national system for registration, follow-up and treatment of GTD. The gynaecologist treating affected women will register them with one of the three centres: Ninewells Hospital (Dundee), Charing Cross Hospital (London) and Weston Park Hospital (Sheffield)

Investigation & management of genital prolapse in women

1) types of vaginal prolapse: urethrocele, cystocele, rectocele, enterocele, vault prolapse (after hysterectomy) 2) uterine prolapse: 1st degree - uterus within vagina; 2nd degree - cervix protrudes through introitus; 3rd degree - entire uterus comes outside vagina 3) diagnosis - a) predisposing factors - chronic cough, constipation b) s/s - mild may be asymptomatic, otherwise sensation of pressure, fullness or heaviness or bulge/protrusion or 'something coming down' (may be felt), spotting, urinary s/s eg/ incontinence, frequency, urgency, weak or prolonged urinary stream, dyspareunia, loss of vaginal sensation/arousal, vaginal flatus, change in body image, constipation/straining, urgency of stool, incontinence of flatus or stool, tenesmus c) speculum in L lateral position for type of prolapse d) bimanual exam to exclude pelvic masses e) stress incontinence - ask pt to cough with a full bladder f) MSU g) cystometry + uroflowmetry to differentiate detrusor instability & stress incontinence g) renal function checked when 3rd degree uterine prolapse 4) mx - tx not necessary if prolapse is mild and/or asymptomatic a) conservative (mild prolapse or want further children) - watchful wait, lifestyle - tx of cough, smoking cessation, constipation, dec BMI, pelvic floor muscle exercises, vaginal oestrogen creams b) vaginal pessary - provide support & relieve pressure on bladder & bowel c) surgery - restore anatomy, improve symptoms and return bowel, bladder and sexual function to normal, may use mesh repair I) bladder/urethral prolapse - anterior colporrhaphy or colposuspension II) uterine prolapse - hysterectomy, sacrohysteropexy, sacrospinous fixation III) vault prolapse - sacrospinous fixation, sacrocolpopexy IV) rectocele/enterocele - posterior colporrhaphy V) obliterative surgery (colpocleisis)

Risks and management of breech presentation

1) types: a) frank breech - legs lie extended along fetal trunk & flexed at hips & extended at knees, buttocks present at pelvic inlet b) flexed breech - legs flexed at hips & knees so both feet present at pelvic inlet c) knee or footling - one or both lower limbs flexed & breech of baby above maternal pelvis so feet descends through into vagina 2) risks - a) cord compression & cord prolapse (especially flexed or footling) b) entrapment of head behind cervix (especially preterm) - may cause asphyxiation = death or brain damage c) intracranial haemorrhage (skull doesn't have time to mould during delivery) d) trauma to viscera during delivery eg/ spleen or gut 3) management - common before 37 weeks (want longer gestation), external cephalic version, occasionally vaginal breech delivery, LCSC

Indications, uses and limitations of ultrasound, magnetic resonance imaging and computed tomography in the diagnosis and management of gynaecological malignancy

1) ultrasound - a) helps look for tumours in certain areas of the body that don't show up well on x-rays b) can be used to guide a needle during biopsy c) quick & don't require special preparation d) commonly used to monitor pregnant women and their unborn babies (no risk to foetus) e) sonograms made by high-frequency sound waves that go through the body. As the sound waves bounce off organs and tissues, they create echoes. The machine makes these echoes into real-time pictures that show organ structure and movement and even blood flow through blood vessels f) can distinguish fluid-filled cysts from solid tumours g) US images not as detailed as those from CT or MRI - cannot tell whether a tumour is cancer h) use is limited in some parts of the body because sound waves can't go through air (eg/ lungs) or through bone i) may need vaginal transducer (uncomfortable) 2) MRI - for some parts of the body and some types of tissues, it can produce clearer results than a CT scan a) can be used to: find a tumour, stage a cancer, assess for metastases b) may need a radioactive dye before the scan to help make the pictures clearer - possible SE: N+V, skin rash, headache, dizziness - rarely pt may have allergic reaction eg/ feeling weak, sweating and difficulty breathing c) in some early cancers eg/ cervical, MRI is better than CT at showing how deeply the tumour has grown into body tissues. It can be particularly useful for showing whether the tissue left behind after treatment is cancer or not d) cannot use if have metal in body eg/ artificial pacemaker, metal rods e) some people feel claustrophobic f) very safe and doesn't use radiation 3) CT - a) CT scan uses x-rays to take detailed pictures of the body from different angles. A computer then puts them together to give a series of pictures. CT (or CAT) stands for computerised (axial) tomography b) CT scanning is painless, noninvasive and accurate & can assess bone, soft tissue and blood vessels all at the same time in detail c) CT is less sensitive to patient movement & more cost effective than MRI d) CT can be performed if pt has an implanted medical device of any kind, unlike MRI e) provides real-time imaging so good for guiding minimally invasive procedures eg/ needle biopsies and needle aspirations of many areas of the body f) diagnosis determined by CT scanning may eliminate need for exploratory surgery and surgical biopsy g) pt may have an allergic reaction to contrast medium eg/ weakness, sweating and difficulty breathing h) exposure to radiation during a CT scan can slightly inc the risk of developing cancer in the future i) pregnant women should only have CT scans in emergencies

Relative legal status of fetus & mother

1) unborn children don't have separate legal recognition under the European or in the common law of England and Wales, or Scotland 2) women are free to make choices against medical advice & can't be forced to accept treatment or not if it's in the unborn child's interest 3) if healthcare providers feel a woman is putting her baby at risk they may make a referral to social care 4) abortion can only be carried out in approved institutions, after 2 doctors have agreed that at least 1 of the conditions in the abortion act relates to this case

Indications, methods and interpretation of the result of urodynamics, cystoscopy, ultrasound scanning and intravenous urography in the diagnosis and management of urogynaecological problems

1) urodynamics & cystometry - necessary prior to surgery for stress incontinence of for women whose OAB doesn't respond to medical therapy. Urodynamics - performed with or without video a) cystometry - directly measures, via a catheter, the pressure in the bladder (vesical pressure) whilst bladder is filled & provoked with coughing I) pressure transducer placed in rectum or vagina to measure abdo pressure II) true detrusor pressure calculated by subtracting abdo pressure from vesical pressure III) detrusor pressure doesn't alter normally with filling or provocation (inc intra-abdo pressure) IV) if leaking occurs with coughing, in the absence of detrusor contraction, then problem is likely 'urodynamic stress incontinence' V) if an involuntary detrusor contraction occurs = 'detrusor overactivity' VI) cystometry widely used to investigate symptoms of urinary incontinence 2) ultrasonography - excludes incomplete bladder emptying, check for congenital abnormalities, calculi & tumours, and detects cortical scarring of kidneys 3) AXR - diagnoses foreign bodies & calculi 4) CT urogram - with use of contrast, integrity & route of ureter examined 5) methylene dye test - blue dye instilled into bladder, leakage from places than than urethra ie/ fistulae, can be seen 6) cystoscopy - inspection of bladder cavity to exclude tumours, stones, fistulae, interstitial cystitis. Gives little indication of bladders performance

Issues relating to care of the perineum

1) vaginal soreness - if had episiotomy or vaginal tear during delivery, the wound might hurt for a few weeks. Extensive tears might take longer to heal a) if sitting is uncomfortable, sit on a pillow or padded ring b) use a squeeze bottle to pour warm water over the vulva while urinating c) press a clean pad or washcloth firmly against the wound when you bear down for a bowel movement d) cool the wound with an ice pack e) pain relievers f) stool softeners g) warm sitz baths (only hips & buttocks submerged) for 20 minutes a few times a day h) local anesthetics eg/ sprays, creams, ointments or pads i) tight clothing, esp underwear, can rub and irritate area and slow healing J) check for signs of infection - warmth, red, discharge, pain 2) vaginal discharge/lochia for a number of weeks after delivery. Bright red, heavy flow of blood for first few days, tapering off, becoming watery and changing from pink or brown to yellow or white 3) bowel movements - may avoid bowel movements out of fear of hurting perineum or episiotomy wound - high fiber, drink plenty of water, stool softener or osmotic laxative, if needed NB/ vaginal birth without tears: entire perineal area & rectum will be swollen after birth, discomfort for 3-5 weeks vaginal tear or an episiotomy: heals in 7-10 days, sore and sensitive for 6+ weeks

Diagnosis, management and implications of infections in pregnancy

1) varicella zoster - a) risk to mum = varicella pneumonia b) risk to fetus = congential varicella syndrome (eye defects, limb hypoplasia, neurological abnormalities) early in pregnancy, neonatal varicella near term (high mortality risk) c) management - non-immune pregnant woman exposed is give zoster Ig to dec risk of infection, if becomes infected given acyclovir to dec risk of maternal complications 2) parvovirus B19 (erythema infectiosum) - a) risk to mum = fever, rash, arthropathy b) risk to fetus = miscarriage earlier, fetal anaemia & heart failure (fetal hydrops) later c) management = simple analgesics + antipyretics, after 20 weeks use serial Doppler US to see blood flow in fetal middle cerebral artery - can give in utero blood transfusion if anaemia 3) influenza H1N1 - a) risk to mum = fever & cough, respiratory failure, secondary bacterial infection b) risk to fetus = preterm, in utero death, neonatal death c) management = antivirals eg/ oseltamivir, zanamivir, pregnant women immunized against H1N1 4) HIV - a) risks to mum = AIDS, dec immune system, inc risk of cancers b) risks to fetus = vertical transmission - transplacentally, vaginal birth, breastfeeding, risk of miscarriage, fetal growth restriction, prematurity, stillbirth c) management = woman managed by obstetrician + HIV physician + input from neonatal team, assessment of viral load + CD4 count, anti-HIV meds (HAART from 2nd trimester onwards), avoid breastfeeding, often have C-section (<400 copies/ml may have NVD), invasive procedures eg/ amniocentesis should be avoided, neonatal screening at birth & or 12 weeks, given antiretrovirals for PEP for several weeks

Causes & management of maternal collapse

1) vasovagal 2) massive haemorrhage 3) septic shock (strep A, B, D, pneumococcus, E.coli) 4) PE 5) epileptic fit 6) eclampsia 7) cerebrovascular accident 8) hypoglycaemia 9) amniotic fluid embolus - anaphylactic reaction to amniotic fluid/ particulate matter in lungs, collapse in labour/ shortly after delivery, central cyanosis, supportive measures - treat shock & coagulopathy 10) cardiac arrythmia/ arrest 11) uterine inversion (post delivery) - mismanagement of 3rd stage, rise in intra-abdominal pressure with relaxed uterus, fundal placenta with short umbilical cord, manage shock + replace uterus ASAP (manually, O'Sullivan's hydrostatic technique, laparotomy and Haultain's procedure) 12) drugs - illicit/ MgSO4/ Lignocaine 13) anaphylaxis

Reproductive surgery

1) wide range of operations designed for different purposes - most done to restore normal uterus, tubes and ovaries 2) endometriosis - surgery can be done to remove, or lessen the amount of abnormal tissue in pelvis 3) fibroid tumours - may be removed via laparotomy or, in select cases, outpatient hysteroscopy 4) pelvic adhesions - can cause major problems with fertility, can be removed during surgery but surgery can sometimes also cause adhesions 5) tubal fertility surgery - prior pelvic infection, surgery, or endometriosis can cause blocked fallopian tubes eg/ hydrosalpinx, laparoscopy is often used to diagnose & treat, but if severely damaged fallopian tubes must be removed to improve chances of contraception with IVF (fluid buildup in fallopian tubes can create a toxic environment for implantation) a) tubal surgery to inc fertility involves reversal of tubal sterilization or opening tubes blocked by endometriosis or infection at distal end next to ovary (hydrosalpinx), or proximal end from uterus (cornua) b) microsurgical tubal reversal, anastomosis and reanastomosis - repair or untying of a tube or both tubes using microsurgery, used after a sterilization procedure eg/ tying, cutting, clipping or burning tubes or after cornual damage to a tube by infection 6) ovarian cysts - large or persistent cysts or lasting several months may require surgery, usually laparoscopically, and do not require removal of the entire ovary 7) congenital structural abnormalities - during fetal development, abnormalities of the uterus, fallopian tubes and vagina may occur. Such malformations may contribute to infertility, severe pelvic pain, or recurrent pregnancy loss eg/ septate uterus, duplicate cervix, longitudinal vaginal septum may be treated with hysteroscopy and laparoscopy 8) acquired uterine abnormalities - uterine abnormalities can contribute to dec fertilization, miscarriage, or pregnancy complications eg/ polyps, scarring, and fibroids, treated with hysteroscopy and/or laparoscopy 9) male surgical treatments - a) testicular biopsy: several small pieces of testicular tissue removed & examined for sperm, which can be used in fertility procedures b) testicular sperm aspiration (TESA): needle biopsy of testicle in which a sample of tissue is taken directly from testis & used to extract sperm for IVF or ICSI c) percutaneous sperm aspiration (PESA): needle inserted into epididymis to locate & aspirate a pocket of sperm d) vasectomy reversal: done to reverse a previous vasectomy & restore male's ability to release sperm into semen from the testicles

Significance & management of menarche

1) women are born with all their ovarian follicles, which lie dormant until puberty arrives and rising hormones lead to the maturation of several ovarian follicles per month; usually only one matures and is released 2) menarche = start of 1st menstrual period, average age is 13, but can be as early as 8 & as late as 16 years and still be normal. Premature or delayed menarche should be investigated 3) how a woman chooses to deal with the physical blood loss is a matter of personal preference. Wide ranges of sizes and types of absorbent disposable towels and tampons are available 4) period pains respond well to anti-inflammatories eg/ ibuprofen 5) some women may need a combination of towels and tampons for overnight use, to prevent soiling bed linen 6) sometimes women may wish to postpone their cycle because of holidays, etc. This can be achieved by: Norethisterone 5 mg tds, tricycling the COCP; running packs together and omitting the pill-free week. This is safe for a maximum of three months, after which shedding of the endometrium should be allowed to occur. Some women may experience breakthrough bleeding if they tricycle the COCP

Sexual healthcare needs of vulnerable groups e.g. the young, commercial sex workers, drug abusers

1) young - a) all children and young people need good-quality sex and relationship education at home, at school and in the community b) need to know how to ask for help, & able to access confidential advice and support about wellbeing, relationships and sexual health c) need to understand consent, sexual consent and issues around abusive relationships d) all have rapid and easy access to appropriate sexual and reproductive health services e) access to free contraception 2) commercial sex workers - a) providing accurate, easily understood info about how HIV & STIs are transmitted & can be prevented b) teaching risk reduction methods and skills eg/ correct use of condoms, negotiation and communication skills; information about non-penetrative and lower risk sexual practices; what to do when condoms break; recognition of STI symptoms and where and how to access STI diagnosis and treatment c) ensuring access to voluntary counselling, testing and HIV care and treatment 3) drug abusers - a) regular contact with pts about tx for drug use gives opportunities to discuss contraception & sexual health b) many issues associated with drug use can impact on sexual health eg/ vulnerability & low self esteem, especially in women - difficult to negotiate satisfactory sexual relationships & safer sex c) risk of coercive and/or abusive relationships, including sexual abuse d) involvement in sex for payment - money, drugs, accommodation etc. e) important to discuss - safer sex, STIs, family planning, gynaecological health

Abortion Act 2003

1967 Abortion Act - made abortion legal when 2 doctors agree in good faith: 1) the pregnancy has not exceeded its twenty-fourth week and that the continuance of the pregnancy would involve risk, greater than if the pregnancy were terminated, of injury to the physical or mental health of the pregnant woman or any existing children of her family 2) or TOP is necessary to prevent grave permanent injury to the physical or mental health of the pregnant woman 3) or that continuance of the pregnancy would involve risk to the life of the pregnant woman, greater than if the pregnancy were terminated 4) or there's a substantial risk that if the child were born it would suffer from such physical or mental abnormalities as to be seriously handicapped NB/ does not apply to Northern Ireland, where the abortion law remains governed by the Bourne Decision

Causes of bleeding and/or pain in early pregnancy

25% pregnancies have vaginal bleeding before 20 weeks can be presenting symptom of something life-threatening eg/ ectopic pregnancy - bleeding should always be considered abnormal & investigated small amount may happen when blastocyst implants in endometrium 5-7 days after fertilization (implantation bleed) - can be confused with period (calculations of gestational age based on LMP)

Detailed knowledge & understanding of emergency contraception

3 types of emergency contraception available 1) IUD - copper coil: can be inserted up to 5 days after sex, most effective method, lasts for 5-10 years 2) UPA/ella-one pill: can be inserted up to 5 days after sex, progesterone receptor antagonist so can't use if used any progesterone method of contraception in past 5 days, and can't start these methods for a few more days, 2nd most effective method, efficacy dec nearer to ovulation 3) levonorgestrel pill eg/ Plan B, One Step: can take up to 3 days after sex, efficacy dec nearer to ovulation NB/ don't use 2 different kinds of morning-after pills (eg/ Plan B and ella) at same time or within 5 days, as may counteract each other and not work at all NB2/ don't take 2+ doses of either type of medication as can cause sickness

Antenatal causes of maternal mortality and morbidity

5 most important direct causes of maternal mortality world-wide: haemorrhage, sepsis, unsafe abortion, eclampsia, and obstructed labour 1) haemorrhage - primarily PPH, causes 25% all maternal deaths. If uncontrolled, haemorrhage can quickly lead to shock and death within 7 days of childbirth. Immediate PPH is most commonly due to uterine atony, inadequate contraction of uterus, and retained placenta or placental fragments 2) infections - a) sepsis = 2nd leading cause of maternal mortality, ~15% all maternal deaths. Risk of puerperal sepsis inc for women with sexually transmitted and other infections, premature rupture of membranes, retained products of conception, diabetes, caesarean or other operation, PPH, anaemia, poor nutritional status, history of previous complications of labour, and poor infection control. Most common sign of puerperal infection is fever b) malaria - women more susceptible to malaria infection during pregnancy - also at inc risk for high parasitemias and anaemia c) viral hepatitis - most common cause of liver disease during pregnancy 3) unsafe abortion - complications eg/ sepsis, haemorrhage, genital and abdominal trauma, tetanus, perforated uterus, and poisoning from meds 4) hypertensive disease of pregnancy - causes 12% all maternal deaths 5) obstructed labour - injuries to multiple organ systems eg/ vesico-vaginal or recto-vaginal fistulae, inc risk of sepsis, haemorrhage, and uterine rupture 6) thrombosis & thromboembolism = leading cause of direct deaths in UK 7) direct causes of death in UK (from most common downwards): thrombosis & thromboembolism, APH & PPH, amniotic fluid embolism, genital tract sepsis, early pregnancy/ectopic, pre-eclampsia/eclampsia, anaesthesia indirect causes of death in UK (from most common downwards): cardiac, indirect sepsis (influenza, pneumonia), indirect neurological conditions (epilepsy, psych)

Structure and reasons for postnatal review

6 week postnatal review 1) history a) note how the baby was delivered b) ask if any particular worries about her health c) ask if perineum/caesarean section scar is healing well - any pain? d) is lochia normal and/or have periods resumed NB/ lochia usually ceased by 6 weeks postnatally, periods do not resume until breast-feeding ceases e) are bowel and bladder functioning normally? Any incontinence? f) breast-feeding? Any problems eg/ soreness or engorgement? 2) psychological problems a) how was birth? Any issues to be talked through? b) how is her mood? Screen for postnatal depression eg/ the Edinburgh Postnatal Depression Score NB/ PN depression can be treated either pharmacologically or with psychological therapies eg/ CBT or interpersonal psychotherapy (IPT) c) any worries about the baby? 3) social problems a) well supported at home? b) sleeping? NB/ If this is a problem, consider how she might gain support from a partner or family c) encourage household smokers to quit (passive smoking = inc risk of SIDS & asthma) d) provide opportunity to talk without her partner - explore issues such as domestic violence 4) exam a) palpate the abdomen - if able to feel uterus, consider retained products of conception or endometritis, if tender b) BP c) VE if had - problems with vaginal tears or episiotomy, abnormal bleeding or vaginal discharge, dyspareunia, urinary or faecal incontinence d) if smears required, delayed until 3 months post-delivery e) weight f) also consider - Hb if previously anaemic, rubella status, GTT for women who developed gestational diabetes 5) sex and contraception a) has intercourse resumed with her partner. If not, reassure her that it is now safe to try b) enquire if contraception meeded - full-time breast-feeding provides good contraception for up to 6 months if she remains amenorrhoeic, otherwise use: condoms, IUD, POP & implants, levonorgestrel-releasing intrauterine system 6) pelvic floor exercises - many incontinence problems begin during antenatal period a) slow contractions: pulling up pelvic floor muscles as though trying to stop herself urinating or passing wind, and holding for 10 seconds - repeat 10x, 3x a day b) quick contractions: contracting pelvic muscles, as before, and relaxing rapidly in succession 10x, 3x a day

Management of urinary tract infection in pregnancy

>100,000 bacteria per ml urine eg/ E. coli, Klebsiella Proteus - treat asymptomatic bacteriuria in women 1) treatment in order of preferance a) amoxicillin: for 1/52 b) nitrofurantoin: for 1/52 (don't give in 3rd trimester) c) trimethoprim: for 1/52 I) folic acid 5 mg daily if 1st trimester II) no trimethoprim if woman is folate deficient, taking a folate antagonist, or treated with trimethoprim in the past year d) cefalexin: for 1/52

Understand in detail the principles underlying semen analysis

Aka "seminogram" - evaluates certain characteristics of a male's semen & sperm eg/ sperm count, motility, morphology, volume, fructose level and pH 1) sperm count - measures concentration of sperm in a man's ejaculate, distinguished from total sperm count, which is sperm count multiplied with volume, > 15 million sperm/ml normal a) Lower sperm count = oligozoospermia, no sperm = azoospermic 2) motility - 50% should be motile, measured within 60mins of collection, parameter of vitality - lower reference limit of 60% live spermatozoa a) measured in motility grades - grade a = best, grade d = immobile NB/ if sperm count is very high, then a low motility (< 60%) might not matter 3) morphology - sample is normal if >4% (or 5th centile) sperm have normal morphology 4) volume - 1.5 ml is lowest semen volume considered normal, low volume may indicate partial or complete blockage of seminal vesicles, or lack of seminal vesicles 5) color - normally whitish-gray color, yellowish tint with age. Deep yellow or green may be due to meds. Brown = infection and inflammation of prostate, urethra, epididymis and seminal vesicles. Also - STIs eg/ gonorrhea and chlamydia, genital surgery and injury to the male sex organs 6) fructose level - amount of energy available to semen for moving, normal = 13 μmol per sample 7) pH - normal = 7.2-7.8, acidic ejaculate means seminal vesicles are blocked, basic ejaculate may indicate infection 8) liquefaction - process when the gel formed by proteins from the seminal vesicles is broken up & semen becomes more liquid. Normally <20 mins, but <60 mins is normal according to NICE 9) MOT - how many million sperm cells per ml are highly motile (grade a + b) - combo of sperm count & motility, vial volume of 0.5 ml should have 20 million motile spermatozoa (for implantation) 10) abnormalities - aspermia: absence of semen, azoospermia: absence of sperm, asthenozoospermia: poor sperm motility, teratozoospermia: more morphological defects than usual, necrozoospermia: all sperm in ejaculate are dead, leucospermia: high WCC

Perform pregnancy dating and calculation of gestational age

At 8-14 weeks of pregnancy all women offered dating scan (USS) - measure crown rump length (CRL). On average: at 10 weeks, baby measures about 3cm, at 12 weeks between 5-6cm, at 13 weeks, 7cm. After 13 weeks use head circumference Calculation of gestational age & due date - LMP + 10 days - 3 months (+1yr)

Principles of safe prescribing in pregnancy + risks & modifications required to continue drug treatment during pregnancy

Can harm foetus through teratogenic effects - malformations of organs, growth restriction, fetal death, carcinogenesis, also some drugs are retinoids so cause damage for 2 yrs after last dose Timing of drugs is important. 3 important phases in human development: 1) pre-embryonic phase - conception to 17 days, implantation & blastocyst formation - drugs are all or nothing (death or survival) 2) embryonic phase - day 18 to 55, most crucial period of organogenesis & damage is irreparable eg/ anencephaly = day 24, cleft lip = day 36 3) fetal phase - 8 weeks to term, fetal growth & development affected by drugs that can cross placenta principles of prescribing: 1) drugs only for clear indications where risks do not outweigh benefits 2) try & avoid all drugs in 1st trimester 3) medication should be in smallest effective dose for smallest time 4) all women of fertile age should be considered pregnant when prescribing 5) use drugs where the effects on fetus are well known 6) most drugs with a molecular weight <1500 can cross the placenta 7) pre-conception counselling for those with chronic medical conditions, especially those on medications - avoid polypharmacy as far as possible

Operative vaginal deliveries; indications, methods and complications

Forceps 1) pair of fenestrated blades connected to handle with a shank. Designed with cephalic curve that fits around fetal head & pelvic curve that fits the pelvis. Mainly used for traction. Kielland's forceps have dec pelvic curve so suitable for rotation of fetal head in malpresentation 2) when to use: a) abdo palpation: fetus should be not too large (suggestive of possible disproportion); head should be 1/5th or less palpable above pelvic brim; good uterine contractions; empty bladder b) VE: cervix fully dilated with ruptured membranes; vertex presentation with no excess caput or moulding; position of vertex known (L occipitoanterior/transverse etc); station <1; descent of head with contraction & bearing down effort NB/ analgesia - local eg/ pudendal block, perineal infiltration; existing epidural block; spinal anesthesia 3) indications - a) delay in 2nd stage due to poor maternal expulsive effort, malposition, or minor disproportion. May be related to lack of urge to bear down (epidural) or inadequate uterine activity (lack of Ferguson reflex) b) fetal distress; or prolapsed cord; or eclampsia in 2nd stage of labour c) to prevent undue maternal effort in women with cardiac & respiratory diseases & severe pre-eclampsia d) may be applied to head if breeched & to remove during caesarian e) assist face presentation in mento-anterior position 4) complications - a) maternal trauma - labial & vaginal & perineal tears, rarely 3rd and 4th degree tears b) fetal trauma - face bruising, VII nerve paralysis (usually resolves), skin abrasions, rarely cephalhaematomas & fracture of skull c) risk of blood clots; urinary & anal incontinence ventouse 1) application of suction cup to fetal scalp & extraction by traction. Associated with less maternal trauma a) indications - often used before forceps unless: baby is <34 weeks; face presentation; breech; bleeding disorders b) complications - cephalhaematomas, neonatal jaundice, & retinal haemorrhage more common; scalp forms chignon (normally resolves within 2 days); risk of blood clots; urinary & anal incontinence

Confidentiality and consent in under 16 year olds (Fraser competence) and vulnerable adults

Fraser guidelines - apply specifically to contraceptive advice. Doctor can give advice & tx if satisfied in the following criteria: 1) the girl (although <16 years old) will understand his advice 2) you cannot persuade her to inform her parents or allow you to inform the parents she is seeking contraceptive advice 3) she is very likely to continue having sexual intercourse +/- contraception 4) unless she receives contraceptive advice or treatment her physical or mental health or both are likely to suffer 5) her best interests require you to give her contraceptive advice, treatment or both without the parental consent

Transmission, diagnosis, management and prevention of HIV

HIV 1) transmission - sex, IVDU, blood, , vertical, perinatally, breast milk 2) diagnosis - a) seroconversion illness (1-6 weeks after infection): glandular fever-type illness - fever, malaise, myalgia, pharyngitis +/- maculopapular rash, consider HIV seroconversion illness if unusual signs eg/ oral candidiasis, recurrent shingles, leukopenia, or CNS signs b) symptomatic infection - fever, night sweats, diarrhoea, weight loss. May also be minor opportunistic infections eg/ oral candida, oral hairy leukoplakia, herpes zoster, recurrent herpes simplex, seborrhoeic dermatitis, tinea infections c) AIDS - severe immunodeficiency. Evidence of life-threatening infections and unusual tumours d) ix - anti-HIV antibodies in serum (ELISA), acute infection detected by presence of p24 antigen or HIV RNA by PCR, assessment of viral load; detection of virus or viral antigen, FBC (anaemia, thrombocytopenia, lymphocytopenia, dec CD4) 3) mx - ART for all patients - 3 drugs: efavirenz + tenofovir or abacavir a) immunise against hep B, pneumococcal disease and Hib. Because of immunosuppression, not BCG vaccine, yellow fever, oral typhoid or live oral polio immunisations b) chemoprophylaxis used to prevent infection in HIV+ve patients 4) prevention - promote lifelong safer sex, barrier contraception and dec number of partners, dec sharing of needles & use needle exchange schemes. Vigorous control of other STIs. Dec unnecessary blood transfusions. Encourage pregnant women to have HIV tests. PEP after occupational and sexual exposure

Understand the significance of failure to conceive

Inability to get pregnant after 1 yr of regular intercourse. Can affect women and their partners emotionally, physically and mentally

Techniques for prevention and management of PPH & appropriate use of blood products

Primary PPH = loss of >500ml of blood from genital tract in first 24hrs after giving birth, secondary if occurs between 24hrs-6 weeks after delivery 4Ts - uterine Tone, Tissue (retained placenta), Trauma (lower genitourinary tract), Thrombin (coagulopathy) 1) prevention - oxytocin with or after delivery, cord traction, uterine massage after placenta 2) resuscitation - a) fluid replacement - 2 large IV 14 gauge bore cannuale with colloid (haemaccel) in one and crystalloid (Hartmann's, 0.9% saline) in other until blood available b) ix - FBC, cross-match 6 units blood, coagulation screen, estimation of fibrin degradation products, electrolytes, urea & creatine - repeated in 4 hrs c) monitoring - continuous oximetry, pulse & BP every 15mins, indwelling catheter (hourly measurements), CV pressure, arterial line if indicated 2) establishing a cause. If uterine atony - a) IV bolus of ergometrine (250ug) b) if bleeding continues give IV oxytocin (10 IU) & bimanual compression c) if continues give 15-methylprostaglandin F2a (Haemobate) 250ug IM into thigh/gluteal muscle - repeated up to 3x at 15min intervals d) if need to can give misoprostol rectally (800ug) e) if still bleeding need surgery 3) surgical options - a) hysterectomy & ligation of internal iliac arteries (last line) b) undersuturing placental bed if placenta praevia or low-lying placenta c) uterine compression sutures - B-Lynch suture & modification d) uterine artery ligation e) utero-ovarian artery anastomosis ligation f) arterial embolization 4) blood - DIC can compound problems causing PPH & must be corrected before PPH will stop. Blood products used in correction of deranged coagulation: 5) cryoprecipitate, 10-15 units (contains fibrinogen & factor VIII) 6) fresh frozen plasma, 5 units (contains fibrinogen and all clotting factors) 7) platelet concentrate, 5-6 units NB/ blood loss >1000 mL = massive hemorrhage - blood, platelets, ICU etc

Normal postpartum period

Puerperium is when body returns to pre-pregnant state with most physiological changes complete by 6 weeks 1) uterus involutes from around 1kg weight to 80g - after delivery uterine fundus lies just below umbilicus, but at 2 weeks postpartum can no longer be felt above symphysis. Involves autolysis of muscle cells with absorption of protein into circulation & excretion with urine a) delayed involution caused by - full bladder, loaded rectum, uterine infection, retained placental fragments or membranes, fibroids, broad ligament hematoma b) by end of 2nd week cervical internal os is closed & unable to admit a finger, vaginal walls regain ruggae by 3rd week c) if no lactation - uterine cavity covered by new endometrium by 3 weeks & 1st menstruation occurs by 6 weeks d) if lactation- ovarian activity suppressed, resumption of menstruation delayed for months e) during first few days, bladder & urethra may show evidence of minor trauma from delivery but this resolves rapidly. Within 2-3 weeks calyceal dilatation & hydoureter becomes less evident although complete return unlikely for 6-8 weeks 2) lochia - bloodstained uterine discharge (blood + necrotic superficial decidua). At the start is bright red, then turns pink, and serous by end of 2nd week a) if persistently red is associated with infection or retained products b) uterine tenderness + pyrexia + offensive lochia highly suggestive of uterine infection - immediate treatment with Abx, and evacuation of any retained products

Gamete donation

Refers to the donation of all gametes - ova or sperm 1) ova - for assisted reproduction purposes, egg donation typically involves IVF technology, with the eggs being fertilized in the laboratory; more rarely, unfertilized eggs may be frozen and stored for later use. Egg donation is a third party reproduction as part of assisted reproductive technology a) each egg donor is first referred to a psychologist who will evaluate if she is mentally prepared to undertake and complete the donation process, then a thorough medical examination, including a pelvic exam, blood draw to check hormone levels and to test for infectious diseases, Rh factor, blood type, and drugs and US to examine ovaries, uterus and other pelvic organs. A family history of approximately the past three generations is also required. Genetic testing to ensure they do not carry mutations (e.g., CF) that could harm the resulting children b) once the screening is complete and a legal contract signed, the donor will begin the donation cycle, which typically takes 3-6weeks. An egg retrieval procedure comprises both the Egg Donor's Cycle and the Recipient's Cycle. Birth control pills are administered during the first few weeks of the egg donation process to synchronize the donor's cycle with the recipient's, followed by a series of injections which halt the normal functioning of the donor's ovaries - may be self-administered on a daily basis for 1-3 weeks. Then FSH given to donor to stimulate egg production and inc no of mature eggs produced by ovaries. Throughout the cycle the donor is monitored often by a physician using blood tests and ultrasound exams to determine the donor's reaction to the hormones and the progress of follicle growth c) once the doctor decides the follicles are mature, he/she will establish the date and time for the egg retrieval procedure, 36 hours before retrieval, the donor must administer one last injection of HCG hormone to ensure eggs are ready to be harvested. The egg retrieval itself is a minimally invasive surgical procedure lasting 20-30 minutes, performed under sedation. A small US-guided needle is inserted through the vagina to aspirate the follicles in both ovaries, which extracts the eggs. Most donors resume regular activities by the next day d) after retrieval, the ova are fertilized by sperm in the laboratory, and, after several days, the best resulting embryo(s) is/are placed in the uterus of the recipient, whose uterine lining has been appropriately prepared for embryo transfer beforehand 2) sperm - principally used for artificial insemination of a female or females who are not their sexual partners. Laws regulating sperm donation address issues such as permissible reimbursement or payment to sperm donors, rights and responsibilities of the donor towards his biological offspring, the child's right to know his/her father's identity, and procedural issues

Structure and use of partograms

Shows cervical dilatation and descent of head - enables early recognition of labour that is non-progressive Sheet of paper shows progress in labour, can also add: frequency & duration of contractions, fetal HR, colour of liquor, caput & moulding, station or decent of head, maternal HR, BP & temperature Should be started on admission & started at 0 time

Issues relating to NHS funding and rationing of treatment relating to subfertility

The level of treatment offered in England is very much a 'postcode lottery' and is determined by the Clinical Commissioning Group (CCG) you come under. Although there is national guidance set NICE (2013) the CCGs can set their own local priorities and decide the level of funding, number of cycles they will fund and what additional criteria a patient has to meet in order to qualify for treatment. There are a few CCGs which do not fund fertility treatment at all

Importance of psychological factors in subfertility

The stress of the non-fulfilment of a wish for a child has been associated with emotional squeal such as anger, depression, anxiety, marital problems and feelings of worthlessness among the parents. In general, women show higher levels of distress than their male partners. Various research studies support the theory that distress is associated with lower pregnancy rates among women pursuing infertility treatment

Significance & management of menstrual problems

abnormalities in menstruation may include: 1) quantity eg/ menorrhagia - excessive menstrual blood loss interfering with QOL (>80 ml blood loss per menses), may cause iron-deficiency anaemia a) tx - hormonal - OCPs eg/ Natazia, Millinette, progestin therapy, levonorgestrel IUS, Gonadotropin-releasing hormone agonists, Danazol non-hormonal - tranexamic acid, mefenamic acid, NSAIDs b) surgery - organic cause eg/ fibroids, or medical therapy fails. Ranges from dilatation and curettage to hysterectomy 2) timing: polymenorrhoea - 2+ periods per calendar month, or oligomenorrhoea or amenorrhoea a) amenorrhea tx I) mullerian agenesis - partial or complete absence of upper vagina, cervix, or uterus - discussion of future fertility, nonsurgical and surgical creation of a neovagina (eg/ dilators) II) androgen insensitivity syndrome - complete gonadectomy after completed puberty to prevent gonadal neoplasia III) anatomic defects eg/ imperforate hymen or transverse vaginal septum - surgical procedure to create a patent outflow tract IV) primary hypogonadism - premature ovarian failure: hormone therapy V) functional hypothalamic amenorrhea - inc weight & dec exercise levels VI) PCOS - treated as in abnormal uterine bleeding VII) polymenorrhagia - tx cause eg/ stress, meds, use OCPs VIII) oligomenorrhea - not serious on its own, menstrual periods controlled by hormonal birth control or progestin. Sometimes indicates underlying problem eg/ eating disorder - need tx 3) duration of bleeding: normally 3-7 days 4) associated symptoms: dysmenorrhea, premenstrual syndrome a) dysmenorrhea - dec QOL: NSAIDs, opioid analgesics, oral contraceptives b) premenstrual syndrome - if mild or moderate change diet & lifestyle eg/ smaller meals more frequently, stop smoking, 2.5 hrs of exercise a week. If psychological symptoms - CBT etc. Medical tx - NSAIDs, COC eg/ Yasmin pill, SSRIs, gonadotrophin-releasing hormone (GnRH) analogues 5) abnormal uterine bleeding: a) PCOS - if want to conceive, clomiphene 1st-line, then gonadotropins. If not trying to conceive give COCP b) hirsutism - COCPs & antiandrogens (eg, spironolactone) c) coagulopathy - antifibrinolytics, COCPs, progestin-only contraceptives d) IMB - low-dose COCP, NSAIDs with bleeding 6) non-reproductive causes a) normal menstruation affected by any failure of clotting system eg/ von Willebrand's disease, leukaemia, ITP and hypersplenism b) hypothyroidism - associated with menorrhagia or IMB c) cirrhosis - associated with dec ability of the liver to metabolise oestrogens, and hypoprothrombinaemia 7) diseases of reproductive tract: a) most common causes at fertile age are related to pregnancy eg/ threatened, incomplete or missed abortion, ectopic, trophoblastic disease b) malignancies - endometrial, cervical and ovarian carcinoma c) endometritis - usually presents as intermenstrual spotting d) fibroids, endometrial polyps and adenomyosis e) cervical lesions - erosions, polyps and cervicitis - can present as postcoital spotting

Detailed knowledge and understanding of LARC

all release progesterone - stops ovulation, thicken cervical mucous to prevent sperm reaching egg, thins lining of uterus to prevent implantation, for all don't have to remember to do anything 1) contraceptive injection - <4 woman per 1000 get pregnant over 2 yrs a) advantages - lasts for 8-13 weeks, can use if breastfeeding, not affected by meds, diarrhea, vomiting b) disadvantages - periods may stop, be irregular or last longer, fertility takes time to return upon stopping, some gain weight, injection can't be removed from body so SE cannot be stopped 2) implant - <1 woman per 1000 get pregnant over 3 yrs a) small flexible rod put under skin of upper arm b) advantages - works for 3 yrs but can take out sooner, fertility returns to normal on removal c) disadvantages - periods may stop, be irregular or last longer, acne, small procedure to fit & remove - tenderness, bruising & swelling, some meds interfere 3) IUS - jaydess and mirena, <1 woman per 1000 gets pregnant over 5 yrs a) small T shaped plastic device in uterus b) advantages - mirena works for 5 yrs, jaydess for 3 yrs, both can be taken out sooner, with mirena periods normally lighter, shorter, sometimes less painful, when removed fertility returns to normal, if fitted after 45 can stay until menopause, can be helpful if heavy/painful periods, not affected by meds c) disadvantages - irregular bleeding or spotting common for 6 months, periods may stop, very small chance of infection in 20 days after insertion, ovarian cysts, insertion may be uncomfortable 4) IUD - <2 women per 1000 get pregnant over 5 yrs a) small plastic & copper device into uterus b) advantages - works as soon as put in, can stay in 5-10 yrs, can be taken out any time, fertility returns to normal, if fitted after 40 can stay in until menopause, check for STI, not affected by meds c) disadvantages - may not be suitable for women at risk of STIs, periods may be heavier or longer & more painful, very small chance of infection in 20 days, insertion can be uncomfortable

Risks and management of prolonged pregnancy

any pregnancy >294 days from the 1st day of LMP 1) risks - a) postmaturity in infant - dry, peeling & cracked skin (particularly hands & feet), absence of vernix caseosa & lanugo, loss of subcutaneous fat I) complications - inc risk of IU aspiration of meconium (2% at 41 weeks, 5% at 42 weeks), unexpected stillbirth, intrapartum fetal distress, inc operative delivery rate 2) management - women between 40-41 weeks have CTG + amniotic fluid index assessment, AFI <5cm offered induction, otherwise induction at 41+5, labour - cervical preparation with prostaglandins or mechanical methods, high-risk pregnancies - may need C-section

Detailed knowledge and understanding of emergency investigation and management of Bartholin's abscess/cyst

bartholin glands are paired glands ~0.5 cm in diameter in labia minora in 4- and 8-o'clock positions. Typically nonpalpable. Each gland secretes mucus into a 2.5 cm duct which emerge onto the vestibule at either side of the vaginal orifice, inferior to the hyme - maintain moisture of the vaginal mucosa's vestibular surface 1) ix - if emergency need FBC, U&Es, LFTs, (STD testing - rare cause) 2) mx - a) if uncomplicated, asymptomatic cyst may be discharged with sitz bath instructions b) generally painful & usually requires incision + drainage with anesthesia c) marsupialization - excision of Bartholin gland & surrounding tissue. Disfiguring, painful, and seldom indicated for abscess, may be used for malignancy

Transmission, diagnosis, management and prevention of chlamydia

chlamydia trachomatis 1) transmission - sex, secretions, or perinatally 2) diagnosis - mostly asymptomatic & detected during screening a) s/s in women - mucopurulent vaginal discharge, dysuria, lower abdo pain, fever, IMB or postcoital bleeding (inflamed cervix), pelvic adnexal tenderness on bimanual palpation, cervical excitation b) s/s in men - urethritis with dysuria & mucopurulent urethral discharge or epididymo-orchitis - unilateral testicular pain ± swelling, fever c) both - young adults with a reactive arthritis (Reiter's syndrome), upper abdo pain (Fitz-Hugh Curtis syndrome) d) samples taken for NAATs - vulvovaginal swab (women), first catch urine (men) 3) mx - abx, screening for other STIs, partner notification a) doxycycline for 1/52 (CI in pregnancy); OR single dose azithromycin pregnant or breast feeding - azithromycin stat, erythromycin for 1/52 b) avoidance of sexual intercourse even with a condom for 1 week after abx c) test for other STIs eg/ HIV and hepatitis B 4) prevention - promote safer sex, encourage early healthcare-seeking a) chlamydial screening - opportunistic screening if sexually active & <25

Detailed understanding of the epidemiology & normal symptoms and signs of early pregnancy

early s/s 2) late period 3) sore breasts +/- inc in size of breasts 4) weird tastes in the mouth eg/ metallic taste (dysgeusia) 5) slight bleeding or cramping as the embryo implants 6) extreme tiredness +/- backache 7) N+V can start very early for some women - a common early pregnancy symptom will be morning sickness. This will usually start at six weeks pregnant 8) inc urinary frequency 9) headaches 10) darkening of areola +/- nipples become erect & bumps around nipples more pronounced 11) food cravings or aversions 12) bloated abdomen + /- stomach cramps 13) thrush 14) trapped wind +/- constipation or diarrhoea later s/s 1) weight gain of 10-15 kg (uterus inc from 50-1000g) 2) effacement of cervix in late 3rd trimester 3) itching - common, sclerae checked for jaundice & LFTs & bile acids assessed 4) pelvic girdle pain (symphysis pubis dysfunction) - common, causes varying degrees of discomfort in pubic + sacroiliac joints a) tx - physio, corsets, analgesics & crutches may be used. Usually cured after delivery 5) abdo pain - usually benign & unexplained, medical & surgical problems can also happen though in pregnancy eg/ UTIs & fibroids 6) heartburn - affects 70% & marked in supine position a) tx - extra pillows, antacids, ranitidine in severe cases. Pre-eclampsia can present with back pain 7) backache - may cause sciatica a) most cases resolve after delivery, physio, advice on posture & lifting, form mattress, corsets may help 8) constipation - exacerbated by oral iron a) helped by high fibre, stool softeners 9) ankle oedema - worsens at end of pregnancy, unreliable sign of pre-eclampsia - sudden inc warrants assessment & follow-up of BP & urinalysis a) helped by raising feet at night, diuretics should not be given 10) leg cramps - affect 30% women a) tx unproven - can give sodium chloride tablets, calcium salts or quinine 11) carpal tunnel syndrome - due to fluid retention compressing median nerve, seldom severe & usually temporary, wrist splints may help 12) vaginitis - due to candidiasis a) give imidazole vaginal pessaries (eg/ clotrimazole) 13) tiredness - often incorrectly attributed to anaemia

Describe in detail the commonly encountered complications of surgery in obstetrics and gynaecology and their prevention

extensive gynecologic surgery often entails dissection near bladder, rectum, ureters, and great vessels of the pelvis 1) haemorrhage 2) infection - a) prevention - Abx prophylaxis used for major abdo or vaginal surgery 3) thromboembolism - a) prevention - COC stopped 4 weeks before major abdo surgery otherwise LMWH used, early mobilisation, thromboembolic disease stockings (TEDs), adequate hydration, LMWH for high risk pts eg/ cancer, prolonged surgery, history of DVT/PE 4) visceral damage a) uterine perforation - potential complication of all intrauterine procedures and may be associated with injury to surrounding blood vessels or viscera (bladder, bowel). Uterine perforations & associated complications not diagnosed at the time of the procedure can result in haemorrhage or sepsis. The risk of uterine perforation is increased by factors that make access to the endometrial cavity difficult (eg/ cervical stenosis) or alter the strength of the myometrial wall (eg/ pregnancy, menopause) b) urinary tract injuries - rates of ureteric & bladder injury vary with the indication for surgery & the procedure. Risk factors for urinary tract injury include surgery for malignant disease and for urinary incontinence or pelvic organ prolapse NB/ most women catheterised before major surgery, indwelling (Foley) catheter left in situ overnight for major abdo or vaginal surgery 5) risk of complications depends upon extent & approach to surgery and patient characteristics. More common complications relate to injuries to viscera & occur during extensive resections for tx of cancer or when anatomy is distorted due to infection or endometriosis 6) other complications eg/ PE, MI, pneumonia, pain, or fluid or electrolyte imbalance are common to all surgery

Transmission, diagnosis, management and prevention of genital herpes

genital herpes (Herpes simplex virus) 1) transmission - type 1 = oral infections (cold sores), type 2 = anogenital infections NB/ both can infect mouth and/or genitals, as oral sex or auto-inoculation 2) diagnosis - many asymptomatic, otherwise multiple painful ulcers a) primary infection - asymptomatic, or febrile flu-like prodrome (5-7 days of myalgia and fever), tingling neuropathic pain in genital area/buttocks/legs, extensive painful ulcers in genital area - lesions bilateral in primary disease & unilateral if recurrent, bilateral tender inguinal LN, dysuria b) recurrent infection - after primary, virus becomes latent in local sensory ganglia near skin, periodic reactivation (virus moves from ganglia to skin) = lesions (symptomatic shedding) or no visible lesions (asymptomatic shedding) c) ix - viral culture, DNA detection using PCR of a swab from the base of an ulcer, type-specific serology tests to identify if asymptomatic & distinguish HSV1 and 2 3) mx - no cure for genital HSV, lifelong infection a) supportive - saline bathing, oral painkillers, topical lidocaine 5% gel or ointment, vaseline applied to prevent pain during micturition, inc fluid intake to dilute urine to reduce pain during micturition b) antivirals - within 5 days of onset or if new lesions are still forming - aciclovir c) suppressive therapy - episodic antiviral treatment to dec duration & severity of episodes of genital herpes, if >6 attacks per year d) counselling - support from Herpes Viruses Association natural history - can get genital herpes even if partner has never shown any sign of infection (latent phase) - 1st episode does not necessarily imply recent infection. (Relationship issues need to be addressed) 4) prevention - no vaccine, dec transmission by: dec number of sexual partners, use of condoms, avoidance of sex with someone who has active genital herpes or active oral herpes, antiviral may dec transmission to partners

Transmission, diagnosis, management and prevention of HPV

genital warts (Human papillomavirus) 1) transmission - sexually, prenatally, auto-inoculation or hetero-inoculation 2) diagnosis - s/s - lesion(s): painless, itchy, urethral/vaginal discharge, pelvic or scrotal pain, IMB or postcoital bleeding, dyspareunia a) often multiple warts on moist, non-hairy skin usually soft and non-keratinised, if on dry hairy skin more likely to be firm and keratinised b) warts pigmented or not (pink, red, white, grey or brown), often on areas subject to trauma during sex c) diagnosis by biopsy (viral typing if diagnosis uncertain, for recalcitrant warts, warts with atypical features or high risk of HPV-related malignancy) 3) mx - condoms until resolution of lesions, can recur, STI screening, partner notification a) no tx - 1/3 regress spontaneously within 6 months b) podophyllotoxin self-application for soft, non-keratinised external genital warts - 2x-daily for 3 days, then 4 rest days. Repeat for 5 weeks c) imiquimod 5% cream self-application for keratinised lesions & non-keratinised external lesions - 3x a week at night until lesions resolve, use <16 weeks d) trichloracetic acid - warts that are very indurated or in pregnant patients e) ablation, cryotherapy, excision, electrocautery or laser can be used 4) prevention - condoms & vaccine a) vaccine - Cervarix protects against HPV16 & HPV18 (cervical cancer). Gardasil protects against HPV16, 18, 6, 11 - also protects against genital warts

Transmission, diagnosis, management and prevention of gonorrhoea

gonorrhoea (Neisseria gonorrhoeae) 1) transmission - infected secretions, sexually or perinatally 2) diagnosis - symptomatic in 90% men & 50% women a) s/s in men - urethral infection (purulent discharge and/or dysuria), rectal infection or perianal/anal pain, pruritus or bleeding b) s/s in women - inc or altered vaginal discharge, lower abdo pain, urethral infection (dysuria), rectal infection, easily induced contact bleeding of endocervix c) culture if s/s consistent with gonorrhoea or a positive NAAT result d) NAAT from genital samples - invasive (eg, urethral, endocervical) and non-invasive (eg, first pass urine) 3) mx - referral to GUM clinic, emergency medical admission if evidence of disseminated gonorrhoea or severe PID a) advise on unprotected sex - avoid until pt & partner(s) have completed treatment b) advise routine screening for other STIs c) partner notification (last 3 months - partners should receive a full STI screen & receive empirical treatment for gonorrhoea and chlamydia in advance of results d) ceftriaxone 500 mg IM stat + azithromycin 1 g orally stat e) a test of cure (with culture >72 hours or with NAAT >2 weeks following Abx) f) abx resistance to ceftriaxone - cefixime, azithromycin 4) prevention - promotion of safer sex methods, consistent use of condoms, testing those sexually active & at risk of acquiring gonorrhoea - high-risk groups (inner-city residents, GUM attendees, military personnel, prisoners and MSM), partner(s) notified quickly & treated

In normal pregnancy, to ask about past obstetric history, including mode and gestation at delivery

gravidity = number of times woman has been pregnant regardless of outcome eg/ primigravida = pregnant for first time, multigravida = pregnant 2+ times parity = number of times woman has given birth after 24 weeks or with a birth weight >500g (live or stillbirth) eg/ nulliparous = never given birth after 24 weeks, primipara = 1st birth after 24 weeks, multiparous = given birth after 24 weeks 2+ times, grand multiparous = given birth 5+ times parturient = in labour, puerpera = woman who's given birth in last 42 days need to record all previous pregnancies (G+P), duration of gestation (especially if history of preterm), antenatal complications, details of induction & duration & presentation & method of delivery at labour, birth weight & sex of each infant condition of each child at birth, need for ICU, complications eg/ PPH, infections of genital & urinary tract, DVT, perineal trauma

Detailed knowledge of the diagnosis, management and implications of antepartum haemorrhage

haemorrhage from vagina after 24th week of gestation, 50% unknown cause, 30% placenta praevia, 20% placental abruption, vasa praevia rare 1) placenta praevia - all or part of placenta implants in lower uterine segment so lies beside or in front of presenting part, grades I + II = NVD, III + IV = C/S a) diagnosis - I) s/s - sudden painless & repeated vaginal bleeding, malpresentation, if labour starts & cervix dilates can have profuse haemorrhage II) abdo exam - displacement of presenting part (vaginal exam after US), normal uterine tone (unlike abruption) III) USS - transabdominal (or transvaginal if placenta is posterior) shows location of placenta, but diagnosis can't be made before ~28 weeks IV) MRI - when placenta praevia is over old C-section scar to determine if tissue has migrated through scar (placenta percreta) or into bladder tissue b) mx - hospital admission until fetal maturity then often C/S, by 38 weeks FBC & cross match, CTG for fetal status, US to find cause, IV line inserted, anti-D Ig if needed, only do vaginal exam if all other ix inconclusive, blood loss treated by oral iron, catheter c) implications - with any APH suspect placenta praevia, risk of massive haemorrhage or spontaneous vaginal delivery by vaginal exam, risk of PPH 2) abruptio placenta - haemorrhage from premature separation of placenta, concealed, revealed or mixed a) s/s - pain, variable vaginal bleeding, inc uterine activity ('hard'), longitudinal lie b) mx - hospital admission, US to diagnose, lengthen gestation if possible, Hb/haematocrit, IV saline, blood transfused, clotting profile, catheter, 38 weeks induce labour, section if fetal distress c) implications - I) afibrogenaemia - clot from severe abruption causes thromboplastin release into maternal circulation = DIC - abnormal bleeding at operative delivery or uncontrolled PPH (hypovolemia) II) Couvelaire uterus = haemorrhage through uterus wall III) renal tubular or cortical necrosis = complication of DIC + hypovolaemia 3) other causes - a) vasa praevia - branch of fetal umbilical vessel lies in membranes & across cervical os - rupture of membranes = tear in vessels & exsanguination of foetus b) umbilical cord, cervical polyp, vaginitis, carcinoma of cervix, ruptured uterus

Transmission, diagnosis, management and prevention of hepatitis B

hepatitis B 1) transmission - blood, sex, IVDU, vertical transmission 2) diagnosis - flu-like illness: jaundice, anorexia + nausea + ache in RUQ, malaise, progressive darkening of urine & lightening of faeces a) ix related to hep B - HBsAg, HBeAg, anti-HBe/s, quantitative hepatitis B virus DNA, HBV genotype, hepatitis delta virus (HDV) serology b) ix for general liver function - FBC, bilirubin, liver enzymes, clotting, ferritin, lipid profile, autoantibody screen, caeruloplasmin 3) mx - avoid unprotected sex until non-infectious or partners fully vaccinated a) acute infection - tx supportive: fluids, antiemetics, rest, avoid alcohol, stop any non-essential meds b) chronic hep B & compensated liver disease - peginterferon alfa-2a, tenofovir c) pregnant or breast-feeding - tenofovir in 3rd trimester to dec risk of transmission of HBV to the baby d) prophylaxis during immunosuppressive therapy - entecavir or tenofovir 4) prevention - hepatitis B testing in asymptomatic pts - MSM, sex workers, IVDU, HIV+ve, sexual assault victims etc a) if non-immune - vaccine or passive immunisation with HBIG after exposure b) intimate contacts - partners should be notified, contact tracing c) pregnancy - all babies born to infected mothers should receive a complete course of vaccine, if highly infectious mother give HBIG + active immunisation

Understand in detail the principles underlying endocrine evaluation

infertility can be caused by problems with hormones 1) pituitary tumours can displace or destroy normal tissue & production of FSH + LH is often the first to be affected. Panhypopituitarism is also called Simmonds' disease 2) Sheehan's disease - pituitary infarction following postpartum haemorrhagic shock 3) hyperprolactinaemia - may present with galactorrhoea or amenorrhoea. Control of prolactin by inhibiting rather than a releasing factor from hypothalamus into hypothalamic-pituitary portal circulation. Hormone also released in response to thyrotropin-releasing factor, as is thyroid-stimulating hormone (TSH), and so it is elevated if thyroxine is low 4) pituitary gland may be responsible for other disorders such as Cushing's syndrome

Sensitive disposal of fetal remains

information given to couples if they want to hear it - cremation regulations don't apply to fetuses <24 weeks, but may be cremated at authorities discretion, no legal duty under burial legislation to bury babies born dead <24 weeks, communal burial permitted, option to bury at home, any baby who is born alive then dies is a live birth & should be treated as such

Genetic modes of inheritance

inheritance patterns for single gene disorders are classified based as autosomal or X-linked and dominant or recessive patterns of inheritance (Mendelian disorders) 1) autosomal dominant - only 1 copy of disease allele needed for an individual to be susceptible to expressing the phenotype, so with each pregnancy, there is a 50% chance the offspring will inherit the disease allele - all affected individuals have at least 1 affected parent a) eg/ myotonic muscular dystrophy and Huntington disease 2) autosomal recessive - 2 copies of disease allele are required for an individual to be susceptible to expressing the phenotype. Parents may not be affected but are gene carriers a) if parents are carriers: with each pregnancy, there's a 25% chance the offspring will inherit 2 copies of the disease allele & therefore have the phenotype, 50% chance of inheriting 1 copy & being a carrier. 25% chance of inheriting no copies of the disease allele b) eg/ sickle cell anemia and cystic fibrosis 3) X-linked dominant - 1 copy of disease allele on X chromosome is required for an individual to be susceptible to an X-linked dominant disease. Males more severely affected than females as they only carry 1 copy of genes on the X chromosome. Some X-linked dominant disorders are lethal in males a) when a female is affected, each pregnancy will have a 50% chance for offspring to inherit the disease allele. When a male is affected, all his daughters will be affected, but none of his sons will be affected b) eg/ hypophosphatemic rickets, oral-facial-digital syndrome type I, and Fragile X syndrome 4) X-linked recessive - 2 copies of disease allele on X chromosome are required for an individual with two X chromosomes (a female) to be affected with an X-linked recessive disease. Since males are hemizygous for X-linked genes, any male with 1 copy of an X-linked recessive disease allele is affected. Females usually carriers because only have 1 copy of disease allele. Affected males are related through carrier females a) for a carrier female, with each pregnancy: 50% chance sons will inherit the disease allele, 50% chance daughters will be carriers b) affected males transmit disease allele to all of daughters (carriers), but none to sons c) women affected when have 2 copies of disease allele. All sons will be affected, and all daughters will be unaffected carriers d) eg/ DMD, hemophilia A, hypohidrotic or anhidrotic ectodermal dysplasia 5) complex inheritance - caused by interactions of variations in multiple genes and environmental factors. Genes involved may make a person susceptible to the disorder, and environmental factors may trigger this susceptibility. Only individuals with enough genetic liability (multiple genes) in the presence of certain environmental factors will exhibit the phenotype a) eg/ heart disease, diabetes, asthma, many birth defects eg/ cleft lip +/- cleft palate

Principles of managed clinical networks with gynaecological oncology

involve clinical staff working across the boundaries between different professions and parts of the health service to ensure existing health service resources and staff are allowed to focus on what matters - patients and their problems. The direct involvement of patients in these networks ensures that the focus is not lost. The prime aim is really making sure that the right treatment gets to the right patient at the right time in the most appropriate place "linked groups of health professionals & organisations from primary, secondary and tertiary care, working in a co-ordinated manner, unconstrained by existing professional and Health Board boundaries, to ensure equitable provision of high quality clinically effective services". IT systems allow MDT to discuss individual cases without travelling so pts are referred & seen without delay pts guaranteed to receive specialist review regardless of geography & all clinicians involved in their care participate in establishing and reviewing their care plans speed of delivery of treatment plan has improved as all relevant information such as laboratory reports and pathology is recorded and collated through one central system

Treatment of cervical shock

normally due to incomplete miscarriage - products of conception stuck causes vaso-vagal effect due to products of conception passing through the cervix = reflex bradycardia remaining products of conception should be removed using a sponge-holding forceps, shock normally resolves spontaneously other causes of shock eg/ hypovolaemia may also occur and are associated with a tachycardia

Ultrasound screening of pelvic organs

organs & structures that are solid and uniform (eg/ uterus, ovaries, or prostate gland) or fluid-filled (eg/ bladder) show up clearly on a pelvic US. Bones may block other organs from being seen. Air-filled organs eg/ intestines, can make the image less clear 3 ways: transabdominal, transrectal, and transvaginal transabdominal - small handheld transducer passed back and forth over lower belly. Commonly done in women to look for large uterine fibroids or other problems. transrectal - transducer shaped to fit into rectum. Checks for problems in men or women. Most common test to look at =male pelvic organs eg/ prostate & seminal vesicles. Sometimes a biopsy may be taken transvaginal - transducer shaped to fit into a woman's vagina. Looks for problems with fertility or pregnancy. In rare cases, a hysterosonogram is done to look at the inside of the uterus by filling the uterus with fluid during a transvaginal ultrasound. Sometimes a biopsy may be taken fertility & women - to look at the size and shape of the uterus and the thickness of the uterine lining (endometrium), look at the size and shape of the ovaries, check the condition and size of the ovaries during treatment for infertility, guide a procedure to remove an ovarian follicle for IVF fertility & men - to look at the seminal vesicles & prostate gland

Issues relating to the diagnosis, investigation and management of tubal patency

pelvic infection & endometriosis can lead to fallopian tubes becoming blocked 1) laparoscopy & dye test - allows visualisation & assessment of fallopian tubes. Methylene blue dye injected through cervix from outside. Whether it enters or spills from tubes can be seen to assess if tubes patent. Hysteroscopy performed first to assess uterine cavity for abnormalities 2) hysterosalpingogram (HSG) - without anaesthetic radio-opaque contrast injected through cervix. Spillage from fimbrial end (& filling defects) see on x-ray. A variant of this tests is performed using TVUS & US opaque liquid (HyCoSy) 3) these tests preferred in women with no risk factors for tubal disease & no s/s suggestive of endometriosis, as less invasive & safer than laparoscopy, however both endometriosis & periovarian adhesions may not be diagnosed unless they cause tubal damage (so no surgery may be performed that could help infertility) 4) mx - may be done by open abdominal surgery or using laparoscopy a) tubal reanastomosis - used to reverse a tubal ligation or to repair a portion of the fallopian tube damaged by disease. Blocked or diseased portion of tube is removed & 2 healthy ends of the tube are joined. This procedure usually is done by laparotomy b) salpingectomy - removal of part of fallopian tube, to improve IVF success when a tube has developed a buildup of fluid (hydrosalpinx). Hydrosalpinx makes it half as likely that an IVF procedure will succeed. Salpingectomy is preferred over salpingostomy c) salpingostomy - when end of the fallopian tube is blocked by a buildup of fluid (hydrosalpinx). This procedure creates a new opening in the part of the tube closest to the ovary, but it is common for scar tissue to regrow after a salpingostomy, reblocking the tube d) fimbrioplasty - may be done when the part of the tube closest to the ovary is partially blocked or has scar tissue, preventing normal egg pickup. This procedure rebuilds the fringed ends of the fallopian tube e) for a tubal blockage next to the uterus, a nonsurgical procedure called selective tubal cannulation is the first treatment of choice. Using fluoroscopy or hysteroscopy to guide the instruments, a doctor inserts a catheter, or cannula, through the cervix and the uterus and into the fallopian tube

Preconception counselling of women with pre-existing illness

pregnancy may make pre-existing illness worse. Pregnancy effect may be transient & return to pre-pregnancy state after delivery (eg/ T2DM). Deterioration may be permanent & progressive & eventually result in death (eg/ severe kidney impairment). When risk is high women may be advised not to attempt pregnancy - specialist advice should be sought. Pregnancy undertaken when illness is in remission, stable or cured will ensure better outcome eg/ good control of diabetes prevents congenital abnormalities in baby

Legal rights of and provisions for pregnant women

pregnant women should not be treated any differently, and any neccessary provisions must be made by their employer to allow them to continue working despite pregnancy eg/ not putting woman in environemnt that could be harmful for foetus entitled to: 1) free prescriptions and NHS dental treatment - free NHS dental & prescriptions (England) whilst pregnant & for a year after baby is born 2) paid time off for antenatal care (to allow you to go to antenatal appointments) - time off in addition to annual leave, includes: medical and midwife appointments, doctor-recommended appointments like relaxation or parenting classes. Baby's father or your partner are entitled to take unpaid time off work to go with you to 2 antenatal appointments 3) statutory maternity leave and pay - entitled to 1yr maternity leave & pay from employer for up to 39 weeks while on leave. To be eligible: average earnings of at least £113 per week, and been working for employer at least 26 weeks. First 6 weeks = 90% average weekly earnings before tax, next 33 weeks = £140.98 or 90% average weekly earnings (whichever is less) 4) maternity allowance - fortnightly or monthly payment from government if you can't claim Statutory Maternity Pay eg/ haven't worked for employer for long enough, self-employed, average pay is less than £113 per week. £140.98 per week or 90% of your average weekly earnings (whichever is less) for up to 39 weeks. £27 per week for up to 14 weeks. 5) shared parental leave and pay - up to 50 weeks' parental leave and 37 weeks' pay shared with your partner if you're eligible 6) employment and support allowance (ESA) - benefit paid if you are unable to work because of sickness or disability & can't claim Statutory Sick Pay. Can be paid until 29th week of pregnancy 7) healthy start food vouchers - weekly vouchers for free: milk, fresh fruit & veg, infant formula, vitamins - can claim if at least 10 weeks pregnant or have a child under four and you and your family get one of the following benefits: income support, income-based Jobseeker's Allowance, income-related Employment Support Allowance, Child Tax Credit if your family's income is £16,190 or less etc. Pregnant women and children aged 1-4 get £3.10 per week, children <1 get £6.20 per week

Epidemiology, aetiology and characteristics of stress incontinence, urge incontinence and urinary frequency

prevalence of incontinence in general population of females - young adult: 20% to 30%; middle age: 30% to 40%; elderly: 30% to 50% 1) stress - a) epidemiology - 50% incontinent b) aetiology - insufficient closure during physical effort, pregnancy & childbirth, surgery, neurological, lifestyle, ageing & co-morbidity c) characteristics - urine leakage when: coughing, sneezing, laughing, standing up, lifting things, exercise, sex 2) urge - a) epidemiology - 22% of those incontinent b) aetiology - overactive detrusor/overactive bladder (OAB - contracts during filling phase), neurological: MS, nerve/brain/spinal cord damage c) characteristics - sudden urge to urinate & involuntary loss of urine at inappropriate times 3) frequency - a) epidemiology - 28% incontinent b) aetiology - cystitis (bacterial cystitis, interstitial cystitis); chemical cystitis (cyclophosphamide), urethritis, vaginitis or vulvar vestibulitis, UTI, detrusor instability, DM, pregnancy, prostate-related (prostatitis, BPH, prostate cancer), meds (diuretics, doxazosin), radiotherapy, schistosomiasis, bladder dysfunction, bladder tumours, urinary tract stones or foreign bodies c) characteristics - dysuria, urgency, haematuria, nocturia, hesitancy, dribbling, abdo pain, urinary incontinence (may suggest detrusor instability), look for distended bladder, may need VE in women or DRE in men

Long-term implications of pregnancy and postpartum problems

puerperium - from birth to 6 weeks after 1) anatomical changes: dec oestrogen levels = a) atrophy of lower genital tract - rapid dec in size of vulva, vagina and cervix b) dec secretions from vaginal & cervical glandular tissue = poorly lubricated vulva & vagina c) perineal lacerations heal rapidly in 14 days postpartum d) internal os of cervix largely closed by 48 hours, only admits tip of finger by 2 weeks postpartum i) external os - 'slit like' by 6 weeks postpartum 2) bleeding - a) heavy blood flow immediately after birth (lochia rubra) - then becomes pink, then heavy white vaginal discharge (lochia alba) until stops b) duration of lochia very variable - mostly stopped by 6 weeks c) passage of blood clots = abnormal & indicative of pathology 3) endometrial regeneration - new endometrium grows a) no lactation - new endometrium in place by 3 weeks postpartum & 1st menstruation 6 weeks postpartum b) lactation - suppresses ovarian activity so menstruation delayed for months 4) skeleton - ligament laxity slowly resolves through puerperium so s/s of sacroiliac joint dysfunction and Pelvic Girdle Pain (PGP/SPD) slowly resolve 5) psychological - hrs after birth may be characterised by PN 'high' a) +ve feelings: satisfaction, closeness to partner & mother, 'falling in love' with baby b) -ve feelings: dissatisfaction, disappointment or distress over delivery, anxiety about baby, rejection or ambivalence about baby, jealousy about baby c) in 16% women, early elation after childbirth can be extreme = hypomania - self-limiting condition 6) lactation - a) progesterone, oestrogen, prolactin, growth hormone and adrenal steroids = hypertrophy in pre-existing alveolar-lobular structures in breast b) formation of new alveolae by budding from milk ducts, with proliferation of milk-collecting ducts c) prolactin released by suckling at nipple sensitive PP - prolactin levels & milk production dependent on frequency & duration of suckling d) most prolactin in early puerperium & dec slowly 7) immunisation - a) all women sero-ve for rubella offered MMR vaccine after delivery b) anti-D Ig prophylaxis within 72 hrs of delivery if needed 8) midwives visit woman & baby in home prn, for at least 10 days post-delivery, up to 28 days 9) 6 week PN exam

Schedules of routine antenatal care

schedule depends on whether pregnancy is high, intermediate, or low risk 8-12 weeks - initial visit, confirm pregnancy, hx of risk factors, cervical smear, advice on health, smoking & diet, discuss & organise screenings, check maternal weight, advice on folic acid & iodine supplementation, dating scan 11-14 weeks - screening for trisomy - nuchal translucency scan & blood tests, dietary supplements of iron is anaemia 16 weeks - check all bloods, offer routine US anomaly scan 20 weeks - check US, BP, fundal height 24 weeks - BP, fundal height, feta activity 28 weeks - BP, fundal height, fetal activity, FBC, antibody screen - give anti-D if Rh -ve, GTT 32 weeks - BP, fundal height, fetal activity + growth scan if fetal growth is abnormal or low-lying placenta or anomaly scan 34 weeks - routine checks, 2nd anti-D dose, GBS vaginal & rectal swabs, FBC 36 weeks - BP, fundal height, fetal activity, determine position 38 & 40 weeks - routine checks, fetal activity, maternal wellbeing 41 weeks - routine checks, pelvic exam to assess cervical favorability, cardiotocograph, amniotic fluid index - consider induction of labour

Causes of stillbirth and perinatal morbidity/mortality

stillborn: 1) some - ?cause 2) many stillbirths linked to placental complications eg/ placental abruption 3) bleeding (haemorrhage) before or during labour 4) pre-eclampsia 5) a problem with the umbilical cord eg/ cord prolapse 6) intrahepatic cholestasis of pregnancy (ICP) or obstetric cholestasis 7) a genetic physical defect in the baby 8) pre-existing or gestational diabetes 9) infection in the mother that also affects the baby eg/ GBS, E.coli, klebsiella, enterococcus, haemophilus influenza, chlamydia, mycoplasma, rubella, influenza, parvovirus B19, coxsackie virus, CMV, herpes simplex, listeriosis, toxoplasmosis, malaria 10) factors that inc risk of stillborn: a) having twins or a multiple pregnancy b) having a baby who doesn't reach his or her growth potential in the womb c) >35 years of age d) smoking, drinking alcohol or misusing drugs while pregnant e) being obese (BMI >30) f) having a pre-existing physical health condition such as epilepsy perinatal mortality: 1) preterm birth is most common cause - 30% neonatal deaths. Infant RDS is the leading cause of death in preterm infants, affecting 1% of newborn infants 2) birth defects cause 21% neonatal death 3) others - meconium aspiration, sepsis, hyaline membrane disease, pulmonary haemorrhage

Ovulation induction

stimulation of ovulation by medication - usually stimulation of the development of ovarian follicles to reverse anovulation or oligoovulation: eg/ using clomifene citrate (or clomid), a selective oestrogen receptor modulator (SERM) that increases production of gonadotropins by inhibiting negative feedback from oestrogen on the hypothalamus can also be used in the sense of triggering oocyte release from relatively mature ovarian follicles. Often used in conjunction with ovulation induction: hCG low dose - injected after completed ovarian stimulation, causes ovulation 38-40 hours after a single injection; GnRH agonist (doesn't have risk of ovarian hyperstimulation syndrome) NB/ risk of ovarian hyperstimulation syndrome

Transmission, diagnosis, management and prevention of syphilis

syphilis (Treponema pallidum) 1) transmission - direct contact with a syphilitic sore/chancre 2) diagnosis - a) s/s of primary syphilis - development of a chancre - asymptomatic, bright red margin, clear discharge. Lasts 2-6 weeks b) s/s of secondary syphilis - I) multisystem involvement within 2 yrs eg/ headaches, malaise, fever, aches II) polymorphic rash on palms, soles and face - non-itchy, lymphadenopathy III) papules enlarge into condylomata lata (pink or grey) IV) 80% cases enter latent asymptomatic stage (50% persist for life) c) latent syphilis - +ve serological tests for syphilis with no clinical evidence of treponemal infection, early = within 2 yrs of infection, late is after 2 yrs d) late disease divided into gummatous disease (15%), CV disease (10%), late neurological complications (7%) e) s/s of tertiary syphilis - 3 clinical manifestations, which may co-exist: neurological syphilis - abnormal CSF but no neurological s/s, dorsal column loss, dementia & meningovascular involvement CV syphilis - aortitis, aortic regurg, aortic aneurysm and angina gummata - inflammatory fibrous nodules or plaques, locally destructive, most commonly affect bone and skin f) treponemal enzyme immunoassay (EIA) - (IgM or IgG), chemiluminescent assay (CLIA), haemagglutination assay (TPHA etc 3) mx - high risk of re-infection so regular STI screening a) primary, secondary, early latent syphilis: benzathine penicillin (IM) I) neurosyphilis: procaine penicillin IM + oral probenecid b) pregnancy: 1st & 2nd trimesters - 1 benzathine penicillin; 3rd - 2 doses benzathine penicillin one week apart 4) prevention - tx asymptomatic contacts, condoms, screening if pregnant

Neonatal adaptation to extrauterine life

transition from a fetus to a newborn is the most complex physiologic adaptation that occurs in human experience major immediate adaptations: 1) establishment of air breathing concurrently with changes in pressures & flows within CV system a) clearance of fetal lung fluid, surfactant secretion, and breathing b) transition of fetal to neonatal circulation c) dec pulmonary vascular resistance & inc pulmonary blood flow 2) changes in endocrine function a) cortisol - major regulatory hormone for terminal maturation of fetus & for neonatal adaption at birth I) "cortisol surge" initiated with switch from maternal-transplacental derived corticosteroids to ability of fetal adrenal to synthesize and release cortisol under fetal hypothalamic control. Fetal cortisol levels ~5-10ug/ml, then inc after 30 weeks to ~45ug/ml prior to labor at term, inc more in labor to ~200ug/ml several hours after term delivery II) cortisol + thyroid hormones activates the sodium pump that clears fetal lung fluid at birth III) effects of cortisol - lung maturation - anatomy and surfactant, clearance of fetal lung fluid, inc β receptor density, gut functional maturation, maturation of thyroid axis, regulate catecholamine release, control energy substrate metabolism b) catecholamines - catecholamine surge is primarily responsible for inc BP following birth, adaption of energy metabolism with support of the primary substrates for metabolism after birth - glucose and fatty acids, and for initiating thermogenesis from brown fat. c) thyroid hormones 3) changes in substrate metabolism 4) changes in thermogenesis - at birth the sympathetic release from inc oxygenation, ventilation, cord occlusion and a cold stimulus to the skin activates thermogenesis by brown adipose tissue - generates heat by uncoupling oxidative metabolism from ATP synthesis in the mitochondria, with the release of heat 5) CV adaptations - changes due to removal of low resistance placenta as source of fetal gas exchange and nutrition a) inc cardiac output (almost doubles) b) soon after birth circulation changes from "parallel" to "series", where RV = LV output c) blood flow inc to pulmonary circulation d) shortly after birth functional closure of ductus arteriosus begins

Transmission, diagnosis, management and prevention of trichomonas vaginalis

trichomonas vaginalis 1) transmission - parasite passes during sex 2) diagnosis - a) women: vaginal discharge - usually a frothy yellow, vulval itching, dysuria or offensive odour, lower abdo discomfort, signs of local inflammation with vulvitis and vaginitis, cervicitis ('strawberry cervix') b) men - usually asymptomatic, otherwise urethritis - dysuria & urethral discharge c) ix - high vaginal swab from posterior fornix followed by NAATs 3) mx - both partners treated with metronidazole, sex avoided for at least 1 week following treatment NB/ metronidazole can be used in all stages of pregnancy and breast-feeding 4) prevention - avoid sex with infected individuals, condom use


Related study sets

Lesson 2 - The Rights and Obligations of Citizenship

View Set

12 - Security Solutions for Cloud and Automation

View Set

Financial Management Exam 2 Whitledge

View Set

Unit 7 - Atmospheric Pollution Test

View Set

Back + Spinal Cord - MCQ Questions

View Set